Slot System
Featured Buckets
Featured Buckets Admin

Should breastfeeding babies be given pacifiers?

Article Type
Changed
Mon, 01/14/2019 - 11:34
Display Headline
Should breastfeeding babies be given pacifiers?
PRACTICE CHANGER

Do not discourage the use of pacifiers by healthy infants who are breastfeeding.1

STRENGTH OF RECOMMENDATION

A: Based on a good-quality meta-analysis

Jaafar SH, Jahanfar S, Angolkar M, et al. Pacifier use versus no pacifier use in breastfeeding term infants for increasing duration of breastfeeding. Cochrane Database Syst Rev. 2011;(3):CD007202.

 

ILLUSTRATIVE CASE

A 28-year-old first-time mother brings her 1-week-old full-term, healthy daughter to your clinic for a routine check-up. The baby is growing as expected. The mother is breastfeeding exclusively and would like to continue until the baby is at least 6 months old. She has begun giving the infant a pacifier and asks whether that’s OK. What should you tell her?

Breast milk is superior to formula for infant nutrition,2,3 and the American Academy of Pediatrics (AAP), World Health Organization (WHO), and UNICEF recommend exclusive breastfeeding through the first 6 months of life.4,5 “Ten steps to successful breastfeeding,” a document developed by the WHO and UNICEF, specifically states that artificial teats and pacifiers should not be given to breastfeeding infants.5

Concern about pacifiers for breastfeeding infants focuses on “nipple confusion”—that is, that pacifiers (and supplemental bottles) do not facilitate effective breast sucking and may contribute to incorrect latch.6,7 Findings from earlier observational stud-ies suggest that early exposure to pacifiers leads to cessation of exclusive breastfeeding by 3 to 6 months and an end to all breastfeeding by 12 months.8,9

Pacifiers have become a cultural norm in many parts of the world,10 and their use appears to be associated with a decrease in the incidence of sudden infant death syndrome (SIDS).11 But both the AAP and the American Academy of Family Physicians recommend delaying paci-fier use until breastfeeding is established.4,12

STUDY SUMMARY: Pacifier or no pacifier— no significant difference

Cochrane reviewers conducted a meta-analysis of randomized controlled trials (RCTs) that assessed the effects of pacifier use on healthy full-term infants whose mothers had initiated breastfeeding and intended to exclusively breastfeed.1

The primary outcome was the duration of breastfeeding, as measured by (1) mean duration of full breastfeeding (in months); (2) mean duration of any or partial breastfeeding; or (3) the prevalence or proportion of infants who were fully or partially breastfed at 3, 4, and 6 months of age. Secondary outcomes were (1) rate of breastfeeding difficulties (cracked nipples, breast engorgement, mastitis); (2) maternal satisfaction and level of confidence in parenting; (3) frequency of infant crying and fussiness; and (4) infant health, including SIDS, oral candidiasis, otitis media, and dental malocclusion. Infants who were given pacifiers (ie, those who had unrestricted or actively encouraged pacifier use) were compared with infants who were not given pacifiers (ie, whose mothers were advised against pacifier use).

Two studies, with a combined total of 1302 infants, were included in the meta-analysis (a third was excluded because the method by which allocation was concealed was un-clear). Both reported blinding of the research nurses and outcome assessors. Blinding of par-ticipants was not feasible.

Each trial measured at least one primary outcome. The dropout rate was <10% in each arm of both studies.

When pacifier use was compared with no pacifier use, no significant difference was found in the proportion of infants who were exclusively breastfed at 3 months (risk ratio [RR]=1.00; 95% confidence interval [CI], 0.95-1.06) or 4 months of age (RR=0.99; 95% CI, 0.92-1.06). Nor was there a significant difference in the proportion of infants who were partially breastfed at 3 months (RR=1.00; 95% CI, 0.97-1.02) or 4 months (RR=1.01; 95% CI, 0.98-1.03).

Neither study reported on any of the secondary outcomes.

 

 

 

WHAT’S NEW: Now we know: Pacifier use by breastfeeding infants is fine

This meta-analysis shows that pacifier use does not decrease breastfeeding duration in full-term infants. The new evidence contradicts current WHO recom-mendations, however, which are based on less rigorous studies.8,9,13,14 The AAP now recommends that pacifier use be implemented after breastfeeding is established.4 Based on the evidence, we think mothers who are motivated to breastfeed their infants should be allowed to make their own decisions regarding pacifier use, and pacifier use should not be discouraged.

CAVEATS: Effects on infant health still unaddressed

This meta-analysis did not report on the potential harms of pacifi-ers to infants >4 months old or to their lactating mothers. Potential problems of prolonged pacifier use (>4 months), such as increased risk of recurrent acute otitis media, oral candidiasis, and dental malocclusion, should be addressed with mothers, but should not lead to discouraging pacifier use in early infancy.15-17

CHALLENGES TO IMPLEMENTATION: There aren’t any

We see no challenges to implementation of this practice changer.

Acknowledgement

The PURLs Surveillance System is supported in part by Grant Number UL1RR024999 from the National Center for Research Resources, a Clinical Translational Science Award to the University of Chicago. The content is solely the responsibility of the authors and does not necessarily represent the official views of the National Center for Research Resources or the National Institutes of Health.

Click here to view PURL METHODOLOGY

References

1. Jaafar SH, Jahanfar S, Angolkar M, et al. Pacifier use versus no pacifier use in breastfeeding term infants for increasing duration of breastfeeding. Cochrane Database Syst Rev. 2011;(3):CD007202.-

2. Chandra RK. Prospective studies on the effect of breastfeeding on the incidence of infection and allergy. Acta Paediatr Scand. 1979;68:691-694.

3. Oddy WH. Breastfeeding protects against illness and infection in infants and children: a review of the evidence. Breastfeed Rev. 2001;9:11-18.

4. American Academy of Pediatrics Section on breastfeeding. Policy Statement. Breastfeeding and the use of human milk. Pediatrics. 2012;129:e827-e841.

5. World Health Organization. Ten steps to successful breastfeeding. WHO/UNICEF statement. Available at: http://www.unicef.org/newsline/tenstps.htm. Accessed April 12, 2012.

6. Gomes CF, Trezza EM, Murade EC, et al. Surface electromyography of facial muscles during natural and artificial feeding of infants. J Pediatr (Rio J). 2006;82:103-109.

7. Righard L. Are breastfeeding problems related to incorrect breastfeeding technique and the use of pacifiers and bottles? Birth. 1998;25:40-44.

8. Mascarenhes ML, Albernaz EP, DaSilva MB, et al. Prevalence of exclusive breastfeeding and its determiners in the first 3 months of life in the South of Brazil. J Pediatr (Rio J). 2006;82:289-294.

9. Scott JA, Binns CW, Oddy WH, et al. Predictors of breastfeeding duration: evidence from a cohort study. Pediatrics. 2006;117:e646-e655.

10. Barros FC, Victora CG, Semer TC, et al. Use of pacifiers is associated with decreased breast-feeding duration. Pediatrics. 1995;95:497-499.

11. American Academy of Pediatrics Task Force on Sudden Infant Death Syndrome. The changing concept of sudden infant death syndrome: diagnostic coding shifts, controversies regarding the sleeping environment, and new variables to consider in reducing risk. Pediatrics. 2005;116:1245-1255.

12. American Academy of Family Physicians. Breastfeeding, family physicians supporting (position paper). Available at: www.aafp.org/online/en/home/policy/policies/b/breastfeedingpositionpaper.html. Accessed April 17, 2012.

13. Howard CR, Howard FM, Lanphear B, et al. The effects of early pacifier use on breastfeeding duration. Pediatrics. 1999;103:E133.-

14. Karabulut E, Yalcin SS, Ozdemir-Geyik P, et al. Effect of pacifier use on exclusive and any breastfeeding: a meta-analysis. Turk J Pediatr. 2009;51:35-43.

15. Jackson JM, Mourino AP. Pacifier use and otitis media in infants twelve months of age or younger. Pediatr Dent. 1999;21:255-260.

16. Darwazeh AM, al-Bashir A. Oral candidal flora in healthy infants. J Oral Pathol Med. 1995;24:361-364.

17. Caglar E, Larsson E, Andersson EM, et al. Feeding, artificial sucking habits, and malocclusions in 3-year-old girls in different regions of the world. J Dent Child (Chic). 2005;72:25-30.

Author and Disclosure Information

Laura Wellington, MD
University of Minnesota North Memorial Family Medicine Residency Program, Minneapolis

Shailendra Prasad, MBBS, MPH
University of Minnesota North Memorial Family Medicine Residency Program, Minneapolis

PURLs EDITOR
Bernard Ewigman, MD, MSPH
University of Chicago

Issue
The Journal of Family Practice - 61(05)
Publications
Topics
Page Number
E1-E3
Legacy Keywords
Laura Wellington;MD; Shailendra Prasad;MBBS;MPH; pacifier use; breast milk; supplemental bottles; nipple confusion; duration of breastfeeding
Sections
Author and Disclosure Information

Laura Wellington, MD
University of Minnesota North Memorial Family Medicine Residency Program, Minneapolis

Shailendra Prasad, MBBS, MPH
University of Minnesota North Memorial Family Medicine Residency Program, Minneapolis

PURLs EDITOR
Bernard Ewigman, MD, MSPH
University of Chicago

Author and Disclosure Information

Laura Wellington, MD
University of Minnesota North Memorial Family Medicine Residency Program, Minneapolis

Shailendra Prasad, MBBS, MPH
University of Minnesota North Memorial Family Medicine Residency Program, Minneapolis

PURLs EDITOR
Bernard Ewigman, MD, MSPH
University of Chicago

PRACTICE CHANGER

Do not discourage the use of pacifiers by healthy infants who are breastfeeding.1

STRENGTH OF RECOMMENDATION

A: Based on a good-quality meta-analysis

Jaafar SH, Jahanfar S, Angolkar M, et al. Pacifier use versus no pacifier use in breastfeeding term infants for increasing duration of breastfeeding. Cochrane Database Syst Rev. 2011;(3):CD007202.

 

ILLUSTRATIVE CASE

A 28-year-old first-time mother brings her 1-week-old full-term, healthy daughter to your clinic for a routine check-up. The baby is growing as expected. The mother is breastfeeding exclusively and would like to continue until the baby is at least 6 months old. She has begun giving the infant a pacifier and asks whether that’s OK. What should you tell her?

Breast milk is superior to formula for infant nutrition,2,3 and the American Academy of Pediatrics (AAP), World Health Organization (WHO), and UNICEF recommend exclusive breastfeeding through the first 6 months of life.4,5 “Ten steps to successful breastfeeding,” a document developed by the WHO and UNICEF, specifically states that artificial teats and pacifiers should not be given to breastfeeding infants.5

Concern about pacifiers for breastfeeding infants focuses on “nipple confusion”—that is, that pacifiers (and supplemental bottles) do not facilitate effective breast sucking and may contribute to incorrect latch.6,7 Findings from earlier observational stud-ies suggest that early exposure to pacifiers leads to cessation of exclusive breastfeeding by 3 to 6 months and an end to all breastfeeding by 12 months.8,9

Pacifiers have become a cultural norm in many parts of the world,10 and their use appears to be associated with a decrease in the incidence of sudden infant death syndrome (SIDS).11 But both the AAP and the American Academy of Family Physicians recommend delaying paci-fier use until breastfeeding is established.4,12

STUDY SUMMARY: Pacifier or no pacifier— no significant difference

Cochrane reviewers conducted a meta-analysis of randomized controlled trials (RCTs) that assessed the effects of pacifier use on healthy full-term infants whose mothers had initiated breastfeeding and intended to exclusively breastfeed.1

The primary outcome was the duration of breastfeeding, as measured by (1) mean duration of full breastfeeding (in months); (2) mean duration of any or partial breastfeeding; or (3) the prevalence or proportion of infants who were fully or partially breastfed at 3, 4, and 6 months of age. Secondary outcomes were (1) rate of breastfeeding difficulties (cracked nipples, breast engorgement, mastitis); (2) maternal satisfaction and level of confidence in parenting; (3) frequency of infant crying and fussiness; and (4) infant health, including SIDS, oral candidiasis, otitis media, and dental malocclusion. Infants who were given pacifiers (ie, those who had unrestricted or actively encouraged pacifier use) were compared with infants who were not given pacifiers (ie, whose mothers were advised against pacifier use).

Two studies, with a combined total of 1302 infants, were included in the meta-analysis (a third was excluded because the method by which allocation was concealed was un-clear). Both reported blinding of the research nurses and outcome assessors. Blinding of par-ticipants was not feasible.

Each trial measured at least one primary outcome. The dropout rate was <10% in each arm of both studies.

When pacifier use was compared with no pacifier use, no significant difference was found in the proportion of infants who were exclusively breastfed at 3 months (risk ratio [RR]=1.00; 95% confidence interval [CI], 0.95-1.06) or 4 months of age (RR=0.99; 95% CI, 0.92-1.06). Nor was there a significant difference in the proportion of infants who were partially breastfed at 3 months (RR=1.00; 95% CI, 0.97-1.02) or 4 months (RR=1.01; 95% CI, 0.98-1.03).

Neither study reported on any of the secondary outcomes.

 

 

 

WHAT’S NEW: Now we know: Pacifier use by breastfeeding infants is fine

This meta-analysis shows that pacifier use does not decrease breastfeeding duration in full-term infants. The new evidence contradicts current WHO recom-mendations, however, which are based on less rigorous studies.8,9,13,14 The AAP now recommends that pacifier use be implemented after breastfeeding is established.4 Based on the evidence, we think mothers who are motivated to breastfeed their infants should be allowed to make their own decisions regarding pacifier use, and pacifier use should not be discouraged.

CAVEATS: Effects on infant health still unaddressed

This meta-analysis did not report on the potential harms of pacifi-ers to infants >4 months old or to their lactating mothers. Potential problems of prolonged pacifier use (>4 months), such as increased risk of recurrent acute otitis media, oral candidiasis, and dental malocclusion, should be addressed with mothers, but should not lead to discouraging pacifier use in early infancy.15-17

CHALLENGES TO IMPLEMENTATION: There aren’t any

We see no challenges to implementation of this practice changer.

Acknowledgement

The PURLs Surveillance System is supported in part by Grant Number UL1RR024999 from the National Center for Research Resources, a Clinical Translational Science Award to the University of Chicago. The content is solely the responsibility of the authors and does not necessarily represent the official views of the National Center for Research Resources or the National Institutes of Health.

Click here to view PURL METHODOLOGY

PRACTICE CHANGER

Do not discourage the use of pacifiers by healthy infants who are breastfeeding.1

STRENGTH OF RECOMMENDATION

A: Based on a good-quality meta-analysis

Jaafar SH, Jahanfar S, Angolkar M, et al. Pacifier use versus no pacifier use in breastfeeding term infants for increasing duration of breastfeeding. Cochrane Database Syst Rev. 2011;(3):CD007202.

 

ILLUSTRATIVE CASE

A 28-year-old first-time mother brings her 1-week-old full-term, healthy daughter to your clinic for a routine check-up. The baby is growing as expected. The mother is breastfeeding exclusively and would like to continue until the baby is at least 6 months old. She has begun giving the infant a pacifier and asks whether that’s OK. What should you tell her?

Breast milk is superior to formula for infant nutrition,2,3 and the American Academy of Pediatrics (AAP), World Health Organization (WHO), and UNICEF recommend exclusive breastfeeding through the first 6 months of life.4,5 “Ten steps to successful breastfeeding,” a document developed by the WHO and UNICEF, specifically states that artificial teats and pacifiers should not be given to breastfeeding infants.5

Concern about pacifiers for breastfeeding infants focuses on “nipple confusion”—that is, that pacifiers (and supplemental bottles) do not facilitate effective breast sucking and may contribute to incorrect latch.6,7 Findings from earlier observational stud-ies suggest that early exposure to pacifiers leads to cessation of exclusive breastfeeding by 3 to 6 months and an end to all breastfeeding by 12 months.8,9

Pacifiers have become a cultural norm in many parts of the world,10 and their use appears to be associated with a decrease in the incidence of sudden infant death syndrome (SIDS).11 But both the AAP and the American Academy of Family Physicians recommend delaying paci-fier use until breastfeeding is established.4,12

STUDY SUMMARY: Pacifier or no pacifier— no significant difference

Cochrane reviewers conducted a meta-analysis of randomized controlled trials (RCTs) that assessed the effects of pacifier use on healthy full-term infants whose mothers had initiated breastfeeding and intended to exclusively breastfeed.1

The primary outcome was the duration of breastfeeding, as measured by (1) mean duration of full breastfeeding (in months); (2) mean duration of any or partial breastfeeding; or (3) the prevalence or proportion of infants who were fully or partially breastfed at 3, 4, and 6 months of age. Secondary outcomes were (1) rate of breastfeeding difficulties (cracked nipples, breast engorgement, mastitis); (2) maternal satisfaction and level of confidence in parenting; (3) frequency of infant crying and fussiness; and (4) infant health, including SIDS, oral candidiasis, otitis media, and dental malocclusion. Infants who were given pacifiers (ie, those who had unrestricted or actively encouraged pacifier use) were compared with infants who were not given pacifiers (ie, whose mothers were advised against pacifier use).

Two studies, with a combined total of 1302 infants, were included in the meta-analysis (a third was excluded because the method by which allocation was concealed was un-clear). Both reported blinding of the research nurses and outcome assessors. Blinding of par-ticipants was not feasible.

Each trial measured at least one primary outcome. The dropout rate was <10% in each arm of both studies.

When pacifier use was compared with no pacifier use, no significant difference was found in the proportion of infants who were exclusively breastfed at 3 months (risk ratio [RR]=1.00; 95% confidence interval [CI], 0.95-1.06) or 4 months of age (RR=0.99; 95% CI, 0.92-1.06). Nor was there a significant difference in the proportion of infants who were partially breastfed at 3 months (RR=1.00; 95% CI, 0.97-1.02) or 4 months (RR=1.01; 95% CI, 0.98-1.03).

Neither study reported on any of the secondary outcomes.

 

 

 

WHAT’S NEW: Now we know: Pacifier use by breastfeeding infants is fine

This meta-analysis shows that pacifier use does not decrease breastfeeding duration in full-term infants. The new evidence contradicts current WHO recom-mendations, however, which are based on less rigorous studies.8,9,13,14 The AAP now recommends that pacifier use be implemented after breastfeeding is established.4 Based on the evidence, we think mothers who are motivated to breastfeed their infants should be allowed to make their own decisions regarding pacifier use, and pacifier use should not be discouraged.

CAVEATS: Effects on infant health still unaddressed

This meta-analysis did not report on the potential harms of pacifi-ers to infants >4 months old or to their lactating mothers. Potential problems of prolonged pacifier use (>4 months), such as increased risk of recurrent acute otitis media, oral candidiasis, and dental malocclusion, should be addressed with mothers, but should not lead to discouraging pacifier use in early infancy.15-17

CHALLENGES TO IMPLEMENTATION: There aren’t any

We see no challenges to implementation of this practice changer.

Acknowledgement

The PURLs Surveillance System is supported in part by Grant Number UL1RR024999 from the National Center for Research Resources, a Clinical Translational Science Award to the University of Chicago. The content is solely the responsibility of the authors and does not necessarily represent the official views of the National Center for Research Resources or the National Institutes of Health.

Click here to view PURL METHODOLOGY

References

1. Jaafar SH, Jahanfar S, Angolkar M, et al. Pacifier use versus no pacifier use in breastfeeding term infants for increasing duration of breastfeeding. Cochrane Database Syst Rev. 2011;(3):CD007202.-

2. Chandra RK. Prospective studies on the effect of breastfeeding on the incidence of infection and allergy. Acta Paediatr Scand. 1979;68:691-694.

3. Oddy WH. Breastfeeding protects against illness and infection in infants and children: a review of the evidence. Breastfeed Rev. 2001;9:11-18.

4. American Academy of Pediatrics Section on breastfeeding. Policy Statement. Breastfeeding and the use of human milk. Pediatrics. 2012;129:e827-e841.

5. World Health Organization. Ten steps to successful breastfeeding. WHO/UNICEF statement. Available at: http://www.unicef.org/newsline/tenstps.htm. Accessed April 12, 2012.

6. Gomes CF, Trezza EM, Murade EC, et al. Surface electromyography of facial muscles during natural and artificial feeding of infants. J Pediatr (Rio J). 2006;82:103-109.

7. Righard L. Are breastfeeding problems related to incorrect breastfeeding technique and the use of pacifiers and bottles? Birth. 1998;25:40-44.

8. Mascarenhes ML, Albernaz EP, DaSilva MB, et al. Prevalence of exclusive breastfeeding and its determiners in the first 3 months of life in the South of Brazil. J Pediatr (Rio J). 2006;82:289-294.

9. Scott JA, Binns CW, Oddy WH, et al. Predictors of breastfeeding duration: evidence from a cohort study. Pediatrics. 2006;117:e646-e655.

10. Barros FC, Victora CG, Semer TC, et al. Use of pacifiers is associated with decreased breast-feeding duration. Pediatrics. 1995;95:497-499.

11. American Academy of Pediatrics Task Force on Sudden Infant Death Syndrome. The changing concept of sudden infant death syndrome: diagnostic coding shifts, controversies regarding the sleeping environment, and new variables to consider in reducing risk. Pediatrics. 2005;116:1245-1255.

12. American Academy of Family Physicians. Breastfeeding, family physicians supporting (position paper). Available at: www.aafp.org/online/en/home/policy/policies/b/breastfeedingpositionpaper.html. Accessed April 17, 2012.

13. Howard CR, Howard FM, Lanphear B, et al. The effects of early pacifier use on breastfeeding duration. Pediatrics. 1999;103:E133.-

14. Karabulut E, Yalcin SS, Ozdemir-Geyik P, et al. Effect of pacifier use on exclusive and any breastfeeding: a meta-analysis. Turk J Pediatr. 2009;51:35-43.

15. Jackson JM, Mourino AP. Pacifier use and otitis media in infants twelve months of age or younger. Pediatr Dent. 1999;21:255-260.

16. Darwazeh AM, al-Bashir A. Oral candidal flora in healthy infants. J Oral Pathol Med. 1995;24:361-364.

17. Caglar E, Larsson E, Andersson EM, et al. Feeding, artificial sucking habits, and malocclusions in 3-year-old girls in different regions of the world. J Dent Child (Chic). 2005;72:25-30.

References

1. Jaafar SH, Jahanfar S, Angolkar M, et al. Pacifier use versus no pacifier use in breastfeeding term infants for increasing duration of breastfeeding. Cochrane Database Syst Rev. 2011;(3):CD007202.-

2. Chandra RK. Prospective studies on the effect of breastfeeding on the incidence of infection and allergy. Acta Paediatr Scand. 1979;68:691-694.

3. Oddy WH. Breastfeeding protects against illness and infection in infants and children: a review of the evidence. Breastfeed Rev. 2001;9:11-18.

4. American Academy of Pediatrics Section on breastfeeding. Policy Statement. Breastfeeding and the use of human milk. Pediatrics. 2012;129:e827-e841.

5. World Health Organization. Ten steps to successful breastfeeding. WHO/UNICEF statement. Available at: http://www.unicef.org/newsline/tenstps.htm. Accessed April 12, 2012.

6. Gomes CF, Trezza EM, Murade EC, et al. Surface electromyography of facial muscles during natural and artificial feeding of infants. J Pediatr (Rio J). 2006;82:103-109.

7. Righard L. Are breastfeeding problems related to incorrect breastfeeding technique and the use of pacifiers and bottles? Birth. 1998;25:40-44.

8. Mascarenhes ML, Albernaz EP, DaSilva MB, et al. Prevalence of exclusive breastfeeding and its determiners in the first 3 months of life in the South of Brazil. J Pediatr (Rio J). 2006;82:289-294.

9. Scott JA, Binns CW, Oddy WH, et al. Predictors of breastfeeding duration: evidence from a cohort study. Pediatrics. 2006;117:e646-e655.

10. Barros FC, Victora CG, Semer TC, et al. Use of pacifiers is associated with decreased breast-feeding duration. Pediatrics. 1995;95:497-499.

11. American Academy of Pediatrics Task Force on Sudden Infant Death Syndrome. The changing concept of sudden infant death syndrome: diagnostic coding shifts, controversies regarding the sleeping environment, and new variables to consider in reducing risk. Pediatrics. 2005;116:1245-1255.

12. American Academy of Family Physicians. Breastfeeding, family physicians supporting (position paper). Available at: www.aafp.org/online/en/home/policy/policies/b/breastfeedingpositionpaper.html. Accessed April 17, 2012.

13. Howard CR, Howard FM, Lanphear B, et al. The effects of early pacifier use on breastfeeding duration. Pediatrics. 1999;103:E133.-

14. Karabulut E, Yalcin SS, Ozdemir-Geyik P, et al. Effect of pacifier use on exclusive and any breastfeeding: a meta-analysis. Turk J Pediatr. 2009;51:35-43.

15. Jackson JM, Mourino AP. Pacifier use and otitis media in infants twelve months of age or younger. Pediatr Dent. 1999;21:255-260.

16. Darwazeh AM, al-Bashir A. Oral candidal flora in healthy infants. J Oral Pathol Med. 1995;24:361-364.

17. Caglar E, Larsson E, Andersson EM, et al. Feeding, artificial sucking habits, and malocclusions in 3-year-old girls in different regions of the world. J Dent Child (Chic). 2005;72:25-30.

Issue
The Journal of Family Practice - 61(05)
Issue
The Journal of Family Practice - 61(05)
Page Number
E1-E3
Page Number
E1-E3
Publications
Publications
Topics
Article Type
Display Headline
Should breastfeeding babies be given pacifiers?
Display Headline
Should breastfeeding babies be given pacifiers?
Legacy Keywords
Laura Wellington;MD; Shailendra Prasad;MBBS;MPH; pacifier use; breast milk; supplemental bottles; nipple confusion; duration of breastfeeding
Legacy Keywords
Laura Wellington;MD; Shailendra Prasad;MBBS;MPH; pacifier use; breast milk; supplemental bottles; nipple confusion; duration of breastfeeding
Sections
PURLs Copyright

Copyright © 2012 The Family Physicians Inquiries Network. All rights reserved.

Disallow All Ads
Alternative CME

Pulse oximetry for newborns: Should it be routine?

Article Type
Changed
Mon, 01/14/2019 - 11:34
Display Headline
Pulse oximetry for newborns: Should it be routine?

 

PRACTICE CHANGER

Ensure that all newborns undergo pulse oximetry screening before discharge—and that abnormal results are immediately followed up with echocardiography.1

STRENGTH OF RECOMMENDATION

B: Based on a single cohort study consistent with multiple studies in other populations.

Ewer AK, Middleton LJ, Furmston AT, et al. Pulse oximetry screening for congenital heart defects in newborn infants (PulseOx): a test accuracy study. Lancet. 2011;378:785-794.

ILLUSTRATIVE CASE

A healthy, full-term baby is admitted to the newborn nursery. Antenatal surveillance, including routine ultrasound, was normal, as are physical examinations, both on admittance to the nursery and on the following day. Should the infant undergo pulse oximetry screening prior to discharge?

Congenital heart defects (CHD) are a leading cause of infant deaths in the developed world, occurring in approximately 9 of every 1000 live births.2 Roughly a quarter of those affected will have CHD serious enough to require either surgery or catheterization within the first year of life. These newborns are susceptible to sudden cardiovascular collapse due to changes in pulmonary vascular resistance and closure of the ductus arteriosus2—changes that often occur after the babies have gone home.

Delayed diagnosis is linked to worsening disease
A study evaluating 286 neonates admitted for cardiac surgery found that delayed diagnosis of CHD was associated with a worse preoperative condition. Cardiovascular compromise and end-organ dysfunction were most common in infants who presented with symptoms after they had gone home, the researchers found.3

In the past, screening for CHD hinged on mid-trimester ultrasound and postnatal physical examination. However, these methods do not reliably detect the condition in a timely fashion.4,5 More recently, pulse oximetry has been used for screening.

International studies prompt US recommendation
In a prospective multicenter trial in Saxony, Germany, more than 41,000 infants born between 2006 and 2008 underwent pulse oximetry screening at 24 to 72 hours of life.5 If the oxygen saturation was ≤95% and confirmed an hour later, echocardiography was performed. Pulse oximetry screening yielded true-positive results in 14 cases, false-positive results in 40, and false-negative results in 4. Sensitivity and specificity were 77.8% and 99.9%, respectively.

In another German study, 3364 term neonates underwent pulse oximetry screening between 6 and 36 hours of life.6 Eighteen neonates (0.5%) had abnormal results, 9 (50%) of whom were found to have heart defects. In this study, pulse oximetry had a sensitivity of 82% and a specificity of 99.9%.

A cohort study of 39,821 newborns in a single region of Sweden found that combining physical examination with pulse oximetry screening had a sensitivity of 82.8% and a specificity of 98%.7 No infants who underwent screening died from undiagnosed ductus arteriosus-dependent lung circulation, compared with 5 such deaths in regions where pulse oximetry screening was not done.

 

Encouraged by these findings, in late 2011 the US Secretary of Health and Human Services, with strong backing from the American Academy of Pediatrics, recommended universal pulse oximetry screening to detect critical CHD.8 The study detailed below took another look at its efficacy.

STUDY SUMMARY: Detection rate is higher for critical heart defects

The cohort study by Ewer et al enrolled 20,055 neonates born at >34 weeks’ gestation.1 All were screened with pulse oximetry on the right hand and on either foot and had a physical exam within their first 24 hours (The TABLE describes the screening protocol). Infants with a normal pulse oximetry and normal clinical exam were followed for a year to identify late-presenting heart defects.

One hundred ninety-five of the neonates who were screened had abnormal pulse oximetry test results; of these, 26 (13%) were found to have either critical (requiring intervention <28 days) or major (requiring intervention <12 months of age) CHD. Of the 169 infants who had positive pulse oximetry results but did not have critical or major heart defects, 6 were found to have less serious heart defects and 40 had infective or respiratory disorders that also required medical intervention.

Among the 19,860 infants with normal pulse oximetry, 27 (0.1%) were found to have either critical or major heart defects.

Pulse oximetry had a sensitivity of 75% (95% confidence interval [CI], 53.3-90.2) and a specificity of 99.1% (95% CI, 98.98-99.24) for detecting critical CHD. Sensitivity of pulse oximetry for all major CHD was 49% (95% CI, 35.0-63.2) and the specificity was 99.2% (95% CI, 99.02-99.28). The specificity may have been better if screening had been done after 24 hours of life; as seen in other studies,5,6 screening within the first 24 hours leads to more false-positive results.

 

 

The detection rate for critical CHD was higher than for major defects. However, most of the defects missed by screening were noncritical lesions, eg, ventricular septal defects.

The authors estimated that in a population of 100,000 newborns, about 120 would have critical CHD, and 90 of those 120 cases would be detected by pulse oximetry. There would be 843 false positives (although 229 of the infants with false-positive results would have other noncardiac conditions). It would be necessary to perform 10.4 echocardiograms to detect one patient with critical CHD.

WHAT’S NEW: A stronger case for newborn pulse oximetry screening

Pulse oximetry prior to discharge from the newborn nursery is not performed routinely in all institutions. And even when screening is done, there may not be a protocol addressing abnormal results. Pulse oximetry is a safe, noninvasive, inexpensive, and reasonably sensitive test that will detect many cases of critical CHD, some of which will not be diagnosed antenatally. Earlier diagnosis of CHD may lead to earlier interventions and improved patient outcomes.

CAVEATS: Timing of screening may alter results

This trial was a cohort study, not a randomized controlled trial (RCT)—the gold standard method of validating a screening test. It is unlikely, however, that an RCT will ever be done.

Screening occurred within the first 24 hours; other investigators have screened >24 hours (up to 38 hours), which may have better results. The critical lesions most likely to be missed by pulse oximetry screening were those causing obstruction to the aortic arch,1 a finding that was also seen in other studies.5,7

TABLE
Screening newborns for congenital heart defects: The protocol
1

 

Pulse oximetry outcome*Physical exam outcomeNext step
NormalNormalNo further action
AbnormalNormalRepeat pulse oximetry in 2 h:
  • normal=no further action
  • abnormal=echocardiogram
AbnormalAbnormalechocardiogram
*Test is normal if pulse oximetry >95% on right hand and difference between the right hand and either foot is <2%.

CHALLENGES TO IMPLEMENTATION: Early discharge, lack of equipment may interfere

Determining the timing of pulse oximetry screening is important. That’s particularly true because early discharge is a common practice, and early screening may increase the number of false-positive results. As a screening tool, pulse oximetry is inexpensive, and follow-up echocardiography—which is needed to exclude serious cases of CHD in patients with positive pulse oximetry—is noninvasive and relatively inexpensive. Echocardiography is not readily available in all communities, however, and transportation to a facility that offers this test would likely increase the cost of screening.

Acknowledgement

The PURLs Surveillance System is supported in part by Grant Number UL1RR024999 from the National Center for Research Resources, a Clinical Translational Science Award to the University of Chicago. The content is solely the responsibility of the authors and does not necessarily represent the official views of the National Center for Research Resources or the National Institutes of Health.

References

 

1. Ewer AK, Middleton LJ, Furmston AT, et al. Pulse oximetry screening for congenital heart defects in newborn infants (PulseOx): a test accuracy study. Lancet. 201;378:785-794.

2. Mahle WT, Newburger JW, Matherne GP, et al. Role of pulse oximetry in examining newborns for congenital heart disease: a scientific statement from the AHA and AAP. Pediatrics. 2009;124:823-836.

3. Brown KL, Ridout DA, Hoskote A, et al. Delayed diagnosis of congenital heart disease worsens preoperative condition and outcome of surgery in neonates. Heart. 2006;92:1298-1302.

4. Acharya G, Sitras V, Maltau JM, et al. Major congenital heart disease in Northern Norway: shortcomings of pre- and postnatal diagnosis. Acta Obstet Gynecol Scand. 2004;83:1124-1129.

5. Riede FT, Worner C, Dahnert I, et al. Effectiveness of neonatal pulse oximetry screening for detection of critical congenital heart disease in daily clinical routine—results from a prospective multicenter study. Eur J Pediatr. 2010;169:975-981.

6. Tautz J, Merkel C, Loersch F, et al. Implication of pulse oxymetry screening for detection of congenital heart defects. Klin Padiatr. 2010;222:291-295.

7. de-Wahl GA, Wennergren M, Sandberg K, et al. Impact of pulse oximetry screening on the detection of duct dependent congenital heart disease: a Swedish prospective screening study in 39,821 newborns. BMJ. 2009;338:a3037.-

8. Mahle WT, Martin GR, Beekman RH III, et al. Endorsement of Health and Human Services recommendation for pulse oximetry screening for critical congenital heart disease. Pediatrics. 2012;129:190-192.

Article PDF
Author and Disclosure Information

 

Amanda K. Swenson, MD, MSPH
Department of Family and Community Medicine, University of Missouri-Columbia

Dionna Brown, MD
Department of Family Medicine, University of Chicago

James J. Stevermer, MD, MSPH
Department of Family and Community Medicine, University of Missouri-Columbia

PURLs EDITOR
John Hickner, MD, MSc
Cleveland Clinic

Issue
The Journal of Family Practice - 61(05)
Publications
Topics
Page Number
283-286
Legacy Keywords
Amanda K. Swenson;MD;MSPH; Dionna Brown;MD; James J. Stevermer;MD;MSPH; pulse oximetry screening; congenital heart defects; CHD; detection rate; early discharge
Sections
Author and Disclosure Information

 

Amanda K. Swenson, MD, MSPH
Department of Family and Community Medicine, University of Missouri-Columbia

Dionna Brown, MD
Department of Family Medicine, University of Chicago

James J. Stevermer, MD, MSPH
Department of Family and Community Medicine, University of Missouri-Columbia

PURLs EDITOR
John Hickner, MD, MSc
Cleveland Clinic

Author and Disclosure Information

 

Amanda K. Swenson, MD, MSPH
Department of Family and Community Medicine, University of Missouri-Columbia

Dionna Brown, MD
Department of Family Medicine, University of Chicago

James J. Stevermer, MD, MSPH
Department of Family and Community Medicine, University of Missouri-Columbia

PURLs EDITOR
John Hickner, MD, MSc
Cleveland Clinic

Article PDF
Article PDF

 

PRACTICE CHANGER

Ensure that all newborns undergo pulse oximetry screening before discharge—and that abnormal results are immediately followed up with echocardiography.1

STRENGTH OF RECOMMENDATION

B: Based on a single cohort study consistent with multiple studies in other populations.

Ewer AK, Middleton LJ, Furmston AT, et al. Pulse oximetry screening for congenital heart defects in newborn infants (PulseOx): a test accuracy study. Lancet. 2011;378:785-794.

ILLUSTRATIVE CASE

A healthy, full-term baby is admitted to the newborn nursery. Antenatal surveillance, including routine ultrasound, was normal, as are physical examinations, both on admittance to the nursery and on the following day. Should the infant undergo pulse oximetry screening prior to discharge?

Congenital heart defects (CHD) are a leading cause of infant deaths in the developed world, occurring in approximately 9 of every 1000 live births.2 Roughly a quarter of those affected will have CHD serious enough to require either surgery or catheterization within the first year of life. These newborns are susceptible to sudden cardiovascular collapse due to changes in pulmonary vascular resistance and closure of the ductus arteriosus2—changes that often occur after the babies have gone home.

Delayed diagnosis is linked to worsening disease
A study evaluating 286 neonates admitted for cardiac surgery found that delayed diagnosis of CHD was associated with a worse preoperative condition. Cardiovascular compromise and end-organ dysfunction were most common in infants who presented with symptoms after they had gone home, the researchers found.3

In the past, screening for CHD hinged on mid-trimester ultrasound and postnatal physical examination. However, these methods do not reliably detect the condition in a timely fashion.4,5 More recently, pulse oximetry has been used for screening.

International studies prompt US recommendation
In a prospective multicenter trial in Saxony, Germany, more than 41,000 infants born between 2006 and 2008 underwent pulse oximetry screening at 24 to 72 hours of life.5 If the oxygen saturation was ≤95% and confirmed an hour later, echocardiography was performed. Pulse oximetry screening yielded true-positive results in 14 cases, false-positive results in 40, and false-negative results in 4. Sensitivity and specificity were 77.8% and 99.9%, respectively.

In another German study, 3364 term neonates underwent pulse oximetry screening between 6 and 36 hours of life.6 Eighteen neonates (0.5%) had abnormal results, 9 (50%) of whom were found to have heart defects. In this study, pulse oximetry had a sensitivity of 82% and a specificity of 99.9%.

A cohort study of 39,821 newborns in a single region of Sweden found that combining physical examination with pulse oximetry screening had a sensitivity of 82.8% and a specificity of 98%.7 No infants who underwent screening died from undiagnosed ductus arteriosus-dependent lung circulation, compared with 5 such deaths in regions where pulse oximetry screening was not done.

 

Encouraged by these findings, in late 2011 the US Secretary of Health and Human Services, with strong backing from the American Academy of Pediatrics, recommended universal pulse oximetry screening to detect critical CHD.8 The study detailed below took another look at its efficacy.

STUDY SUMMARY: Detection rate is higher for critical heart defects

The cohort study by Ewer et al enrolled 20,055 neonates born at >34 weeks’ gestation.1 All were screened with pulse oximetry on the right hand and on either foot and had a physical exam within their first 24 hours (The TABLE describes the screening protocol). Infants with a normal pulse oximetry and normal clinical exam were followed for a year to identify late-presenting heart defects.

One hundred ninety-five of the neonates who were screened had abnormal pulse oximetry test results; of these, 26 (13%) were found to have either critical (requiring intervention <28 days) or major (requiring intervention <12 months of age) CHD. Of the 169 infants who had positive pulse oximetry results but did not have critical or major heart defects, 6 were found to have less serious heart defects and 40 had infective or respiratory disorders that also required medical intervention.

Among the 19,860 infants with normal pulse oximetry, 27 (0.1%) were found to have either critical or major heart defects.

Pulse oximetry had a sensitivity of 75% (95% confidence interval [CI], 53.3-90.2) and a specificity of 99.1% (95% CI, 98.98-99.24) for detecting critical CHD. Sensitivity of pulse oximetry for all major CHD was 49% (95% CI, 35.0-63.2) and the specificity was 99.2% (95% CI, 99.02-99.28). The specificity may have been better if screening had been done after 24 hours of life; as seen in other studies,5,6 screening within the first 24 hours leads to more false-positive results.

 

 

The detection rate for critical CHD was higher than for major defects. However, most of the defects missed by screening were noncritical lesions, eg, ventricular septal defects.

The authors estimated that in a population of 100,000 newborns, about 120 would have critical CHD, and 90 of those 120 cases would be detected by pulse oximetry. There would be 843 false positives (although 229 of the infants with false-positive results would have other noncardiac conditions). It would be necessary to perform 10.4 echocardiograms to detect one patient with critical CHD.

WHAT’S NEW: A stronger case for newborn pulse oximetry screening

Pulse oximetry prior to discharge from the newborn nursery is not performed routinely in all institutions. And even when screening is done, there may not be a protocol addressing abnormal results. Pulse oximetry is a safe, noninvasive, inexpensive, and reasonably sensitive test that will detect many cases of critical CHD, some of which will not be diagnosed antenatally. Earlier diagnosis of CHD may lead to earlier interventions and improved patient outcomes.

CAVEATS: Timing of screening may alter results

This trial was a cohort study, not a randomized controlled trial (RCT)—the gold standard method of validating a screening test. It is unlikely, however, that an RCT will ever be done.

Screening occurred within the first 24 hours; other investigators have screened >24 hours (up to 38 hours), which may have better results. The critical lesions most likely to be missed by pulse oximetry screening were those causing obstruction to the aortic arch,1 a finding that was also seen in other studies.5,7

TABLE
Screening newborns for congenital heart defects: The protocol
1

 

Pulse oximetry outcome*Physical exam outcomeNext step
NormalNormalNo further action
AbnormalNormalRepeat pulse oximetry in 2 h:
  • normal=no further action
  • abnormal=echocardiogram
AbnormalAbnormalechocardiogram
*Test is normal if pulse oximetry >95% on right hand and difference between the right hand and either foot is <2%.

CHALLENGES TO IMPLEMENTATION: Early discharge, lack of equipment may interfere

Determining the timing of pulse oximetry screening is important. That’s particularly true because early discharge is a common practice, and early screening may increase the number of false-positive results. As a screening tool, pulse oximetry is inexpensive, and follow-up echocardiography—which is needed to exclude serious cases of CHD in patients with positive pulse oximetry—is noninvasive and relatively inexpensive. Echocardiography is not readily available in all communities, however, and transportation to a facility that offers this test would likely increase the cost of screening.

Acknowledgement

The PURLs Surveillance System is supported in part by Grant Number UL1RR024999 from the National Center for Research Resources, a Clinical Translational Science Award to the University of Chicago. The content is solely the responsibility of the authors and does not necessarily represent the official views of the National Center for Research Resources or the National Institutes of Health.

 

PRACTICE CHANGER

Ensure that all newborns undergo pulse oximetry screening before discharge—and that abnormal results are immediately followed up with echocardiography.1

STRENGTH OF RECOMMENDATION

B: Based on a single cohort study consistent with multiple studies in other populations.

Ewer AK, Middleton LJ, Furmston AT, et al. Pulse oximetry screening for congenital heart defects in newborn infants (PulseOx): a test accuracy study. Lancet. 2011;378:785-794.

ILLUSTRATIVE CASE

A healthy, full-term baby is admitted to the newborn nursery. Antenatal surveillance, including routine ultrasound, was normal, as are physical examinations, both on admittance to the nursery and on the following day. Should the infant undergo pulse oximetry screening prior to discharge?

Congenital heart defects (CHD) are a leading cause of infant deaths in the developed world, occurring in approximately 9 of every 1000 live births.2 Roughly a quarter of those affected will have CHD serious enough to require either surgery or catheterization within the first year of life. These newborns are susceptible to sudden cardiovascular collapse due to changes in pulmonary vascular resistance and closure of the ductus arteriosus2—changes that often occur after the babies have gone home.

Delayed diagnosis is linked to worsening disease
A study evaluating 286 neonates admitted for cardiac surgery found that delayed diagnosis of CHD was associated with a worse preoperative condition. Cardiovascular compromise and end-organ dysfunction were most common in infants who presented with symptoms after they had gone home, the researchers found.3

In the past, screening for CHD hinged on mid-trimester ultrasound and postnatal physical examination. However, these methods do not reliably detect the condition in a timely fashion.4,5 More recently, pulse oximetry has been used for screening.

International studies prompt US recommendation
In a prospective multicenter trial in Saxony, Germany, more than 41,000 infants born between 2006 and 2008 underwent pulse oximetry screening at 24 to 72 hours of life.5 If the oxygen saturation was ≤95% and confirmed an hour later, echocardiography was performed. Pulse oximetry screening yielded true-positive results in 14 cases, false-positive results in 40, and false-negative results in 4. Sensitivity and specificity were 77.8% and 99.9%, respectively.

In another German study, 3364 term neonates underwent pulse oximetry screening between 6 and 36 hours of life.6 Eighteen neonates (0.5%) had abnormal results, 9 (50%) of whom were found to have heart defects. In this study, pulse oximetry had a sensitivity of 82% and a specificity of 99.9%.

A cohort study of 39,821 newborns in a single region of Sweden found that combining physical examination with pulse oximetry screening had a sensitivity of 82.8% and a specificity of 98%.7 No infants who underwent screening died from undiagnosed ductus arteriosus-dependent lung circulation, compared with 5 such deaths in regions where pulse oximetry screening was not done.

 

Encouraged by these findings, in late 2011 the US Secretary of Health and Human Services, with strong backing from the American Academy of Pediatrics, recommended universal pulse oximetry screening to detect critical CHD.8 The study detailed below took another look at its efficacy.

STUDY SUMMARY: Detection rate is higher for critical heart defects

The cohort study by Ewer et al enrolled 20,055 neonates born at >34 weeks’ gestation.1 All were screened with pulse oximetry on the right hand and on either foot and had a physical exam within their first 24 hours (The TABLE describes the screening protocol). Infants with a normal pulse oximetry and normal clinical exam were followed for a year to identify late-presenting heart defects.

One hundred ninety-five of the neonates who were screened had abnormal pulse oximetry test results; of these, 26 (13%) were found to have either critical (requiring intervention <28 days) or major (requiring intervention <12 months of age) CHD. Of the 169 infants who had positive pulse oximetry results but did not have critical or major heart defects, 6 were found to have less serious heart defects and 40 had infective or respiratory disorders that also required medical intervention.

Among the 19,860 infants with normal pulse oximetry, 27 (0.1%) were found to have either critical or major heart defects.

Pulse oximetry had a sensitivity of 75% (95% confidence interval [CI], 53.3-90.2) and a specificity of 99.1% (95% CI, 98.98-99.24) for detecting critical CHD. Sensitivity of pulse oximetry for all major CHD was 49% (95% CI, 35.0-63.2) and the specificity was 99.2% (95% CI, 99.02-99.28). The specificity may have been better if screening had been done after 24 hours of life; as seen in other studies,5,6 screening within the first 24 hours leads to more false-positive results.

 

 

The detection rate for critical CHD was higher than for major defects. However, most of the defects missed by screening were noncritical lesions, eg, ventricular septal defects.

The authors estimated that in a population of 100,000 newborns, about 120 would have critical CHD, and 90 of those 120 cases would be detected by pulse oximetry. There would be 843 false positives (although 229 of the infants with false-positive results would have other noncardiac conditions). It would be necessary to perform 10.4 echocardiograms to detect one patient with critical CHD.

WHAT’S NEW: A stronger case for newborn pulse oximetry screening

Pulse oximetry prior to discharge from the newborn nursery is not performed routinely in all institutions. And even when screening is done, there may not be a protocol addressing abnormal results. Pulse oximetry is a safe, noninvasive, inexpensive, and reasonably sensitive test that will detect many cases of critical CHD, some of which will not be diagnosed antenatally. Earlier diagnosis of CHD may lead to earlier interventions and improved patient outcomes.

CAVEATS: Timing of screening may alter results

This trial was a cohort study, not a randomized controlled trial (RCT)—the gold standard method of validating a screening test. It is unlikely, however, that an RCT will ever be done.

Screening occurred within the first 24 hours; other investigators have screened >24 hours (up to 38 hours), which may have better results. The critical lesions most likely to be missed by pulse oximetry screening were those causing obstruction to the aortic arch,1 a finding that was also seen in other studies.5,7

TABLE
Screening newborns for congenital heart defects: The protocol
1

 

Pulse oximetry outcome*Physical exam outcomeNext step
NormalNormalNo further action
AbnormalNormalRepeat pulse oximetry in 2 h:
  • normal=no further action
  • abnormal=echocardiogram
AbnormalAbnormalechocardiogram
*Test is normal if pulse oximetry >95% on right hand and difference between the right hand and either foot is <2%.

CHALLENGES TO IMPLEMENTATION: Early discharge, lack of equipment may interfere

Determining the timing of pulse oximetry screening is important. That’s particularly true because early discharge is a common practice, and early screening may increase the number of false-positive results. As a screening tool, pulse oximetry is inexpensive, and follow-up echocardiography—which is needed to exclude serious cases of CHD in patients with positive pulse oximetry—is noninvasive and relatively inexpensive. Echocardiography is not readily available in all communities, however, and transportation to a facility that offers this test would likely increase the cost of screening.

Acknowledgement

The PURLs Surveillance System is supported in part by Grant Number UL1RR024999 from the National Center for Research Resources, a Clinical Translational Science Award to the University of Chicago. The content is solely the responsibility of the authors and does not necessarily represent the official views of the National Center for Research Resources or the National Institutes of Health.

References

 

1. Ewer AK, Middleton LJ, Furmston AT, et al. Pulse oximetry screening for congenital heart defects in newborn infants (PulseOx): a test accuracy study. Lancet. 201;378:785-794.

2. Mahle WT, Newburger JW, Matherne GP, et al. Role of pulse oximetry in examining newborns for congenital heart disease: a scientific statement from the AHA and AAP. Pediatrics. 2009;124:823-836.

3. Brown KL, Ridout DA, Hoskote A, et al. Delayed diagnosis of congenital heart disease worsens preoperative condition and outcome of surgery in neonates. Heart. 2006;92:1298-1302.

4. Acharya G, Sitras V, Maltau JM, et al. Major congenital heart disease in Northern Norway: shortcomings of pre- and postnatal diagnosis. Acta Obstet Gynecol Scand. 2004;83:1124-1129.

5. Riede FT, Worner C, Dahnert I, et al. Effectiveness of neonatal pulse oximetry screening for detection of critical congenital heart disease in daily clinical routine—results from a prospective multicenter study. Eur J Pediatr. 2010;169:975-981.

6. Tautz J, Merkel C, Loersch F, et al. Implication of pulse oxymetry screening for detection of congenital heart defects. Klin Padiatr. 2010;222:291-295.

7. de-Wahl GA, Wennergren M, Sandberg K, et al. Impact of pulse oximetry screening on the detection of duct dependent congenital heart disease: a Swedish prospective screening study in 39,821 newborns. BMJ. 2009;338:a3037.-

8. Mahle WT, Martin GR, Beekman RH III, et al. Endorsement of Health and Human Services recommendation for pulse oximetry screening for critical congenital heart disease. Pediatrics. 2012;129:190-192.

References

 

1. Ewer AK, Middleton LJ, Furmston AT, et al. Pulse oximetry screening for congenital heart defects in newborn infants (PulseOx): a test accuracy study. Lancet. 201;378:785-794.

2. Mahle WT, Newburger JW, Matherne GP, et al. Role of pulse oximetry in examining newborns for congenital heart disease: a scientific statement from the AHA and AAP. Pediatrics. 2009;124:823-836.

3. Brown KL, Ridout DA, Hoskote A, et al. Delayed diagnosis of congenital heart disease worsens preoperative condition and outcome of surgery in neonates. Heart. 2006;92:1298-1302.

4. Acharya G, Sitras V, Maltau JM, et al. Major congenital heart disease in Northern Norway: shortcomings of pre- and postnatal diagnosis. Acta Obstet Gynecol Scand. 2004;83:1124-1129.

5. Riede FT, Worner C, Dahnert I, et al. Effectiveness of neonatal pulse oximetry screening for detection of critical congenital heart disease in daily clinical routine—results from a prospective multicenter study. Eur J Pediatr. 2010;169:975-981.

6. Tautz J, Merkel C, Loersch F, et al. Implication of pulse oxymetry screening for detection of congenital heart defects. Klin Padiatr. 2010;222:291-295.

7. de-Wahl GA, Wennergren M, Sandberg K, et al. Impact of pulse oximetry screening on the detection of duct dependent congenital heart disease: a Swedish prospective screening study in 39,821 newborns. BMJ. 2009;338:a3037.-

8. Mahle WT, Martin GR, Beekman RH III, et al. Endorsement of Health and Human Services recommendation for pulse oximetry screening for critical congenital heart disease. Pediatrics. 2012;129:190-192.

Issue
The Journal of Family Practice - 61(05)
Issue
The Journal of Family Practice - 61(05)
Page Number
283-286
Page Number
283-286
Publications
Publications
Topics
Article Type
Display Headline
Pulse oximetry for newborns: Should it be routine?
Display Headline
Pulse oximetry for newborns: Should it be routine?
Legacy Keywords
Amanda K. Swenson;MD;MSPH; Dionna Brown;MD; James J. Stevermer;MD;MSPH; pulse oximetry screening; congenital heart defects; CHD; detection rate; early discharge
Legacy Keywords
Amanda K. Swenson;MD;MSPH; Dionna Brown;MD; James J. Stevermer;MD;MSPH; pulse oximetry screening; congenital heart defects; CHD; detection rate; early discharge
Sections
PURLs Copyright

Copyright © 2012 The Family Physicians Inquiries Network. All rights reserved.

Disallow All Ads
Alternative CME
Article PDF Media

What’s best for IBS?

Article Type
Changed
Mon, 01/14/2019 - 11:34
Display Headline
What’s best for IBS?

PRACTICE CHANGER

Recommend antispasmodics or antidepressants for patients with irritable bowel syndrome (IBS) and explain that, while fiber may have other benefits, it is unlikely to relieve IBS symptoms.1

STRENGTH OF RECOMMENDATION

A: Based on a meta-analysis.

Ruepert L, Quartero AO, deWit NJ, et al. Bulking agents, antispasmodics and antidepressants for the treatment of irritable bowel syndrome. Cochrane Database Syst Rev. 2011;(8):CD003460.

ILLUSTRATIVE CASE

A 25-year-old woman who has been your patient for several years has intermittent bouts of abdominal pain, constipation, gas, and bloating. You believe she can benefit from treatment for IBS. What should you recommend?

IBS is the most common functional disorder of the gastrointestinal (GI) tract, affecting approximately 15% of the US population2 and accounting for annual health care costs of roughly $30 billion.3 The primary symptoms are bloating, gas, and abdominal pain that often improves immediately after a bowel movement. Patients may have intermittent diarrhea and constipation, as well.

IBS may be related to “brain-gut dysfunction”
The etiology of IBS is unclear, but many agree that a combination of abnormal GI motility, visceral hypersensitivity, and “brain-gut dysfunction”—the inability of the brain to send signals that turn down pain produced in the GI tract—are contributing factors. Although IBS is not life threatening, it has a significant personal, social, and psychological impact. Despite its high prevalence and impact, only a limited number of large studies have assessed the effectiveness of various treatments.

STUDY SUMMARY: Antispasmodics, antidepressants offer relief—fiber does not

The Cochrane review included 56 randomized controlled trials (RCTs) comparing the efficacy of bulking agents (fiber supplements), anti-spasmodics, or antidepressants with placebo for the treatment of IBS. Twelve RCTs (n=621) focused on bulking agents, 29 (n=2333) on antispasmodics, and 15 (n=922) on antidepressants. Inclusion criteria included age (>12 years) and an IBS diagnosis. The outcomes analyzed were improvement in abdominal pain, global health assessments, and IBS symptom scores. Adverse effects were not evaluated.

Bulking agents. In studies ranging from 4 to 16 weeks, bulking agents were found to have no significant effect on abdominal pain (4 studies; standardized mean difference [SMD], 0.03; 95% confidence interval [CI], -0.34 to 0.40; P=.87) or global functioning (11 studies; risk ratio [RR]=1.11; 95% CI, 0.91-1.35; P=.32). Nor was there an improvement in IBS symptom score (3 studies; SMD=0.00; 95% CI, -0.43 to 0.43; P=1.00).

Antispasmodics. Assessed in RCTs ranging from one week to 6 months, antispasmodics significantly improved abdominal pain (RR=1.3; 95% CI, 1.1-1.55; P<.001; number needed to treat [NNT]=7); global functioning (RR=1.5; 95% CI, 1.2-1.8; P<.0001; NNT=5), and IBS symptom score (RR=1.9; 95% CI, 1.3-2.8; P<.01; NNT=3). Ten different antispasmodic agents were studied; in subgroup analyses, 5 of them— cimetropium/dicyclomine, peppermint oil, pinaverium, and trimebutine—were found to have statistically significant benefits.

Antidepressants. In studies of both tricyclics and selective serotonin reuptake inhibitors (SSRIs), antidepressants were found to have a significant effect on improving abdominal pain (RR=1.5; 95% CI, 1.0-2.1; P<.03; NNT=5), global functioning (RR=1.6; 95% CI, 1.2-2; P<.001; NNT=4), and IBS symptom score (RR=2.0; 95% CI, 1.3-3.0; P<.001; NNT=4). Subgroup analyses found statistically significant benefits in global functioning for SSRIs, and in abdominal pain and symptom scores for tricyclics.

WHAT’S NEW: More evidence against fiber for IBS symptoms

This Cochrane review confirms earlier findings—that both antispasmodics and antidepressants are effective treatments for IBS, but bulking agents are not. This is an important finding because dietary fiber adjustment is still among the first recommendations made by leading organizations like the American Gastroenterological Association and the World Gastroenterology Organisation.4,5

CAVEATS: Limitations of studies included in the meta-analysis

Adverse effects of antispasmodics and antidepressants, which may limit compliance and treatment efficacy, were not addressed by the Cochrane reviewers. The total number of participants in trials of bulking agents was much smaller than that of the other treatments, so it is possible that clinically meaningful improvements were missed due to inadequate statistical power. In addition, the duration of interventions was highly variable, ranging from one to 4 months for bulking agents and antidepressants and from one week to 6 months for antispasmodics.

It is also important to note that 8 of the 12 studies of bulking agents were conducted in GI clinics. (The settings in which the other 4 studies were conducted is unclear.) Given the possibility that patients referred to GI clinics have already tried and failed to respond to fiber (and thus, that those who do respond to fiber are not given referrals), it may be reasonable for family physicians to recommend a trial of bulking agents for patients with IBS and to monitor them for symptom improvement.

 

 

CHALLENGES TO IMPLEMENTATION: Patients may favor fiber

Patients with IBS may be reluctant to take antidepressants or antispasmodics, due to concern about adverse effects (eg, headache, insomnia, nervousness, dry mouth, and constipation) or because of a preference for what they see as a more “natural” remedy. It may be helpful to explain that while fiber may have some health benefits, such as lowering cholesterol,6 antispasmodics and antidepressants have been found to improve IBS symptoms but thus far, fiber has not.

Acknowledgement

The PURLs Surveillance System is supported in part by Grant Number UL1RR024999 from the National center for Research Resources, a clinical Translational Science Award to the University of chicago. The content is solely the responsibility of the authors and does not necessarily represent the official views of the National center for Research Resources or the National Institutes of Health.

References

1. Ruepert L, Quartero AO, deWit NJ, et al. Bulking agents, antispasmodics and antidepressants for the treatment of irritable bowel syndrome. Cochrane Database Syst Rev. 2011;(8):CD003460.-

2. Saito YA, Schoenfeld P, Locke GR 3rd. The epidemiology of irritable bowel syndrome in North America: a systematic review. Am J Gastroenterol. 2002;97:1910-1915.

3. Hulisz D. The burden of illness of irritable bowel syndrome: current challenges and hope for the future. J Manag Care Pharm. 2004;10:299-309.

4. American Gastroenterological Association. IBS: A patient’s guide to living with irritable bowel syndrome. Available at: http://www.gastro.org/patient-center/digestive-conditions/irritable-bowel-syndrome. Accessed March 21, 2012.

5. World Gastroenterology Organisation.WGO practice guideline— irritable bowel syndrome: a global perspective. 2009. Available at: http://www.worldgastroenterology.org/irritable-bowel-syndrome.html. Accessed March 16, 2012.

6. Gunness P, Gidley MJ. Mechanisms underlying the cholesterol-lowering properties of soluble dietary fibre polysaccharides. Food Funct. 2010;1:149-155.

Article PDF
Author and Disclosure Information

Sonia Oyola, MD
University of Chicago

Goutham Rao, MD
University of Chicago

PURLs EDITOR
Bernard Ewigman, MD, MSPH

University of Chicago

Issue
The Journal of Family Practice - 61(4)
Publications
Topics
Page Number
213-214
Legacy Keywords
Sonia Oyola;MD; Goutham Rao;MD; antispasmodics; antidepressants; irritable bowel syndrome; IBS; bulking agents; fiber supplements; IBS symptom score; dietary fiber;
Sections
Author and Disclosure Information

Sonia Oyola, MD
University of Chicago

Goutham Rao, MD
University of Chicago

PURLs EDITOR
Bernard Ewigman, MD, MSPH

University of Chicago

Author and Disclosure Information

Sonia Oyola, MD
University of Chicago

Goutham Rao, MD
University of Chicago

PURLs EDITOR
Bernard Ewigman, MD, MSPH

University of Chicago

Article PDF
Article PDF

PRACTICE CHANGER

Recommend antispasmodics or antidepressants for patients with irritable bowel syndrome (IBS) and explain that, while fiber may have other benefits, it is unlikely to relieve IBS symptoms.1

STRENGTH OF RECOMMENDATION

A: Based on a meta-analysis.

Ruepert L, Quartero AO, deWit NJ, et al. Bulking agents, antispasmodics and antidepressants for the treatment of irritable bowel syndrome. Cochrane Database Syst Rev. 2011;(8):CD003460.

ILLUSTRATIVE CASE

A 25-year-old woman who has been your patient for several years has intermittent bouts of abdominal pain, constipation, gas, and bloating. You believe she can benefit from treatment for IBS. What should you recommend?

IBS is the most common functional disorder of the gastrointestinal (GI) tract, affecting approximately 15% of the US population2 and accounting for annual health care costs of roughly $30 billion.3 The primary symptoms are bloating, gas, and abdominal pain that often improves immediately after a bowel movement. Patients may have intermittent diarrhea and constipation, as well.

IBS may be related to “brain-gut dysfunction”
The etiology of IBS is unclear, but many agree that a combination of abnormal GI motility, visceral hypersensitivity, and “brain-gut dysfunction”—the inability of the brain to send signals that turn down pain produced in the GI tract—are contributing factors. Although IBS is not life threatening, it has a significant personal, social, and psychological impact. Despite its high prevalence and impact, only a limited number of large studies have assessed the effectiveness of various treatments.

STUDY SUMMARY: Antispasmodics, antidepressants offer relief—fiber does not

The Cochrane review included 56 randomized controlled trials (RCTs) comparing the efficacy of bulking agents (fiber supplements), anti-spasmodics, or antidepressants with placebo for the treatment of IBS. Twelve RCTs (n=621) focused on bulking agents, 29 (n=2333) on antispasmodics, and 15 (n=922) on antidepressants. Inclusion criteria included age (>12 years) and an IBS diagnosis. The outcomes analyzed were improvement in abdominal pain, global health assessments, and IBS symptom scores. Adverse effects were not evaluated.

Bulking agents. In studies ranging from 4 to 16 weeks, bulking agents were found to have no significant effect on abdominal pain (4 studies; standardized mean difference [SMD], 0.03; 95% confidence interval [CI], -0.34 to 0.40; P=.87) or global functioning (11 studies; risk ratio [RR]=1.11; 95% CI, 0.91-1.35; P=.32). Nor was there an improvement in IBS symptom score (3 studies; SMD=0.00; 95% CI, -0.43 to 0.43; P=1.00).

Antispasmodics. Assessed in RCTs ranging from one week to 6 months, antispasmodics significantly improved abdominal pain (RR=1.3; 95% CI, 1.1-1.55; P<.001; number needed to treat [NNT]=7); global functioning (RR=1.5; 95% CI, 1.2-1.8; P<.0001; NNT=5), and IBS symptom score (RR=1.9; 95% CI, 1.3-2.8; P<.01; NNT=3). Ten different antispasmodic agents were studied; in subgroup analyses, 5 of them— cimetropium/dicyclomine, peppermint oil, pinaverium, and trimebutine—were found to have statistically significant benefits.

Antidepressants. In studies of both tricyclics and selective serotonin reuptake inhibitors (SSRIs), antidepressants were found to have a significant effect on improving abdominal pain (RR=1.5; 95% CI, 1.0-2.1; P<.03; NNT=5), global functioning (RR=1.6; 95% CI, 1.2-2; P<.001; NNT=4), and IBS symptom score (RR=2.0; 95% CI, 1.3-3.0; P<.001; NNT=4). Subgroup analyses found statistically significant benefits in global functioning for SSRIs, and in abdominal pain and symptom scores for tricyclics.

WHAT’S NEW: More evidence against fiber for IBS symptoms

This Cochrane review confirms earlier findings—that both antispasmodics and antidepressants are effective treatments for IBS, but bulking agents are not. This is an important finding because dietary fiber adjustment is still among the first recommendations made by leading organizations like the American Gastroenterological Association and the World Gastroenterology Organisation.4,5

CAVEATS: Limitations of studies included in the meta-analysis

Adverse effects of antispasmodics and antidepressants, which may limit compliance and treatment efficacy, were not addressed by the Cochrane reviewers. The total number of participants in trials of bulking agents was much smaller than that of the other treatments, so it is possible that clinically meaningful improvements were missed due to inadequate statistical power. In addition, the duration of interventions was highly variable, ranging from one to 4 months for bulking agents and antidepressants and from one week to 6 months for antispasmodics.

It is also important to note that 8 of the 12 studies of bulking agents were conducted in GI clinics. (The settings in which the other 4 studies were conducted is unclear.) Given the possibility that patients referred to GI clinics have already tried and failed to respond to fiber (and thus, that those who do respond to fiber are not given referrals), it may be reasonable for family physicians to recommend a trial of bulking agents for patients with IBS and to monitor them for symptom improvement.

 

 

CHALLENGES TO IMPLEMENTATION: Patients may favor fiber

Patients with IBS may be reluctant to take antidepressants or antispasmodics, due to concern about adverse effects (eg, headache, insomnia, nervousness, dry mouth, and constipation) or because of a preference for what they see as a more “natural” remedy. It may be helpful to explain that while fiber may have some health benefits, such as lowering cholesterol,6 antispasmodics and antidepressants have been found to improve IBS symptoms but thus far, fiber has not.

Acknowledgement

The PURLs Surveillance System is supported in part by Grant Number UL1RR024999 from the National center for Research Resources, a clinical Translational Science Award to the University of chicago. The content is solely the responsibility of the authors and does not necessarily represent the official views of the National center for Research Resources or the National Institutes of Health.

PRACTICE CHANGER

Recommend antispasmodics or antidepressants for patients with irritable bowel syndrome (IBS) and explain that, while fiber may have other benefits, it is unlikely to relieve IBS symptoms.1

STRENGTH OF RECOMMENDATION

A: Based on a meta-analysis.

Ruepert L, Quartero AO, deWit NJ, et al. Bulking agents, antispasmodics and antidepressants for the treatment of irritable bowel syndrome. Cochrane Database Syst Rev. 2011;(8):CD003460.

ILLUSTRATIVE CASE

A 25-year-old woman who has been your patient for several years has intermittent bouts of abdominal pain, constipation, gas, and bloating. You believe she can benefit from treatment for IBS. What should you recommend?

IBS is the most common functional disorder of the gastrointestinal (GI) tract, affecting approximately 15% of the US population2 and accounting for annual health care costs of roughly $30 billion.3 The primary symptoms are bloating, gas, and abdominal pain that often improves immediately after a bowel movement. Patients may have intermittent diarrhea and constipation, as well.

IBS may be related to “brain-gut dysfunction”
The etiology of IBS is unclear, but many agree that a combination of abnormal GI motility, visceral hypersensitivity, and “brain-gut dysfunction”—the inability of the brain to send signals that turn down pain produced in the GI tract—are contributing factors. Although IBS is not life threatening, it has a significant personal, social, and psychological impact. Despite its high prevalence and impact, only a limited number of large studies have assessed the effectiveness of various treatments.

STUDY SUMMARY: Antispasmodics, antidepressants offer relief—fiber does not

The Cochrane review included 56 randomized controlled trials (RCTs) comparing the efficacy of bulking agents (fiber supplements), anti-spasmodics, or antidepressants with placebo for the treatment of IBS. Twelve RCTs (n=621) focused on bulking agents, 29 (n=2333) on antispasmodics, and 15 (n=922) on antidepressants. Inclusion criteria included age (>12 years) and an IBS diagnosis. The outcomes analyzed were improvement in abdominal pain, global health assessments, and IBS symptom scores. Adverse effects were not evaluated.

Bulking agents. In studies ranging from 4 to 16 weeks, bulking agents were found to have no significant effect on abdominal pain (4 studies; standardized mean difference [SMD], 0.03; 95% confidence interval [CI], -0.34 to 0.40; P=.87) or global functioning (11 studies; risk ratio [RR]=1.11; 95% CI, 0.91-1.35; P=.32). Nor was there an improvement in IBS symptom score (3 studies; SMD=0.00; 95% CI, -0.43 to 0.43; P=1.00).

Antispasmodics. Assessed in RCTs ranging from one week to 6 months, antispasmodics significantly improved abdominal pain (RR=1.3; 95% CI, 1.1-1.55; P<.001; number needed to treat [NNT]=7); global functioning (RR=1.5; 95% CI, 1.2-1.8; P<.0001; NNT=5), and IBS symptom score (RR=1.9; 95% CI, 1.3-2.8; P<.01; NNT=3). Ten different antispasmodic agents were studied; in subgroup analyses, 5 of them— cimetropium/dicyclomine, peppermint oil, pinaverium, and trimebutine—were found to have statistically significant benefits.

Antidepressants. In studies of both tricyclics and selective serotonin reuptake inhibitors (SSRIs), antidepressants were found to have a significant effect on improving abdominal pain (RR=1.5; 95% CI, 1.0-2.1; P<.03; NNT=5), global functioning (RR=1.6; 95% CI, 1.2-2; P<.001; NNT=4), and IBS symptom score (RR=2.0; 95% CI, 1.3-3.0; P<.001; NNT=4). Subgroup analyses found statistically significant benefits in global functioning for SSRIs, and in abdominal pain and symptom scores for tricyclics.

WHAT’S NEW: More evidence against fiber for IBS symptoms

This Cochrane review confirms earlier findings—that both antispasmodics and antidepressants are effective treatments for IBS, but bulking agents are not. This is an important finding because dietary fiber adjustment is still among the first recommendations made by leading organizations like the American Gastroenterological Association and the World Gastroenterology Organisation.4,5

CAVEATS: Limitations of studies included in the meta-analysis

Adverse effects of antispasmodics and antidepressants, which may limit compliance and treatment efficacy, were not addressed by the Cochrane reviewers. The total number of participants in trials of bulking agents was much smaller than that of the other treatments, so it is possible that clinically meaningful improvements were missed due to inadequate statistical power. In addition, the duration of interventions was highly variable, ranging from one to 4 months for bulking agents and antidepressants and from one week to 6 months for antispasmodics.

It is also important to note that 8 of the 12 studies of bulking agents were conducted in GI clinics. (The settings in which the other 4 studies were conducted is unclear.) Given the possibility that patients referred to GI clinics have already tried and failed to respond to fiber (and thus, that those who do respond to fiber are not given referrals), it may be reasonable for family physicians to recommend a trial of bulking agents for patients with IBS and to monitor them for symptom improvement.

 

 

CHALLENGES TO IMPLEMENTATION: Patients may favor fiber

Patients with IBS may be reluctant to take antidepressants or antispasmodics, due to concern about adverse effects (eg, headache, insomnia, nervousness, dry mouth, and constipation) or because of a preference for what they see as a more “natural” remedy. It may be helpful to explain that while fiber may have some health benefits, such as lowering cholesterol,6 antispasmodics and antidepressants have been found to improve IBS symptoms but thus far, fiber has not.

Acknowledgement

The PURLs Surveillance System is supported in part by Grant Number UL1RR024999 from the National center for Research Resources, a clinical Translational Science Award to the University of chicago. The content is solely the responsibility of the authors and does not necessarily represent the official views of the National center for Research Resources or the National Institutes of Health.

References

1. Ruepert L, Quartero AO, deWit NJ, et al. Bulking agents, antispasmodics and antidepressants for the treatment of irritable bowel syndrome. Cochrane Database Syst Rev. 2011;(8):CD003460.-

2. Saito YA, Schoenfeld P, Locke GR 3rd. The epidemiology of irritable bowel syndrome in North America: a systematic review. Am J Gastroenterol. 2002;97:1910-1915.

3. Hulisz D. The burden of illness of irritable bowel syndrome: current challenges and hope for the future. J Manag Care Pharm. 2004;10:299-309.

4. American Gastroenterological Association. IBS: A patient’s guide to living with irritable bowel syndrome. Available at: http://www.gastro.org/patient-center/digestive-conditions/irritable-bowel-syndrome. Accessed March 21, 2012.

5. World Gastroenterology Organisation.WGO practice guideline— irritable bowel syndrome: a global perspective. 2009. Available at: http://www.worldgastroenterology.org/irritable-bowel-syndrome.html. Accessed March 16, 2012.

6. Gunness P, Gidley MJ. Mechanisms underlying the cholesterol-lowering properties of soluble dietary fibre polysaccharides. Food Funct. 2010;1:149-155.

References

1. Ruepert L, Quartero AO, deWit NJ, et al. Bulking agents, antispasmodics and antidepressants for the treatment of irritable bowel syndrome. Cochrane Database Syst Rev. 2011;(8):CD003460.-

2. Saito YA, Schoenfeld P, Locke GR 3rd. The epidemiology of irritable bowel syndrome in North America: a systematic review. Am J Gastroenterol. 2002;97:1910-1915.

3. Hulisz D. The burden of illness of irritable bowel syndrome: current challenges and hope for the future. J Manag Care Pharm. 2004;10:299-309.

4. American Gastroenterological Association. IBS: A patient’s guide to living with irritable bowel syndrome. Available at: http://www.gastro.org/patient-center/digestive-conditions/irritable-bowel-syndrome. Accessed March 21, 2012.

5. World Gastroenterology Organisation.WGO practice guideline— irritable bowel syndrome: a global perspective. 2009. Available at: http://www.worldgastroenterology.org/irritable-bowel-syndrome.html. Accessed March 16, 2012.

6. Gunness P, Gidley MJ. Mechanisms underlying the cholesterol-lowering properties of soluble dietary fibre polysaccharides. Food Funct. 2010;1:149-155.

Issue
The Journal of Family Practice - 61(4)
Issue
The Journal of Family Practice - 61(4)
Page Number
213-214
Page Number
213-214
Publications
Publications
Topics
Article Type
Display Headline
What’s best for IBS?
Display Headline
What’s best for IBS?
Legacy Keywords
Sonia Oyola;MD; Goutham Rao;MD; antispasmodics; antidepressants; irritable bowel syndrome; IBS; bulking agents; fiber supplements; IBS symptom score; dietary fiber;
Legacy Keywords
Sonia Oyola;MD; Goutham Rao;MD; antispasmodics; antidepressants; irritable bowel syndrome; IBS; bulking agents; fiber supplements; IBS symptom score; dietary fiber;
Sections
Article Source

PURLs Copyright

Copyright ©2012 The Family Physicians Inquiries Network. All rights reserved.

Inside the Article

Article PDF Media

BP meds: This simple change improves outcomes

Article Type
Changed
Mon, 01/14/2019 - 11:33
Display Headline
BP meds: This simple change improves outcomes

PRACTICE CHANGER

Advise patients with uncontrolled hypertension to take at least one of their blood pressure (BP) medications at bedtime instead of in the morning. Nighttime dosing leads to better control and lowers the risk of major cardiovascular events.1,2

STRENGTH OF RECOMMENDATION

B: Based on a well-done randomized clinical trial (RCT) and a subgroup analysis.

Hermida RC, Ayala DE, Mojón A, et al. Influence of circadian time of hypertension treatment on cardiovascular risk: results of the MAPEC study. Chronobiol Int. 2010;27:1629-1651.

Hermida RC, Ayala DE, Mojón A, et al. Influence of time of day of blood pressure-lowering treatment on cardiovascular risk in hypertensive patients with type 2 diabetes. Diabetes Care. 2011;34:1270-1276.

ILLUSTRATIVE CASES

  • A 60-year-old man has struggled to get his BP under control despite the use of 3 anti-hypertensives. Is there anything you can recommend to improve his BP control and lower his cardiovascular risk?
  • You prescribe hydrochlorothiazide for a 55-year-old woman with newly diagnosed hypertension. What can you tell her about how to take the medication to maximize its beneficial effects?

Management of hypertension often centers around BP measurements taken in a doctor’s office during the day, although both BP and metabolism fluctuate with circadian rhythms. Most people experience an increase in pressure during the day, with peaks in the morning and evening, followed by a decline in BP while they sleep at night.3

The focus belongs on nighttime BP
Sleeping BP is getting considerable attention, particularly the phenomenon of nondipping. Commonly defined as a <10% decline in systolic pressure during sleep, nondipping is associated with an increased risk of cardiovascular events, such as heart attack and stroke.4 What’s more, mean BP during the night is a better predictor of cardiovascular disease (CVD) risk than BP while the patient is awake.5,6

Evidence suggests that taking an anti-hypertensive medication at night increases its therapeutic effect,7 yet most patients take it in the morning.8 The study detailed in this PURL was designed to investigate whether bedtime dosing significantly affects BP control and CVD risk.

STUDY SUMMARY: Bedtime dosing benefits patients, and there’s no downside

The MAPEC study was an open-label RCT conducted at a single center in Spain.1 Patients were enrolled if they had a diagnosis of either untreated hypertension (based on ambulatory BP monitoring [ABPM] criteria) or resistant hypertension (uncontrolled on ≥3 optimally dosed antihypertensive medications). Exclusion criteria included pregnancy, a history of drug/alcohol abuse, night shift work, acquired immune deficiency syndrome, type 1 diabetes, secondary hypertension, and a previous CVD diagnosis.

Patients were randomly assigned to one of 2 time-of-day dosing groups: morning dosing of all their BP medications (n=1109) or dosing of ≥1 BP medications at bedtime (n=1092). ABPM—in which patients wore a monitor that recorded their BP every 20 minutes during the day and every 30 minutes at night for 48 hours—was conducted once a year, or more frequently when medication adjustments occurred. The use of a specific drug was not required, but physicians were instructed to adjust medications according to a study-specific ABPM protocol.

Patients were followed for a mean of 5.6 years for the endpoints of CVD events and mortality. These endpoints were assessed by researchers blinded to patients’ treatment assignment.

At baseline, the 2 groups were similar in age (mean of 55 years), percentage of men (48%), presence of comorbidities, and baseline clinic and ambulatory BP. Throughout the study, patients in the bedtime dosing group had lower mean asleep systolic and diastolic BP, a lower prevalence of a non-dipping pattern, and a higher prevalence of controlled ambulatory BP. The bedtime group also had a lower risk of total CVD events (relative risk [RR]=0.39; 95% confidence interval [CI], 0.29-0.51; P<.001) and major CVD events (RR=0.33; 95% CI, 0.19-0.55; P<.001), and fewer overall deaths (4.16/1000 vs 2.11/1000 patient-years; P=.008) (TABLE). To prevent one CVD event, 63 patients would need to take their BP medication at bedtime instead of in the morning for one year. To prevent one death, 488 patient would need to adhere to the nighttime schedule for one year.

A subgroup analysis of patients with type 2 diabetes (n=448)2 had similar results: For this population, too, bedtime dosing led to lower asleep BP, a lower prevalence of a non-dipping pattern, and a higher prevalence of controlled ambulatory BP, as well as a lower risk of total CVD events, major CVD events, and CVD-related death. The differences persisted after correction for the use of statins and aspirin. Among those in this subgroup analysis, 29 patients would need to take their BP medications at bedtime for one year to prevent one CVD event, and 263 patients would need to be treated for one year to prevent one death.

 

 

TABLE
Dosing of BP meds: A look at outcomes

Events/1000 patient-yearsMorning dosingBedtime dosingP between groups
overall (n=2201)1
Total events*27.8011.95<.001
CVd death2.080.53.006
Cardiovascular events11.005.27<.001
Cerebrovascular events3.571.23.001
Diabetes subgroup (n=448)2
Total events*54.2419.80<.001
CVd death4.790.86.038
Cardiovascular events15.956.89.008
Cerebrovascular events6.380.86.010
*Includes death from all causes and cardiovascular and cerebrovascular events.
BP, blood pressure; CVD, cardiovascular disease.

WHAT’S NEW: Advantages of preventing nondipping are clearly established

We’ve known that a nondipping pattern is associated with higher cardiovascular risks and that taking antihypertensives at bedtime decreases the prevalence of nondipping patterns. The MAPEC study, however, is the first prospective trial to show that bedtime dosing of BP medications lowers the risk of CVD events and death.

CAVEATS: Methodology, non-US guidelines raise questions about applicability here

MAPEC was an open-label study, meaning that the physicians adjusting BP medications were aware of the treatment groups to which their patients were allocated. Physicians were given guidelines for the titration of medications, but it is unclear whether they treated patients in both treatment groups identically. Patients were also aware of their treatment group, which creates the potential for bias if one group adhered to their medications more closely than the other.

The study was a single-center trial conducted in Spain, which may limit its generalizability to the United States. Notably, Spain’s medication guidelines differ from ours, with angiotensin-converting enzyme inhibitors, angiotensin receptor blockers, beta-blockers, and calcium channel blockers as first-line medications and hydrochlorothiazide as a second-line option.

While ABPM appears to be a better indicator of CVD risk compared with clinic BP monitoring, most US physicians still rely on readings taken in their office for diagnosing and managing hypertension. How ambulatory BP translates to clinic BP is somewhat unclear.

CHALLENGES TO IMPLEMENTATION: Some patients and providers may resist the switch

We see few challenges to implementing bedtime dosing of BP medications for patients with uncontrolled hypertension. It is possible, however, that patients who have a long-standing routine of taking their medications in the morning may be resistant to change. Also, pharmacists and nurses, as well as some physicians, may continue recommending morning dosing, which could be confusing for patients.

Acknowledgement

The PURLs Surveillance System is supported in part by Grant Number UL1RR024999 from the National Center for Research Resources, a Clinical Translational Science Award to the University of Chicago. The content is solely the responsibility of the authors and does not necessarily represent the official views of the National Center for Research Resources or the National Institutes of Health.

References

1. Hermida RC, Ayala DE, Mojón A, et al. Influence of circadian time of hypertension treatment on cardiovascular risk: results of the MAPEC study. Chronobiol Int. 2010;27:1629-1651.

2. Hermida RC, Ayala DE, Mojón A, et al. Influence of time of day of blood pressure-lowering treatment on cardiovascular risk in hypertensive patients with type 2 diabetes. Diabetes Care. 2011;34:1270-1276.

3. Hermida RC, Ayala DE, Portaluppi F. Circadian variation of blood pressure: the basis for the chronotherapy of hypertension. Adv Drug Deliv Rev. 2007;59:904-922.

4. Verdecchia P, Porcellati C, Schillaci G, et al. Ambulatory blood pressure: an independent predictor of prognosis in essential hypertension. Hypertension. 1994;24:793-801.

5. Dolan E, Stanton A, Thijs L, et al. Superiority of ambulatory over clinic blood pressure measurement in predicting mortality: the Dublin study outcome. Hypertension. 2005;46:156-161.

6. Hermida RC, Ayala DE, Mojón A, et al. Decreasing sleep-time blood pressure determined by ambulatory monitoring reduces cardiovascular risk. J Am Coll Cardiol. 2011;58:1165-1173.

7. De la Sierra A, Redon J, Banegas JR, et al. Prevalence and factors associated with circadian blood pressure patterns in hypertensive patients. Hypertension. 2009;53:466-472.

8. Hermida RC, Ayala DE, Calvo C, et al. Chronotherapy of hypertension: administration-time-dependent effects of treatment on the circadian pattern of blood pressure. Adv Drug Deliv Rev. 2007;59:923-939.

Article PDF
Author and Disclosure Information

Kate Kirley, MD
University of Chicago

Umang Sharma, MD
University of Chicago

Kate Rowland, MD
University of Chicago

PURLs EDITOR
John Hickner, MD, MSc

Cleveland Clinic

Issue
The Journal of Family Practice - 61(3)
Publications
Topics
Page Number
153-155
Legacy Keywords
Kate Kirley;MD; Umang Sharma;MD; Kate Rowland;MD; uncontrolled hypertension; bedtime dosing; blood pressure medications; BP meds; cardiovascular events; open-label RCT; CVD risk; morning dosing
Sections
Author and Disclosure Information

Kate Kirley, MD
University of Chicago

Umang Sharma, MD
University of Chicago

Kate Rowland, MD
University of Chicago

PURLs EDITOR
John Hickner, MD, MSc

Cleveland Clinic

Author and Disclosure Information

Kate Kirley, MD
University of Chicago

Umang Sharma, MD
University of Chicago

Kate Rowland, MD
University of Chicago

PURLs EDITOR
John Hickner, MD, MSc

Cleveland Clinic

Article PDF
Article PDF

PRACTICE CHANGER

Advise patients with uncontrolled hypertension to take at least one of their blood pressure (BP) medications at bedtime instead of in the morning. Nighttime dosing leads to better control and lowers the risk of major cardiovascular events.1,2

STRENGTH OF RECOMMENDATION

B: Based on a well-done randomized clinical trial (RCT) and a subgroup analysis.

Hermida RC, Ayala DE, Mojón A, et al. Influence of circadian time of hypertension treatment on cardiovascular risk: results of the MAPEC study. Chronobiol Int. 2010;27:1629-1651.

Hermida RC, Ayala DE, Mojón A, et al. Influence of time of day of blood pressure-lowering treatment on cardiovascular risk in hypertensive patients with type 2 diabetes. Diabetes Care. 2011;34:1270-1276.

ILLUSTRATIVE CASES

  • A 60-year-old man has struggled to get his BP under control despite the use of 3 anti-hypertensives. Is there anything you can recommend to improve his BP control and lower his cardiovascular risk?
  • You prescribe hydrochlorothiazide for a 55-year-old woman with newly diagnosed hypertension. What can you tell her about how to take the medication to maximize its beneficial effects?

Management of hypertension often centers around BP measurements taken in a doctor’s office during the day, although both BP and metabolism fluctuate with circadian rhythms. Most people experience an increase in pressure during the day, with peaks in the morning and evening, followed by a decline in BP while they sleep at night.3

The focus belongs on nighttime BP
Sleeping BP is getting considerable attention, particularly the phenomenon of nondipping. Commonly defined as a <10% decline in systolic pressure during sleep, nondipping is associated with an increased risk of cardiovascular events, such as heart attack and stroke.4 What’s more, mean BP during the night is a better predictor of cardiovascular disease (CVD) risk than BP while the patient is awake.5,6

Evidence suggests that taking an anti-hypertensive medication at night increases its therapeutic effect,7 yet most patients take it in the morning.8 The study detailed in this PURL was designed to investigate whether bedtime dosing significantly affects BP control and CVD risk.

STUDY SUMMARY: Bedtime dosing benefits patients, and there’s no downside

The MAPEC study was an open-label RCT conducted at a single center in Spain.1 Patients were enrolled if they had a diagnosis of either untreated hypertension (based on ambulatory BP monitoring [ABPM] criteria) or resistant hypertension (uncontrolled on ≥3 optimally dosed antihypertensive medications). Exclusion criteria included pregnancy, a history of drug/alcohol abuse, night shift work, acquired immune deficiency syndrome, type 1 diabetes, secondary hypertension, and a previous CVD diagnosis.

Patients were randomly assigned to one of 2 time-of-day dosing groups: morning dosing of all their BP medications (n=1109) or dosing of ≥1 BP medications at bedtime (n=1092). ABPM—in which patients wore a monitor that recorded their BP every 20 minutes during the day and every 30 minutes at night for 48 hours—was conducted once a year, or more frequently when medication adjustments occurred. The use of a specific drug was not required, but physicians were instructed to adjust medications according to a study-specific ABPM protocol.

Patients were followed for a mean of 5.6 years for the endpoints of CVD events and mortality. These endpoints were assessed by researchers blinded to patients’ treatment assignment.

At baseline, the 2 groups were similar in age (mean of 55 years), percentage of men (48%), presence of comorbidities, and baseline clinic and ambulatory BP. Throughout the study, patients in the bedtime dosing group had lower mean asleep systolic and diastolic BP, a lower prevalence of a non-dipping pattern, and a higher prevalence of controlled ambulatory BP. The bedtime group also had a lower risk of total CVD events (relative risk [RR]=0.39; 95% confidence interval [CI], 0.29-0.51; P<.001) and major CVD events (RR=0.33; 95% CI, 0.19-0.55; P<.001), and fewer overall deaths (4.16/1000 vs 2.11/1000 patient-years; P=.008) (TABLE). To prevent one CVD event, 63 patients would need to take their BP medication at bedtime instead of in the morning for one year. To prevent one death, 488 patient would need to adhere to the nighttime schedule for one year.

A subgroup analysis of patients with type 2 diabetes (n=448)2 had similar results: For this population, too, bedtime dosing led to lower asleep BP, a lower prevalence of a non-dipping pattern, and a higher prevalence of controlled ambulatory BP, as well as a lower risk of total CVD events, major CVD events, and CVD-related death. The differences persisted after correction for the use of statins and aspirin. Among those in this subgroup analysis, 29 patients would need to take their BP medications at bedtime for one year to prevent one CVD event, and 263 patients would need to be treated for one year to prevent one death.

 

 

TABLE
Dosing of BP meds: A look at outcomes

Events/1000 patient-yearsMorning dosingBedtime dosingP between groups
overall (n=2201)1
Total events*27.8011.95<.001
CVd death2.080.53.006
Cardiovascular events11.005.27<.001
Cerebrovascular events3.571.23.001
Diabetes subgroup (n=448)2
Total events*54.2419.80<.001
CVd death4.790.86.038
Cardiovascular events15.956.89.008
Cerebrovascular events6.380.86.010
*Includes death from all causes and cardiovascular and cerebrovascular events.
BP, blood pressure; CVD, cardiovascular disease.

WHAT’S NEW: Advantages of preventing nondipping are clearly established

We’ve known that a nondipping pattern is associated with higher cardiovascular risks and that taking antihypertensives at bedtime decreases the prevalence of nondipping patterns. The MAPEC study, however, is the first prospective trial to show that bedtime dosing of BP medications lowers the risk of CVD events and death.

CAVEATS: Methodology, non-US guidelines raise questions about applicability here

MAPEC was an open-label study, meaning that the physicians adjusting BP medications were aware of the treatment groups to which their patients were allocated. Physicians were given guidelines for the titration of medications, but it is unclear whether they treated patients in both treatment groups identically. Patients were also aware of their treatment group, which creates the potential for bias if one group adhered to their medications more closely than the other.

The study was a single-center trial conducted in Spain, which may limit its generalizability to the United States. Notably, Spain’s medication guidelines differ from ours, with angiotensin-converting enzyme inhibitors, angiotensin receptor blockers, beta-blockers, and calcium channel blockers as first-line medications and hydrochlorothiazide as a second-line option.

While ABPM appears to be a better indicator of CVD risk compared with clinic BP monitoring, most US physicians still rely on readings taken in their office for diagnosing and managing hypertension. How ambulatory BP translates to clinic BP is somewhat unclear.

CHALLENGES TO IMPLEMENTATION: Some patients and providers may resist the switch

We see few challenges to implementing bedtime dosing of BP medications for patients with uncontrolled hypertension. It is possible, however, that patients who have a long-standing routine of taking their medications in the morning may be resistant to change. Also, pharmacists and nurses, as well as some physicians, may continue recommending morning dosing, which could be confusing for patients.

Acknowledgement

The PURLs Surveillance System is supported in part by Grant Number UL1RR024999 from the National Center for Research Resources, a Clinical Translational Science Award to the University of Chicago. The content is solely the responsibility of the authors and does not necessarily represent the official views of the National Center for Research Resources or the National Institutes of Health.

PRACTICE CHANGER

Advise patients with uncontrolled hypertension to take at least one of their blood pressure (BP) medications at bedtime instead of in the morning. Nighttime dosing leads to better control and lowers the risk of major cardiovascular events.1,2

STRENGTH OF RECOMMENDATION

B: Based on a well-done randomized clinical trial (RCT) and a subgroup analysis.

Hermida RC, Ayala DE, Mojón A, et al. Influence of circadian time of hypertension treatment on cardiovascular risk: results of the MAPEC study. Chronobiol Int. 2010;27:1629-1651.

Hermida RC, Ayala DE, Mojón A, et al. Influence of time of day of blood pressure-lowering treatment on cardiovascular risk in hypertensive patients with type 2 diabetes. Diabetes Care. 2011;34:1270-1276.

ILLUSTRATIVE CASES

  • A 60-year-old man has struggled to get his BP under control despite the use of 3 anti-hypertensives. Is there anything you can recommend to improve his BP control and lower his cardiovascular risk?
  • You prescribe hydrochlorothiazide for a 55-year-old woman with newly diagnosed hypertension. What can you tell her about how to take the medication to maximize its beneficial effects?

Management of hypertension often centers around BP measurements taken in a doctor’s office during the day, although both BP and metabolism fluctuate with circadian rhythms. Most people experience an increase in pressure during the day, with peaks in the morning and evening, followed by a decline in BP while they sleep at night.3

The focus belongs on nighttime BP
Sleeping BP is getting considerable attention, particularly the phenomenon of nondipping. Commonly defined as a <10% decline in systolic pressure during sleep, nondipping is associated with an increased risk of cardiovascular events, such as heart attack and stroke.4 What’s more, mean BP during the night is a better predictor of cardiovascular disease (CVD) risk than BP while the patient is awake.5,6

Evidence suggests that taking an anti-hypertensive medication at night increases its therapeutic effect,7 yet most patients take it in the morning.8 The study detailed in this PURL was designed to investigate whether bedtime dosing significantly affects BP control and CVD risk.

STUDY SUMMARY: Bedtime dosing benefits patients, and there’s no downside

The MAPEC study was an open-label RCT conducted at a single center in Spain.1 Patients were enrolled if they had a diagnosis of either untreated hypertension (based on ambulatory BP monitoring [ABPM] criteria) or resistant hypertension (uncontrolled on ≥3 optimally dosed antihypertensive medications). Exclusion criteria included pregnancy, a history of drug/alcohol abuse, night shift work, acquired immune deficiency syndrome, type 1 diabetes, secondary hypertension, and a previous CVD diagnosis.

Patients were randomly assigned to one of 2 time-of-day dosing groups: morning dosing of all their BP medications (n=1109) or dosing of ≥1 BP medications at bedtime (n=1092). ABPM—in which patients wore a monitor that recorded their BP every 20 minutes during the day and every 30 minutes at night for 48 hours—was conducted once a year, or more frequently when medication adjustments occurred. The use of a specific drug was not required, but physicians were instructed to adjust medications according to a study-specific ABPM protocol.

Patients were followed for a mean of 5.6 years for the endpoints of CVD events and mortality. These endpoints were assessed by researchers blinded to patients’ treatment assignment.

At baseline, the 2 groups were similar in age (mean of 55 years), percentage of men (48%), presence of comorbidities, and baseline clinic and ambulatory BP. Throughout the study, patients in the bedtime dosing group had lower mean asleep systolic and diastolic BP, a lower prevalence of a non-dipping pattern, and a higher prevalence of controlled ambulatory BP. The bedtime group also had a lower risk of total CVD events (relative risk [RR]=0.39; 95% confidence interval [CI], 0.29-0.51; P<.001) and major CVD events (RR=0.33; 95% CI, 0.19-0.55; P<.001), and fewer overall deaths (4.16/1000 vs 2.11/1000 patient-years; P=.008) (TABLE). To prevent one CVD event, 63 patients would need to take their BP medication at bedtime instead of in the morning for one year. To prevent one death, 488 patient would need to adhere to the nighttime schedule for one year.

A subgroup analysis of patients with type 2 diabetes (n=448)2 had similar results: For this population, too, bedtime dosing led to lower asleep BP, a lower prevalence of a non-dipping pattern, and a higher prevalence of controlled ambulatory BP, as well as a lower risk of total CVD events, major CVD events, and CVD-related death. The differences persisted after correction for the use of statins and aspirin. Among those in this subgroup analysis, 29 patients would need to take their BP medications at bedtime for one year to prevent one CVD event, and 263 patients would need to be treated for one year to prevent one death.

 

 

TABLE
Dosing of BP meds: A look at outcomes

Events/1000 patient-yearsMorning dosingBedtime dosingP between groups
overall (n=2201)1
Total events*27.8011.95<.001
CVd death2.080.53.006
Cardiovascular events11.005.27<.001
Cerebrovascular events3.571.23.001
Diabetes subgroup (n=448)2
Total events*54.2419.80<.001
CVd death4.790.86.038
Cardiovascular events15.956.89.008
Cerebrovascular events6.380.86.010
*Includes death from all causes and cardiovascular and cerebrovascular events.
BP, blood pressure; CVD, cardiovascular disease.

WHAT’S NEW: Advantages of preventing nondipping are clearly established

We’ve known that a nondipping pattern is associated with higher cardiovascular risks and that taking antihypertensives at bedtime decreases the prevalence of nondipping patterns. The MAPEC study, however, is the first prospective trial to show that bedtime dosing of BP medications lowers the risk of CVD events and death.

CAVEATS: Methodology, non-US guidelines raise questions about applicability here

MAPEC was an open-label study, meaning that the physicians adjusting BP medications were aware of the treatment groups to which their patients were allocated. Physicians were given guidelines for the titration of medications, but it is unclear whether they treated patients in both treatment groups identically. Patients were also aware of their treatment group, which creates the potential for bias if one group adhered to their medications more closely than the other.

The study was a single-center trial conducted in Spain, which may limit its generalizability to the United States. Notably, Spain’s medication guidelines differ from ours, with angiotensin-converting enzyme inhibitors, angiotensin receptor blockers, beta-blockers, and calcium channel blockers as first-line medications and hydrochlorothiazide as a second-line option.

While ABPM appears to be a better indicator of CVD risk compared with clinic BP monitoring, most US physicians still rely on readings taken in their office for diagnosing and managing hypertension. How ambulatory BP translates to clinic BP is somewhat unclear.

CHALLENGES TO IMPLEMENTATION: Some patients and providers may resist the switch

We see few challenges to implementing bedtime dosing of BP medications for patients with uncontrolled hypertension. It is possible, however, that patients who have a long-standing routine of taking their medications in the morning may be resistant to change. Also, pharmacists and nurses, as well as some physicians, may continue recommending morning dosing, which could be confusing for patients.

Acknowledgement

The PURLs Surveillance System is supported in part by Grant Number UL1RR024999 from the National Center for Research Resources, a Clinical Translational Science Award to the University of Chicago. The content is solely the responsibility of the authors and does not necessarily represent the official views of the National Center for Research Resources or the National Institutes of Health.

References

1. Hermida RC, Ayala DE, Mojón A, et al. Influence of circadian time of hypertension treatment on cardiovascular risk: results of the MAPEC study. Chronobiol Int. 2010;27:1629-1651.

2. Hermida RC, Ayala DE, Mojón A, et al. Influence of time of day of blood pressure-lowering treatment on cardiovascular risk in hypertensive patients with type 2 diabetes. Diabetes Care. 2011;34:1270-1276.

3. Hermida RC, Ayala DE, Portaluppi F. Circadian variation of blood pressure: the basis for the chronotherapy of hypertension. Adv Drug Deliv Rev. 2007;59:904-922.

4. Verdecchia P, Porcellati C, Schillaci G, et al. Ambulatory blood pressure: an independent predictor of prognosis in essential hypertension. Hypertension. 1994;24:793-801.

5. Dolan E, Stanton A, Thijs L, et al. Superiority of ambulatory over clinic blood pressure measurement in predicting mortality: the Dublin study outcome. Hypertension. 2005;46:156-161.

6. Hermida RC, Ayala DE, Mojón A, et al. Decreasing sleep-time blood pressure determined by ambulatory monitoring reduces cardiovascular risk. J Am Coll Cardiol. 2011;58:1165-1173.

7. De la Sierra A, Redon J, Banegas JR, et al. Prevalence and factors associated with circadian blood pressure patterns in hypertensive patients. Hypertension. 2009;53:466-472.

8. Hermida RC, Ayala DE, Calvo C, et al. Chronotherapy of hypertension: administration-time-dependent effects of treatment on the circadian pattern of blood pressure. Adv Drug Deliv Rev. 2007;59:923-939.

References

1. Hermida RC, Ayala DE, Mojón A, et al. Influence of circadian time of hypertension treatment on cardiovascular risk: results of the MAPEC study. Chronobiol Int. 2010;27:1629-1651.

2. Hermida RC, Ayala DE, Mojón A, et al. Influence of time of day of blood pressure-lowering treatment on cardiovascular risk in hypertensive patients with type 2 diabetes. Diabetes Care. 2011;34:1270-1276.

3. Hermida RC, Ayala DE, Portaluppi F. Circadian variation of blood pressure: the basis for the chronotherapy of hypertension. Adv Drug Deliv Rev. 2007;59:904-922.

4. Verdecchia P, Porcellati C, Schillaci G, et al. Ambulatory blood pressure: an independent predictor of prognosis in essential hypertension. Hypertension. 1994;24:793-801.

5. Dolan E, Stanton A, Thijs L, et al. Superiority of ambulatory over clinic blood pressure measurement in predicting mortality: the Dublin study outcome. Hypertension. 2005;46:156-161.

6. Hermida RC, Ayala DE, Mojón A, et al. Decreasing sleep-time blood pressure determined by ambulatory monitoring reduces cardiovascular risk. J Am Coll Cardiol. 2011;58:1165-1173.

7. De la Sierra A, Redon J, Banegas JR, et al. Prevalence and factors associated with circadian blood pressure patterns in hypertensive patients. Hypertension. 2009;53:466-472.

8. Hermida RC, Ayala DE, Calvo C, et al. Chronotherapy of hypertension: administration-time-dependent effects of treatment on the circadian pattern of blood pressure. Adv Drug Deliv Rev. 2007;59:923-939.

Issue
The Journal of Family Practice - 61(3)
Issue
The Journal of Family Practice - 61(3)
Page Number
153-155
Page Number
153-155
Publications
Publications
Topics
Article Type
Display Headline
BP meds: This simple change improves outcomes
Display Headline
BP meds: This simple change improves outcomes
Legacy Keywords
Kate Kirley;MD; Umang Sharma;MD; Kate Rowland;MD; uncontrolled hypertension; bedtime dosing; blood pressure medications; BP meds; cardiovascular events; open-label RCT; CVD risk; morning dosing
Legacy Keywords
Kate Kirley;MD; Umang Sharma;MD; Kate Rowland;MD; uncontrolled hypertension; bedtime dosing; blood pressure medications; BP meds; cardiovascular events; open-label RCT; CVD risk; morning dosing
Sections
Article Source

PURLs Copyright

Copyright © 2012 The Family Physicians Inquiries Network.
All rights reserved.

Inside the Article

Article PDF Media

Counseling is a must with this smoking cessation aid

Article Type
Changed
Mon, 01/14/2019 - 11:33
Display Headline
Counseling is a must with this smoking cessation aid
PRACTICE CHANGER

Inform patients who are interested in taking varenicline (Chantix) that there is a small cardiovascular (CV) risk associated with it, as well as neuropsychiatric risks—and consider recommending that smokers with a history of cardiovascular disease (CVD) use nicotine replacement therapy (NRT) or bupropion instead.1

STRENGTH OF RECOMMENDATION

A: Based on a meta-analysis.

Singh S, Loke YK, Spangler JG, et al. Risk of serious adverse cardiovascular events associated with varenicline: a systematic review and meta-analysis. CMAJ. 2011;183:1359-1366.

ILLUSTRATIVE CASE

A 53-year-old man asks you to prescribe Chantix to help him stop smoking. He has made several attempts to quit in the past, but never managed to stop for more than 6 months— and has smoked a pack a day for 30 years. The patient does not have a history of heart disease, but he is on statin therapy for hyperlipidemia. What should you tell him about varenicline’s potential benefits and risks?

Tobacco use remains the largest preventable contributor to death and disease in the United States.2 In smokers with coronary heart disease, smoking cessation is associated with a 36% reduction in all-cause mortality (relative risk [RR], 0.64; 95% confidence interval [CI], 0.58-0.71)—a risk reduction greater than that of statins (29%), aspirin (15%), beta-blockers (23%), or ACE inhibitors (23%).3

Varenicline now has 2 black box warnings
In its 2009 update on recommendations for smoking cessation, the United States Preventive Services Task Force cited NRT and controlled-release bupropion, as well as varenicline, as effective smoking cessation aids.4 Varenicline received US Food and Drug Administration (FDA) approval in 2006. In 2009, the FDA added a black box warning based on evidence of its adverse neuropsychiatric effects, including suicidality.5

In July 2011, the FDA required another label change,6 based on a double-blind RCT published in 2010 showing that for patients with CVD, varenicline is associated with an increased risk.7 As a partial nicotine agonist, varenicline could confer some of the CV risk associated with nicotine abuse.8 The FDA has asked its manufacturer, Pfizer Inc, to conduct further studies.6 The meta-analysis reviewed below—which was not associated with Pfizer or the FDA—was published in September 2011, just a couple of months after the label change.1

 

STUDY SUMMARY: Risk of ischemic or arrhythmic event is small but significant

Singh et al searched for double-blind RCTs that tested varenicline against a control in tobacco users.1 All included studies had to have reported adverse CV events. The primary outcome was any ischemic or arrhythmic CV event.

The researchers found 15 such studies (n=8216), which ranged in duration from 7 to 52 weeks. Most used a placebo control, but some included bupropion or NRT. The researchers used a Peto odds ratio (OR) for the meta-analysis, useful when combining uncommon events and including studies with no events.9

Compared with placebo, varenicline significantly increased the risk of CV events (odds ratio [OR], 1.72; 95% CI, 1.09-2.71). The incidence of CV events was 1.06% (52 of 4908) among varenicline users vs 0.82% (27 of 3308) in the controls (number needed to harm [NNH]=417).

The limited number of deaths (1.4% among patients taking varenicline vs 2.1% in the placebo groups) prevented analysis of mortality risk. The study with the most statistical power, which accounted for 57% of the overall effect, was the only one that included patients with known stable CV disease. (None included patients with unstable CV disease, whose risk may be greater.) Even when this study was removed, however, the outcome (OR, 2.54; 95% CI, 1.26-5.12) was consistent with the primary result for CV events. A sensitivity analysis comparing the risk associated with varenicline with that of either NRT or bupropion yielded similar results (OR, 1.67; 95% CI, 1.07-26.2). For a higher risk population with stable CVD (5.6% annual risk at baseline), the authors estimated an overall NNH of 28 per year (95% CI, 13-213).

 

 

 

WHAT’S NEW: Evidence of CV risk is cause for concern

This meta-analysis provides evidence that varenicline is associated with a small but significant harmful effect on CV outcomes. The methods Singh et al used for review and article selection appear to be sound, and analysis of the included studies reveals little likelihood of publication bias.

CAVEATS: For many, benefits of quitting outweigh the risks

The absolute risk of a CV event found in this meta-analysis was small—just 0.24%. What’s more, the primary outcome was a composite of a diverse group of outcomes, some more serious than others. And, when compared with the highly positive effects of smoking cessation, the benefit-harm analysis still appears to favor varenicline for most patients. The estimated number needed to treat to get one person to stop smoking for ≥24 weeks is about 10 (95% CI, 8-13).8

CHALLENGES TO IMPLEMENTATION: Finding time to educate patients

The additional time needed to discuss the CV and neuropsychiatric risks of varenicline will be a challenge to physicians working in busy outpatient settings. Proper documentation of this discussion is prudent, however, given the increase in risk with this medication.

Acknowledgement

The Purls Surveillance System is supported in part by Grant Number UL1RR024999 from the National Center for Research Resources, a Clinical Translational Science Award to the University of Chicago. The content is solely the responsibility of the authors and does not necessarily represent the official views of the National Center for Research Resources or the National Institutes of Health.

Click here to view PURL METHODOLOGY

References

1. Singh S, Loke YK, Spangler JG, et al. Risk of serious adverse cardiovascular events associated with varenicline: a systematic review and meta-analysis. CMAJ. 2011;183:1359-1366.

2. Centers for Disease Control and Prevention. Smoking-attributable mortality, years of potential life lost, and productivity Losses-United States, 2000-2004. MMWR Morbidity and Mortality Weekly Report. 2008;57:1226-1228.

3. Critchley JA, Capewell S. Mortality risk reduction associated with smoking cessation in patients with coronary heart disease. JAMA. 2003;290:86-97.

4. US Preventive Services Task Force. Counseling and interventions to prevent tobacco use and tobacco-caused disease in adults and pregnant women. Ann Intern Med. 2009;150:551-555.

5. US Food and Drug Administration. Boxed warning on serious mental health events to be required for Chantix and Zyban [press release]. July 1, 2009. Available at: http://www.fda.gov/NewsEvents/Newsroom/PressAnnouncements/ucm170100.htm#.Ttab-ZCbYtE. Accessed January 21, 2012.

6. US Food and Drug Administration. Chantix (varenicline): label change - risk of certain cardiovascular adverse events. 2011. Available at: http://www.fda.gov/Safety/MedWatch/SafetyInformation/SafetyAlertsforHumanMedicalProducts/ucm259469.htm. Accessed January 21, 2012.

7. Rigotti NA, Pipe AL, Benowitz NL, et al. Efficacy and safety of varenicline for smoking cessation in patients with cardiovascular disease: a randomized trial. Circulation. 2010;121:221-229.

8. Cahill K, Stead LF, Lancaster T. Nicotine receptor partial agonists for smoking cessation. Cochrane Database Syst Rev. 2011;(2):CD006103.

9. Singh S, Loke YK, Spangler JG, et al. Authors’ response. CMAJ. 2011;183:1405, 1407.

Author and Disclosure Information

Luke A. Stephens, MD
Department of Family and Community Medicine, University of Missouri– Columbia

James J. Stevermer, MD, MSPH
Department of Family and Community Medicine, University of Missouri– Columbia

PURLs EDITOR
John Hickner, MD, MSc
Department of Family Medicine, Cleveland Clinic

Issue
The Journal of Family Practice - 61(3)
Publications
Topics
Page Number
156-176
Sections
Author and Disclosure Information

Luke A. Stephens, MD
Department of Family and Community Medicine, University of Missouri– Columbia

James J. Stevermer, MD, MSPH
Department of Family and Community Medicine, University of Missouri– Columbia

PURLs EDITOR
John Hickner, MD, MSc
Department of Family Medicine, Cleveland Clinic

Author and Disclosure Information

Luke A. Stephens, MD
Department of Family and Community Medicine, University of Missouri– Columbia

James J. Stevermer, MD, MSPH
Department of Family and Community Medicine, University of Missouri– Columbia

PURLs EDITOR
John Hickner, MD, MSc
Department of Family Medicine, Cleveland Clinic

PRACTICE CHANGER

Inform patients who are interested in taking varenicline (Chantix) that there is a small cardiovascular (CV) risk associated with it, as well as neuropsychiatric risks—and consider recommending that smokers with a history of cardiovascular disease (CVD) use nicotine replacement therapy (NRT) or bupropion instead.1

STRENGTH OF RECOMMENDATION

A: Based on a meta-analysis.

Singh S, Loke YK, Spangler JG, et al. Risk of serious adverse cardiovascular events associated with varenicline: a systematic review and meta-analysis. CMAJ. 2011;183:1359-1366.

ILLUSTRATIVE CASE

A 53-year-old man asks you to prescribe Chantix to help him stop smoking. He has made several attempts to quit in the past, but never managed to stop for more than 6 months— and has smoked a pack a day for 30 years. The patient does not have a history of heart disease, but he is on statin therapy for hyperlipidemia. What should you tell him about varenicline’s potential benefits and risks?

Tobacco use remains the largest preventable contributor to death and disease in the United States.2 In smokers with coronary heart disease, smoking cessation is associated with a 36% reduction in all-cause mortality (relative risk [RR], 0.64; 95% confidence interval [CI], 0.58-0.71)—a risk reduction greater than that of statins (29%), aspirin (15%), beta-blockers (23%), or ACE inhibitors (23%).3

Varenicline now has 2 black box warnings
In its 2009 update on recommendations for smoking cessation, the United States Preventive Services Task Force cited NRT and controlled-release bupropion, as well as varenicline, as effective smoking cessation aids.4 Varenicline received US Food and Drug Administration (FDA) approval in 2006. In 2009, the FDA added a black box warning based on evidence of its adverse neuropsychiatric effects, including suicidality.5

In July 2011, the FDA required another label change,6 based on a double-blind RCT published in 2010 showing that for patients with CVD, varenicline is associated with an increased risk.7 As a partial nicotine agonist, varenicline could confer some of the CV risk associated with nicotine abuse.8 The FDA has asked its manufacturer, Pfizer Inc, to conduct further studies.6 The meta-analysis reviewed below—which was not associated with Pfizer or the FDA—was published in September 2011, just a couple of months after the label change.1

 

STUDY SUMMARY: Risk of ischemic or arrhythmic event is small but significant

Singh et al searched for double-blind RCTs that tested varenicline against a control in tobacco users.1 All included studies had to have reported adverse CV events. The primary outcome was any ischemic or arrhythmic CV event.

The researchers found 15 such studies (n=8216), which ranged in duration from 7 to 52 weeks. Most used a placebo control, but some included bupropion or NRT. The researchers used a Peto odds ratio (OR) for the meta-analysis, useful when combining uncommon events and including studies with no events.9

Compared with placebo, varenicline significantly increased the risk of CV events (odds ratio [OR], 1.72; 95% CI, 1.09-2.71). The incidence of CV events was 1.06% (52 of 4908) among varenicline users vs 0.82% (27 of 3308) in the controls (number needed to harm [NNH]=417).

The limited number of deaths (1.4% among patients taking varenicline vs 2.1% in the placebo groups) prevented analysis of mortality risk. The study with the most statistical power, which accounted for 57% of the overall effect, was the only one that included patients with known stable CV disease. (None included patients with unstable CV disease, whose risk may be greater.) Even when this study was removed, however, the outcome (OR, 2.54; 95% CI, 1.26-5.12) was consistent with the primary result for CV events. A sensitivity analysis comparing the risk associated with varenicline with that of either NRT or bupropion yielded similar results (OR, 1.67; 95% CI, 1.07-26.2). For a higher risk population with stable CVD (5.6% annual risk at baseline), the authors estimated an overall NNH of 28 per year (95% CI, 13-213).

 

 

 

WHAT’S NEW: Evidence of CV risk is cause for concern

This meta-analysis provides evidence that varenicline is associated with a small but significant harmful effect on CV outcomes. The methods Singh et al used for review and article selection appear to be sound, and analysis of the included studies reveals little likelihood of publication bias.

CAVEATS: For many, benefits of quitting outweigh the risks

The absolute risk of a CV event found in this meta-analysis was small—just 0.24%. What’s more, the primary outcome was a composite of a diverse group of outcomes, some more serious than others. And, when compared with the highly positive effects of smoking cessation, the benefit-harm analysis still appears to favor varenicline for most patients. The estimated number needed to treat to get one person to stop smoking for ≥24 weeks is about 10 (95% CI, 8-13).8

CHALLENGES TO IMPLEMENTATION: Finding time to educate patients

The additional time needed to discuss the CV and neuropsychiatric risks of varenicline will be a challenge to physicians working in busy outpatient settings. Proper documentation of this discussion is prudent, however, given the increase in risk with this medication.

Acknowledgement

The Purls Surveillance System is supported in part by Grant Number UL1RR024999 from the National Center for Research Resources, a Clinical Translational Science Award to the University of Chicago. The content is solely the responsibility of the authors and does not necessarily represent the official views of the National Center for Research Resources or the National Institutes of Health.

Click here to view PURL METHODOLOGY

PRACTICE CHANGER

Inform patients who are interested in taking varenicline (Chantix) that there is a small cardiovascular (CV) risk associated with it, as well as neuropsychiatric risks—and consider recommending that smokers with a history of cardiovascular disease (CVD) use nicotine replacement therapy (NRT) or bupropion instead.1

STRENGTH OF RECOMMENDATION

A: Based on a meta-analysis.

Singh S, Loke YK, Spangler JG, et al. Risk of serious adverse cardiovascular events associated with varenicline: a systematic review and meta-analysis. CMAJ. 2011;183:1359-1366.

ILLUSTRATIVE CASE

A 53-year-old man asks you to prescribe Chantix to help him stop smoking. He has made several attempts to quit in the past, but never managed to stop for more than 6 months— and has smoked a pack a day for 30 years. The patient does not have a history of heart disease, but he is on statin therapy for hyperlipidemia. What should you tell him about varenicline’s potential benefits and risks?

Tobacco use remains the largest preventable contributor to death and disease in the United States.2 In smokers with coronary heart disease, smoking cessation is associated with a 36% reduction in all-cause mortality (relative risk [RR], 0.64; 95% confidence interval [CI], 0.58-0.71)—a risk reduction greater than that of statins (29%), aspirin (15%), beta-blockers (23%), or ACE inhibitors (23%).3

Varenicline now has 2 black box warnings
In its 2009 update on recommendations for smoking cessation, the United States Preventive Services Task Force cited NRT and controlled-release bupropion, as well as varenicline, as effective smoking cessation aids.4 Varenicline received US Food and Drug Administration (FDA) approval in 2006. In 2009, the FDA added a black box warning based on evidence of its adverse neuropsychiatric effects, including suicidality.5

In July 2011, the FDA required another label change,6 based on a double-blind RCT published in 2010 showing that for patients with CVD, varenicline is associated with an increased risk.7 As a partial nicotine agonist, varenicline could confer some of the CV risk associated with nicotine abuse.8 The FDA has asked its manufacturer, Pfizer Inc, to conduct further studies.6 The meta-analysis reviewed below—which was not associated with Pfizer or the FDA—was published in September 2011, just a couple of months after the label change.1

 

STUDY SUMMARY: Risk of ischemic or arrhythmic event is small but significant

Singh et al searched for double-blind RCTs that tested varenicline against a control in tobacco users.1 All included studies had to have reported adverse CV events. The primary outcome was any ischemic or arrhythmic CV event.

The researchers found 15 such studies (n=8216), which ranged in duration from 7 to 52 weeks. Most used a placebo control, but some included bupropion or NRT. The researchers used a Peto odds ratio (OR) for the meta-analysis, useful when combining uncommon events and including studies with no events.9

Compared with placebo, varenicline significantly increased the risk of CV events (odds ratio [OR], 1.72; 95% CI, 1.09-2.71). The incidence of CV events was 1.06% (52 of 4908) among varenicline users vs 0.82% (27 of 3308) in the controls (number needed to harm [NNH]=417).

The limited number of deaths (1.4% among patients taking varenicline vs 2.1% in the placebo groups) prevented analysis of mortality risk. The study with the most statistical power, which accounted for 57% of the overall effect, was the only one that included patients with known stable CV disease. (None included patients with unstable CV disease, whose risk may be greater.) Even when this study was removed, however, the outcome (OR, 2.54; 95% CI, 1.26-5.12) was consistent with the primary result for CV events. A sensitivity analysis comparing the risk associated with varenicline with that of either NRT or bupropion yielded similar results (OR, 1.67; 95% CI, 1.07-26.2). For a higher risk population with stable CVD (5.6% annual risk at baseline), the authors estimated an overall NNH of 28 per year (95% CI, 13-213).

 

 

 

WHAT’S NEW: Evidence of CV risk is cause for concern

This meta-analysis provides evidence that varenicline is associated with a small but significant harmful effect on CV outcomes. The methods Singh et al used for review and article selection appear to be sound, and analysis of the included studies reveals little likelihood of publication bias.

CAVEATS: For many, benefits of quitting outweigh the risks

The absolute risk of a CV event found in this meta-analysis was small—just 0.24%. What’s more, the primary outcome was a composite of a diverse group of outcomes, some more serious than others. And, when compared with the highly positive effects of smoking cessation, the benefit-harm analysis still appears to favor varenicline for most patients. The estimated number needed to treat to get one person to stop smoking for ≥24 weeks is about 10 (95% CI, 8-13).8

CHALLENGES TO IMPLEMENTATION: Finding time to educate patients

The additional time needed to discuss the CV and neuropsychiatric risks of varenicline will be a challenge to physicians working in busy outpatient settings. Proper documentation of this discussion is prudent, however, given the increase in risk with this medication.

Acknowledgement

The Purls Surveillance System is supported in part by Grant Number UL1RR024999 from the National Center for Research Resources, a Clinical Translational Science Award to the University of Chicago. The content is solely the responsibility of the authors and does not necessarily represent the official views of the National Center for Research Resources or the National Institutes of Health.

Click here to view PURL METHODOLOGY

References

1. Singh S, Loke YK, Spangler JG, et al. Risk of serious adverse cardiovascular events associated with varenicline: a systematic review and meta-analysis. CMAJ. 2011;183:1359-1366.

2. Centers for Disease Control and Prevention. Smoking-attributable mortality, years of potential life lost, and productivity Losses-United States, 2000-2004. MMWR Morbidity and Mortality Weekly Report. 2008;57:1226-1228.

3. Critchley JA, Capewell S. Mortality risk reduction associated with smoking cessation in patients with coronary heart disease. JAMA. 2003;290:86-97.

4. US Preventive Services Task Force. Counseling and interventions to prevent tobacco use and tobacco-caused disease in adults and pregnant women. Ann Intern Med. 2009;150:551-555.

5. US Food and Drug Administration. Boxed warning on serious mental health events to be required for Chantix and Zyban [press release]. July 1, 2009. Available at: http://www.fda.gov/NewsEvents/Newsroom/PressAnnouncements/ucm170100.htm#.Ttab-ZCbYtE. Accessed January 21, 2012.

6. US Food and Drug Administration. Chantix (varenicline): label change - risk of certain cardiovascular adverse events. 2011. Available at: http://www.fda.gov/Safety/MedWatch/SafetyInformation/SafetyAlertsforHumanMedicalProducts/ucm259469.htm. Accessed January 21, 2012.

7. Rigotti NA, Pipe AL, Benowitz NL, et al. Efficacy and safety of varenicline for smoking cessation in patients with cardiovascular disease: a randomized trial. Circulation. 2010;121:221-229.

8. Cahill K, Stead LF, Lancaster T. Nicotine receptor partial agonists for smoking cessation. Cochrane Database Syst Rev. 2011;(2):CD006103.

9. Singh S, Loke YK, Spangler JG, et al. Authors’ response. CMAJ. 2011;183:1405, 1407.

References

1. Singh S, Loke YK, Spangler JG, et al. Risk of serious adverse cardiovascular events associated with varenicline: a systematic review and meta-analysis. CMAJ. 2011;183:1359-1366.

2. Centers for Disease Control and Prevention. Smoking-attributable mortality, years of potential life lost, and productivity Losses-United States, 2000-2004. MMWR Morbidity and Mortality Weekly Report. 2008;57:1226-1228.

3. Critchley JA, Capewell S. Mortality risk reduction associated with smoking cessation in patients with coronary heart disease. JAMA. 2003;290:86-97.

4. US Preventive Services Task Force. Counseling and interventions to prevent tobacco use and tobacco-caused disease in adults and pregnant women. Ann Intern Med. 2009;150:551-555.

5. US Food and Drug Administration. Boxed warning on serious mental health events to be required for Chantix and Zyban [press release]. July 1, 2009. Available at: http://www.fda.gov/NewsEvents/Newsroom/PressAnnouncements/ucm170100.htm#.Ttab-ZCbYtE. Accessed January 21, 2012.

6. US Food and Drug Administration. Chantix (varenicline): label change - risk of certain cardiovascular adverse events. 2011. Available at: http://www.fda.gov/Safety/MedWatch/SafetyInformation/SafetyAlertsforHumanMedicalProducts/ucm259469.htm. Accessed January 21, 2012.

7. Rigotti NA, Pipe AL, Benowitz NL, et al. Efficacy and safety of varenicline for smoking cessation in patients with cardiovascular disease: a randomized trial. Circulation. 2010;121:221-229.

8. Cahill K, Stead LF, Lancaster T. Nicotine receptor partial agonists for smoking cessation. Cochrane Database Syst Rev. 2011;(2):CD006103.

9. Singh S, Loke YK, Spangler JG, et al. Authors’ response. CMAJ. 2011;183:1405, 1407.

Issue
The Journal of Family Practice - 61(3)
Issue
The Journal of Family Practice - 61(3)
Page Number
156-176
Page Number
156-176
Publications
Publications
Topics
Article Type
Display Headline
Counseling is a must with this smoking cessation aid
Display Headline
Counseling is a must with this smoking cessation aid
Sections
PURLs Copyright

Copyright © 2012 The Family Physicians Inquiries Network.
All rights reserved.

Disallow All Ads
Alternative CME

Would this long-acting bronchodilator be better for your patient?

Article Type
Changed
Mon, 01/14/2019 - 13:40
Display Headline
Would this long-acting bronchodilator be better for your patient?
PRACTICE CHANGER

Consider adding tiotropium to the medication regimen of patients with moderate to very severe chronic obstructive pulmonary disease (COPD), as a multinational study found it to be more effective than salmeterol in preventing exacerbations.1

STRENGTH OF RECOMMENDATION

A: Based on one well-designed randomized controlled trial.

Vogelmeier C, Hederer B, Glaab T, et al; POET-COPD investigators. Tiotropium versus salmeterol for the prevention of exacerbations of COPD. N Engl J Med. 2011;364:1093-1103.

 

ILLUSTRATIVE CASE

A 60-year-old patient with moderate COPD and a history of frequent exacerbations comes in for a follow-up visit. She has been using albuterol and ipratropium intermittently. you want to add a longer-acting bronchodilator and wonder if tiotropium or salmeterol is more effective for reducing exacerbations.

COPD is the fourth leading cause of death in the United States.2 More than 12 million Americans have been diagnosed with COPD, and it is estimated that another 12 million would have a COPD diagnosis if all smokers older than 45 years underwent spirometry.2 The disorder accounts for some 16 million physician visits each year and costs the US health care system approximately $19 billion annually, with acute exacerbations and hospitalizations representing 58% of the total.2,3

Despite guidelines, COPD is often undertreated
One of the main goals of COPD treatment is to reduce the frequency and intensity of acute exacerbations, both to improve patients’ quality of life and reduce health care costs. The Global Initiative for Chronic Obstructive Lung Disease (GOLD) has developed guidelines for effective management of COPD, which recommend long-acting bronchodilators as first-line maintenance therapy for patients whose disease is moderate to very severe.4

Evidence suggests that physicians frequently undertreat moderate to severe COPD, however, following national guidelines only about a quarter of the time.5 This is, in part, because many clinicians doubt the efficacy of COPD treatment for improving symptoms or decreasing exacerbations.5,6 Yet studies have shown that the long-acting broncho dilators tiotropium (an anticholinergic agent) and salmeterol (a beta2-adrenergic agonist), used with or without inhaled corticosteroids, are effective in reducing the frequency of COPD exacerbations, improving quality of life and lung function, and reducing the number of hospitalizations.7-10

Long-acting bronchodilators are therefore clearly indicated but, until recently, there was little evidence as to which one is better.

STUDY SUMMARY: Tiotropium group had fewer exacerbations…

The Prevention Of Exacerbations with Tiotropium in COPD (POET-COPD) trial compared tiotropium with salmeterol for their ability to prevent exacerbations.1 This was a randomized double-blind trial of 7376 patients with moderate to very severe COPD diagnosed by spirometry. Participants were recruited from 725 medical centers in 25 countries. To be eligible, they had to be ≥40 years, with at least a 10 pack-year history of smoking, a forced expiratory volume in 1 second (FEV1) <70% predicted, an FEV1/forced vital capacity (FVC ) <70%, and at least one exacerbation in the previous year.

Patients were randomly assigned to either the tiotropium or the salmeterol group. Those on tiotropium received a daily dose of 18 mcg through a HandiHaler device, plus a placebo with a metered-dose inhaler twice a day. Patients in the other group received 50 mcg salmeterol through a metered-dose inhaler twice daily, plus a placebo with a HandiHaler once a day. These medications were in addition to patients’ current medication regimens, including inhaled corticosteroids, with this exception: Use of anticholinergics and long-acting beta-agonists was discontinued for the course of the trial.

All participants were followed for one year, with clinic visits at 2, 4, 8, and 12 months to assess for medication adherence and symptoms of exacerbation. The primary endpoint was the time to first exacerbation. This was defined as an increase in, or a new onset of, more than one symptom of COPD (ie, cough, sputum production, wheezing, dyspnea, and chest tightness), with at least one symptom lasting ≥3 days and leading to treatment with glucocorticoids and/or antibiotics, or hospitalization. Secondary outcomes were times to first moderate and severe exacerbations and use of steroids and antibiotics.

There were significant differences in several outcomes. The time to first exacerbation was 187 days for tiotropium vs 145 days for salmeterol, a difference of 42 days (hazard ratio [HR]=0.83; 95% confidence interval [CI], 0.77-0.90; P<.001). In addition, tiotropium reduced the annual number of exacerbations compared with salmeterol (rate ratio=0.89; 95% CI, 0.83-0.96; P=.002), with a number needed to treat (NNT) of 24 patients to prevent one moderate to severe exacerbation per year.

 

 

 

…and used fewer drugs

Compared with salmeterol, there was a 14% reduction in risk of a moderate exacerbation associated with tiotropium (HR=0.86; 95% CI, 0.79-0.93; P<.001; NNT=32) and a 28% reduction in risk of a severe exacerbation (HR=0.72; 95% CI, 0.61-0.85; P<.001; NNT=48). In addition, the tiotropium group had a 23% risk reduction in the use of systemic glucocorticoids (HR=0.77; 95% CI, 0.69-0.85; P<.001; NNT=26) compared with the salmeterol group, and a 15% risk reduction in the use of antibiotics (HR=0.85; 95% CI, 0.78-0.92; P<0.001; NNT=31). The difference in reduction in death rates between the 2 groups was not statistically significant.

The observed differences were consistent across all major subgroups (age, sex, smoking status, and severity of COPD) of patients studied. Interestingly, patients with low BMI or very severe COPD appeared to benefit the most from tiotropium.

WHAT’S NEW: The difference between 2 agents is clear

Although national guidelines recommend long-acting bronchodilators for COPD that is moderate or worse, there have been few data to guide clinicians in determining which one to use. The findings of this study suggest that tiotropium should be our first choice. Tiotropium’s once-a-day dosing is an additional benefit, as patients using it will likely have better compliance than those using twice-daily salmeterol. The data may also prompt development of a once-daily inhaled corticosteroid/ long-acting anticholinergic combination.

CAVEATS: Cost, funding source

Cost may be an issue. Spiriva and Serevent, the brand names for tiotropium and salmeterol, respectively, are second-tier medications on several formularies, and tiotropium is about 45% more expensive (tiotropium=$262, salmeterol=$181 for one month’s supply; www.drugstore.com, accessed January 19, 2012). There are also several long-acting beta-agonists in development that will be dosed once daily; once they’re approved, tiotropium’s once-a-day dosing may no longer be seen as an advantage.

It is also worth noting that this trial was supported by Boehringer Ingelheim and Pfizer, which jointly market Spiriva.

Finally, smoking must be addressed. Strongly encouraging patients to kick the habit is still the most important intervention we can make in helping to improve the quality of life, and survival, of patients with COPD.

CHALLENGES TO IMPLEMENTATION: COPD guidelines need updating

There are no major challenges to incorporating this recommendation into clinical practice; the key challenge lies in diagnosing COPD and adequately monitoring and helping patients manage the disease.

Current guidelines do not distinguish between the efficacy of long-acting bronchodilators, but findings from this study are important enough to change future versions of national guidelines. The GOLD committee is due to release a new guideline report soon, and will likely update its recommendations at that time.

Acknowledgement

The PURLs Surveillance System is supported in part by Grant Number UL1RR024999 from the National Center for Research Resources, a Clinical Translational Science Award to the University of Chicago. The content is solely the responsibility of the authors and does not necessarily represent the official views of the National Center for Research Resources or the National Institutes of Health.

Click here to view PURL METHODOLOGY

References

1. Vogelmeier C, Hederer B, Glaab T, et al. POET-COPD investigators. Tiotropium versus salmeterol for the prevention of exacerbations of COPD. N Engl J Med. 2011;364:1093-1103.

2. National Heart, Lung, and Blood Institute. Morbidity and mortality: 2009 chart book on cardiovascular, lung, and blood diseases. Available at: http://www.nhlbi.nih.gov/resources/docs/04chtbk.pdf. Accessed October 1, 2011.

3. Miravitlles M, Murio C, Guerrero T, et al. DAFNE Study Group. Pharmacoeconomic evaluation of acute exacerbations of chronic bronchitis and COPD. Chest. 2002;121:1449-1455.

4. Global Initiative for Chronic Obstructive Lung Disease. Global strategy for the diagnosis, management, and prevention of chronic obstructive pulmonary disease. Updated 2010. Available at: http://www.goldcopd.org. Accessed October 1, 2011.

5. Salinas GD, Williamson JC, Kalhan R, et al. Barriers to adherence to chronic obstructive pulmonary disease guidelines by primary care physicians. Int J Chron Obstruct Pulmon Dis. 2011;6:171-179.

6. Yawn BP, Wollan PC. Knowledge and attitudes of family physicians coming to COPD continuing medical education. Int J Chron Obstruct Pulmon Dis. 2008;3:311-318.

7. Calverly PMA, Anderson JA, Celli B, et al. Salmeterol and fluticasone propionate and survival in chronic obstructive pulmonary disease. N Engl J Med. 2007;356:775-789.

8. Casaburi R, Mahler DA, Jones PW, et al. A long-term evaluation of once-daily inhaled tiotropium in chronic obstructive pulmonary disease. Eur Respir J. 2002;19:217-224.

9. Donahue JF, van Noord JA, Bateman ED, et al. A 6-month, placebo-controlled study comparing lung function and health status changes in COPD patients treated with tiotropium or salmeterol. Chest. 2002;122:47-55.

10. Tashkin DP, Celli B, Senn S, et al. A 4-year trial of tiotropium in chronic obstructive pulmonary disease. N Engl J Med. 2008;359:1543-1554.

Article PDF
Author and Disclosure Information

Mark Gwynne, DO
Department of Family Medicine, University of North Carolina at Chapel Hill

Anne Mounsey, MD
Department of Family Medicine, University of North Carolina at Chapel Hill

PURLs EDITOR
John Hickner, MD, MSc
Department of Family, Medicine, Cleveland Clinic

Issue
The Journal of Family Practice - 61(02)
Publications
Topics
Page Number
94-96
Sections
Author and Disclosure Information

Mark Gwynne, DO
Department of Family Medicine, University of North Carolina at Chapel Hill

Anne Mounsey, MD
Department of Family Medicine, University of North Carolina at Chapel Hill

PURLs EDITOR
John Hickner, MD, MSc
Department of Family, Medicine, Cleveland Clinic

Author and Disclosure Information

Mark Gwynne, DO
Department of Family Medicine, University of North Carolina at Chapel Hill

Anne Mounsey, MD
Department of Family Medicine, University of North Carolina at Chapel Hill

PURLs EDITOR
John Hickner, MD, MSc
Department of Family, Medicine, Cleveland Clinic

Article PDF
Article PDF
PRACTICE CHANGER

Consider adding tiotropium to the medication regimen of patients with moderate to very severe chronic obstructive pulmonary disease (COPD), as a multinational study found it to be more effective than salmeterol in preventing exacerbations.1

STRENGTH OF RECOMMENDATION

A: Based on one well-designed randomized controlled trial.

Vogelmeier C, Hederer B, Glaab T, et al; POET-COPD investigators. Tiotropium versus salmeterol for the prevention of exacerbations of COPD. N Engl J Med. 2011;364:1093-1103.

 

ILLUSTRATIVE CASE

A 60-year-old patient with moderate COPD and a history of frequent exacerbations comes in for a follow-up visit. She has been using albuterol and ipratropium intermittently. you want to add a longer-acting bronchodilator and wonder if tiotropium or salmeterol is more effective for reducing exacerbations.

COPD is the fourth leading cause of death in the United States.2 More than 12 million Americans have been diagnosed with COPD, and it is estimated that another 12 million would have a COPD diagnosis if all smokers older than 45 years underwent spirometry.2 The disorder accounts for some 16 million physician visits each year and costs the US health care system approximately $19 billion annually, with acute exacerbations and hospitalizations representing 58% of the total.2,3

Despite guidelines, COPD is often undertreated
One of the main goals of COPD treatment is to reduce the frequency and intensity of acute exacerbations, both to improve patients’ quality of life and reduce health care costs. The Global Initiative for Chronic Obstructive Lung Disease (GOLD) has developed guidelines for effective management of COPD, which recommend long-acting bronchodilators as first-line maintenance therapy for patients whose disease is moderate to very severe.4

Evidence suggests that physicians frequently undertreat moderate to severe COPD, however, following national guidelines only about a quarter of the time.5 This is, in part, because many clinicians doubt the efficacy of COPD treatment for improving symptoms or decreasing exacerbations.5,6 Yet studies have shown that the long-acting broncho dilators tiotropium (an anticholinergic agent) and salmeterol (a beta2-adrenergic agonist), used with or without inhaled corticosteroids, are effective in reducing the frequency of COPD exacerbations, improving quality of life and lung function, and reducing the number of hospitalizations.7-10

Long-acting bronchodilators are therefore clearly indicated but, until recently, there was little evidence as to which one is better.

STUDY SUMMARY: Tiotropium group had fewer exacerbations…

The Prevention Of Exacerbations with Tiotropium in COPD (POET-COPD) trial compared tiotropium with salmeterol for their ability to prevent exacerbations.1 This was a randomized double-blind trial of 7376 patients with moderate to very severe COPD diagnosed by spirometry. Participants were recruited from 725 medical centers in 25 countries. To be eligible, they had to be ≥40 years, with at least a 10 pack-year history of smoking, a forced expiratory volume in 1 second (FEV1) <70% predicted, an FEV1/forced vital capacity (FVC ) <70%, and at least one exacerbation in the previous year.

Patients were randomly assigned to either the tiotropium or the salmeterol group. Those on tiotropium received a daily dose of 18 mcg through a HandiHaler device, plus a placebo with a metered-dose inhaler twice a day. Patients in the other group received 50 mcg salmeterol through a metered-dose inhaler twice daily, plus a placebo with a HandiHaler once a day. These medications were in addition to patients’ current medication regimens, including inhaled corticosteroids, with this exception: Use of anticholinergics and long-acting beta-agonists was discontinued for the course of the trial.

All participants were followed for one year, with clinic visits at 2, 4, 8, and 12 months to assess for medication adherence and symptoms of exacerbation. The primary endpoint was the time to first exacerbation. This was defined as an increase in, or a new onset of, more than one symptom of COPD (ie, cough, sputum production, wheezing, dyspnea, and chest tightness), with at least one symptom lasting ≥3 days and leading to treatment with glucocorticoids and/or antibiotics, or hospitalization. Secondary outcomes were times to first moderate and severe exacerbations and use of steroids and antibiotics.

There were significant differences in several outcomes. The time to first exacerbation was 187 days for tiotropium vs 145 days for salmeterol, a difference of 42 days (hazard ratio [HR]=0.83; 95% confidence interval [CI], 0.77-0.90; P<.001). In addition, tiotropium reduced the annual number of exacerbations compared with salmeterol (rate ratio=0.89; 95% CI, 0.83-0.96; P=.002), with a number needed to treat (NNT) of 24 patients to prevent one moderate to severe exacerbation per year.

 

 

 

…and used fewer drugs

Compared with salmeterol, there was a 14% reduction in risk of a moderate exacerbation associated with tiotropium (HR=0.86; 95% CI, 0.79-0.93; P<.001; NNT=32) and a 28% reduction in risk of a severe exacerbation (HR=0.72; 95% CI, 0.61-0.85; P<.001; NNT=48). In addition, the tiotropium group had a 23% risk reduction in the use of systemic glucocorticoids (HR=0.77; 95% CI, 0.69-0.85; P<.001; NNT=26) compared with the salmeterol group, and a 15% risk reduction in the use of antibiotics (HR=0.85; 95% CI, 0.78-0.92; P<0.001; NNT=31). The difference in reduction in death rates between the 2 groups was not statistically significant.

The observed differences were consistent across all major subgroups (age, sex, smoking status, and severity of COPD) of patients studied. Interestingly, patients with low BMI or very severe COPD appeared to benefit the most from tiotropium.

WHAT’S NEW: The difference between 2 agents is clear

Although national guidelines recommend long-acting bronchodilators for COPD that is moderate or worse, there have been few data to guide clinicians in determining which one to use. The findings of this study suggest that tiotropium should be our first choice. Tiotropium’s once-a-day dosing is an additional benefit, as patients using it will likely have better compliance than those using twice-daily salmeterol. The data may also prompt development of a once-daily inhaled corticosteroid/ long-acting anticholinergic combination.

CAVEATS: Cost, funding source

Cost may be an issue. Spiriva and Serevent, the brand names for tiotropium and salmeterol, respectively, are second-tier medications on several formularies, and tiotropium is about 45% more expensive (tiotropium=$262, salmeterol=$181 for one month’s supply; www.drugstore.com, accessed January 19, 2012). There are also several long-acting beta-agonists in development that will be dosed once daily; once they’re approved, tiotropium’s once-a-day dosing may no longer be seen as an advantage.

It is also worth noting that this trial was supported by Boehringer Ingelheim and Pfizer, which jointly market Spiriva.

Finally, smoking must be addressed. Strongly encouraging patients to kick the habit is still the most important intervention we can make in helping to improve the quality of life, and survival, of patients with COPD.

CHALLENGES TO IMPLEMENTATION: COPD guidelines need updating

There are no major challenges to incorporating this recommendation into clinical practice; the key challenge lies in diagnosing COPD and adequately monitoring and helping patients manage the disease.

Current guidelines do not distinguish between the efficacy of long-acting bronchodilators, but findings from this study are important enough to change future versions of national guidelines. The GOLD committee is due to release a new guideline report soon, and will likely update its recommendations at that time.

Acknowledgement

The PURLs Surveillance System is supported in part by Grant Number UL1RR024999 from the National Center for Research Resources, a Clinical Translational Science Award to the University of Chicago. The content is solely the responsibility of the authors and does not necessarily represent the official views of the National Center for Research Resources or the National Institutes of Health.

Click here to view PURL METHODOLOGY

PRACTICE CHANGER

Consider adding tiotropium to the medication regimen of patients with moderate to very severe chronic obstructive pulmonary disease (COPD), as a multinational study found it to be more effective than salmeterol in preventing exacerbations.1

STRENGTH OF RECOMMENDATION

A: Based on one well-designed randomized controlled trial.

Vogelmeier C, Hederer B, Glaab T, et al; POET-COPD investigators. Tiotropium versus salmeterol for the prevention of exacerbations of COPD. N Engl J Med. 2011;364:1093-1103.

 

ILLUSTRATIVE CASE

A 60-year-old patient with moderate COPD and a history of frequent exacerbations comes in for a follow-up visit. She has been using albuterol and ipratropium intermittently. you want to add a longer-acting bronchodilator and wonder if tiotropium or salmeterol is more effective for reducing exacerbations.

COPD is the fourth leading cause of death in the United States.2 More than 12 million Americans have been diagnosed with COPD, and it is estimated that another 12 million would have a COPD diagnosis if all smokers older than 45 years underwent spirometry.2 The disorder accounts for some 16 million physician visits each year and costs the US health care system approximately $19 billion annually, with acute exacerbations and hospitalizations representing 58% of the total.2,3

Despite guidelines, COPD is often undertreated
One of the main goals of COPD treatment is to reduce the frequency and intensity of acute exacerbations, both to improve patients’ quality of life and reduce health care costs. The Global Initiative for Chronic Obstructive Lung Disease (GOLD) has developed guidelines for effective management of COPD, which recommend long-acting bronchodilators as first-line maintenance therapy for patients whose disease is moderate to very severe.4

Evidence suggests that physicians frequently undertreat moderate to severe COPD, however, following national guidelines only about a quarter of the time.5 This is, in part, because many clinicians doubt the efficacy of COPD treatment for improving symptoms or decreasing exacerbations.5,6 Yet studies have shown that the long-acting broncho dilators tiotropium (an anticholinergic agent) and salmeterol (a beta2-adrenergic agonist), used with or without inhaled corticosteroids, are effective in reducing the frequency of COPD exacerbations, improving quality of life and lung function, and reducing the number of hospitalizations.7-10

Long-acting bronchodilators are therefore clearly indicated but, until recently, there was little evidence as to which one is better.

STUDY SUMMARY: Tiotropium group had fewer exacerbations…

The Prevention Of Exacerbations with Tiotropium in COPD (POET-COPD) trial compared tiotropium with salmeterol for their ability to prevent exacerbations.1 This was a randomized double-blind trial of 7376 patients with moderate to very severe COPD diagnosed by spirometry. Participants were recruited from 725 medical centers in 25 countries. To be eligible, they had to be ≥40 years, with at least a 10 pack-year history of smoking, a forced expiratory volume in 1 second (FEV1) <70% predicted, an FEV1/forced vital capacity (FVC ) <70%, and at least one exacerbation in the previous year.

Patients were randomly assigned to either the tiotropium or the salmeterol group. Those on tiotropium received a daily dose of 18 mcg through a HandiHaler device, plus a placebo with a metered-dose inhaler twice a day. Patients in the other group received 50 mcg salmeterol through a metered-dose inhaler twice daily, plus a placebo with a HandiHaler once a day. These medications were in addition to patients’ current medication regimens, including inhaled corticosteroids, with this exception: Use of anticholinergics and long-acting beta-agonists was discontinued for the course of the trial.

All participants were followed for one year, with clinic visits at 2, 4, 8, and 12 months to assess for medication adherence and symptoms of exacerbation. The primary endpoint was the time to first exacerbation. This was defined as an increase in, or a new onset of, more than one symptom of COPD (ie, cough, sputum production, wheezing, dyspnea, and chest tightness), with at least one symptom lasting ≥3 days and leading to treatment with glucocorticoids and/or antibiotics, or hospitalization. Secondary outcomes were times to first moderate and severe exacerbations and use of steroids and antibiotics.

There were significant differences in several outcomes. The time to first exacerbation was 187 days for tiotropium vs 145 days for salmeterol, a difference of 42 days (hazard ratio [HR]=0.83; 95% confidence interval [CI], 0.77-0.90; P<.001). In addition, tiotropium reduced the annual number of exacerbations compared with salmeterol (rate ratio=0.89; 95% CI, 0.83-0.96; P=.002), with a number needed to treat (NNT) of 24 patients to prevent one moderate to severe exacerbation per year.

 

 

 

…and used fewer drugs

Compared with salmeterol, there was a 14% reduction in risk of a moderate exacerbation associated with tiotropium (HR=0.86; 95% CI, 0.79-0.93; P<.001; NNT=32) and a 28% reduction in risk of a severe exacerbation (HR=0.72; 95% CI, 0.61-0.85; P<.001; NNT=48). In addition, the tiotropium group had a 23% risk reduction in the use of systemic glucocorticoids (HR=0.77; 95% CI, 0.69-0.85; P<.001; NNT=26) compared with the salmeterol group, and a 15% risk reduction in the use of antibiotics (HR=0.85; 95% CI, 0.78-0.92; P<0.001; NNT=31). The difference in reduction in death rates between the 2 groups was not statistically significant.

The observed differences were consistent across all major subgroups (age, sex, smoking status, and severity of COPD) of patients studied. Interestingly, patients with low BMI or very severe COPD appeared to benefit the most from tiotropium.

WHAT’S NEW: The difference between 2 agents is clear

Although national guidelines recommend long-acting bronchodilators for COPD that is moderate or worse, there have been few data to guide clinicians in determining which one to use. The findings of this study suggest that tiotropium should be our first choice. Tiotropium’s once-a-day dosing is an additional benefit, as patients using it will likely have better compliance than those using twice-daily salmeterol. The data may also prompt development of a once-daily inhaled corticosteroid/ long-acting anticholinergic combination.

CAVEATS: Cost, funding source

Cost may be an issue. Spiriva and Serevent, the brand names for tiotropium and salmeterol, respectively, are second-tier medications on several formularies, and tiotropium is about 45% more expensive (tiotropium=$262, salmeterol=$181 for one month’s supply; www.drugstore.com, accessed January 19, 2012). There are also several long-acting beta-agonists in development that will be dosed once daily; once they’re approved, tiotropium’s once-a-day dosing may no longer be seen as an advantage.

It is also worth noting that this trial was supported by Boehringer Ingelheim and Pfizer, which jointly market Spiriva.

Finally, smoking must be addressed. Strongly encouraging patients to kick the habit is still the most important intervention we can make in helping to improve the quality of life, and survival, of patients with COPD.

CHALLENGES TO IMPLEMENTATION: COPD guidelines need updating

There are no major challenges to incorporating this recommendation into clinical practice; the key challenge lies in diagnosing COPD and adequately monitoring and helping patients manage the disease.

Current guidelines do not distinguish between the efficacy of long-acting bronchodilators, but findings from this study are important enough to change future versions of national guidelines. The GOLD committee is due to release a new guideline report soon, and will likely update its recommendations at that time.

Acknowledgement

The PURLs Surveillance System is supported in part by Grant Number UL1RR024999 from the National Center for Research Resources, a Clinical Translational Science Award to the University of Chicago. The content is solely the responsibility of the authors and does not necessarily represent the official views of the National Center for Research Resources or the National Institutes of Health.

Click here to view PURL METHODOLOGY

References

1. Vogelmeier C, Hederer B, Glaab T, et al. POET-COPD investigators. Tiotropium versus salmeterol for the prevention of exacerbations of COPD. N Engl J Med. 2011;364:1093-1103.

2. National Heart, Lung, and Blood Institute. Morbidity and mortality: 2009 chart book on cardiovascular, lung, and blood diseases. Available at: http://www.nhlbi.nih.gov/resources/docs/04chtbk.pdf. Accessed October 1, 2011.

3. Miravitlles M, Murio C, Guerrero T, et al. DAFNE Study Group. Pharmacoeconomic evaluation of acute exacerbations of chronic bronchitis and COPD. Chest. 2002;121:1449-1455.

4. Global Initiative for Chronic Obstructive Lung Disease. Global strategy for the diagnosis, management, and prevention of chronic obstructive pulmonary disease. Updated 2010. Available at: http://www.goldcopd.org. Accessed October 1, 2011.

5. Salinas GD, Williamson JC, Kalhan R, et al. Barriers to adherence to chronic obstructive pulmonary disease guidelines by primary care physicians. Int J Chron Obstruct Pulmon Dis. 2011;6:171-179.

6. Yawn BP, Wollan PC. Knowledge and attitudes of family physicians coming to COPD continuing medical education. Int J Chron Obstruct Pulmon Dis. 2008;3:311-318.

7. Calverly PMA, Anderson JA, Celli B, et al. Salmeterol and fluticasone propionate and survival in chronic obstructive pulmonary disease. N Engl J Med. 2007;356:775-789.

8. Casaburi R, Mahler DA, Jones PW, et al. A long-term evaluation of once-daily inhaled tiotropium in chronic obstructive pulmonary disease. Eur Respir J. 2002;19:217-224.

9. Donahue JF, van Noord JA, Bateman ED, et al. A 6-month, placebo-controlled study comparing lung function and health status changes in COPD patients treated with tiotropium or salmeterol. Chest. 2002;122:47-55.

10. Tashkin DP, Celli B, Senn S, et al. A 4-year trial of tiotropium in chronic obstructive pulmonary disease. N Engl J Med. 2008;359:1543-1554.

References

1. Vogelmeier C, Hederer B, Glaab T, et al. POET-COPD investigators. Tiotropium versus salmeterol for the prevention of exacerbations of COPD. N Engl J Med. 2011;364:1093-1103.

2. National Heart, Lung, and Blood Institute. Morbidity and mortality: 2009 chart book on cardiovascular, lung, and blood diseases. Available at: http://www.nhlbi.nih.gov/resources/docs/04chtbk.pdf. Accessed October 1, 2011.

3. Miravitlles M, Murio C, Guerrero T, et al. DAFNE Study Group. Pharmacoeconomic evaluation of acute exacerbations of chronic bronchitis and COPD. Chest. 2002;121:1449-1455.

4. Global Initiative for Chronic Obstructive Lung Disease. Global strategy for the diagnosis, management, and prevention of chronic obstructive pulmonary disease. Updated 2010. Available at: http://www.goldcopd.org. Accessed October 1, 2011.

5. Salinas GD, Williamson JC, Kalhan R, et al. Barriers to adherence to chronic obstructive pulmonary disease guidelines by primary care physicians. Int J Chron Obstruct Pulmon Dis. 2011;6:171-179.

6. Yawn BP, Wollan PC. Knowledge and attitudes of family physicians coming to COPD continuing medical education. Int J Chron Obstruct Pulmon Dis. 2008;3:311-318.

7. Calverly PMA, Anderson JA, Celli B, et al. Salmeterol and fluticasone propionate and survival in chronic obstructive pulmonary disease. N Engl J Med. 2007;356:775-789.

8. Casaburi R, Mahler DA, Jones PW, et al. A long-term evaluation of once-daily inhaled tiotropium in chronic obstructive pulmonary disease. Eur Respir J. 2002;19:217-224.

9. Donahue JF, van Noord JA, Bateman ED, et al. A 6-month, placebo-controlled study comparing lung function and health status changes in COPD patients treated with tiotropium or salmeterol. Chest. 2002;122:47-55.

10. Tashkin DP, Celli B, Senn S, et al. A 4-year trial of tiotropium in chronic obstructive pulmonary disease. N Engl J Med. 2008;359:1543-1554.

Issue
The Journal of Family Practice - 61(02)
Issue
The Journal of Family Practice - 61(02)
Page Number
94-96
Page Number
94-96
Publications
Publications
Topics
Article Type
Display Headline
Would this long-acting bronchodilator be better for your patient?
Display Headline
Would this long-acting bronchodilator be better for your patient?
Sections
PURLs Copyright

Copyright © 2012 The Family Physicians Inquiries Network.
All rights reserved.

Disallow All Ads
Alternative CME
Article PDF Media

Combatting lice in a single treatment

Article Type
Changed
Mon, 01/14/2019 - 11:33
Display Headline
Combatting lice in a single treatment
PRACTICE CHANGER

Consider prescribing spinosad—a recently approved topical pediculicide that is more effective than permethrin and eliminates the need for nit combingfor children with head lice.1

Stough D, Shellabarger S, Quiring J, et al. Efficacy and safety of spinosad and permethrin crème rinses for pediculosis capitis (head lice). Pediatrics. 2009;124:e389-e395.

STRENGTH OF RECOMMENDATION

B: Based on a single randomized controlled trial (RCT).

 

ILLUSTRATIVE CASE

A 6-year-old girl is brought in by her mother, who has just been notified of an outbreak of head lice in the child’s school. You examine her scalp and observe viable eggs (nits) at the hair shaft, and tell mom that her daughter has head lice. What’s the most effective treatment?

Head lice are common and easily spread among schoolchildren—and a source of frustration for patients, parents, and family physicians alike. Permethrin cream 1% (Nix), one of the most widely used and well-studied pediculicides, is available over the counter.2

Prescription pediculicides, including malathion (Ovide) and lindane (Kwell), are used less frequently due to their side effect profiles: Malathion is highly flammable due to a high alcohol content and lindane can cause neurotoxicity.2 Both are typically reserved for cases resistant to permethrin 1% cream. Local resistance patterns influence physician prescribing, of course, and greater resistance has increased the need for multiple treatments.3

Dual treatment—and combing— are often required
Permethrin is pediculicidal (ie, it kills only live lice); it is not ovicidal (ie, it does not kill nits at every stage).4,5 Thus, a second application is usually needed 7 to 10 days after the first to ensure complete eradication. And, because permethrin is not ovicidal, nit combing—a tedious, time-consuming, and often painful processis required to remove lice and unhatched eggs from the hair shafts. Another downside: Children in districts that do not allow students to return to school until they are completely nit-free often miss school as a result of this lengthy process.

The US Food and Drug Administration (FDA) recently approved spinosad (Natroba topical suspension 0.9% cream rinse) for the treatment of lice in patients ≥4 years old.6 Spinosad, which is available only by prescription, eliminates the need for nit combing. But how well does it work?

STUDY SUMMARY: For most patients, a single application is sufficient

The study by Stough et al included 2 identical multicenter RCTs comparing 0.9% spinosad without nit combing vs 1% permethrin with combing in patients ≥6 months old who had active head lice. There were 446 patients in the primary treatment (spinosad) group and 470 in the control (permethrin) group. All participants within the same household were assigned to the same group.

Participants were evaluated on Days 1, 7, and 14. Those with active lice on Day 7 received another treatment (a second application with the same medication). The primary outcome was the percentage of participants who were lice free on Day 14.

 

 

 

The findings: After 14 days, 85% of the children in the spinosad group vs 44% of those in the permethrin group were lice free. In addition, 75% of those treated with spinosad required only one application for a cure, compared with 37% in the permethrin group.

There were no serious adverse events in either group. Participants in the permethrin group were more likely to experience scalp erythema than those in the spinosad group (6.8% vs 3.1%; P<.007). Complete metabolic panels and hematology studies were performed, and no significant differences pre- or posttreatment were found. The dropout rate was <10%, for similar reasons in both groups.

WHAT’S NEW: Extensive nit combing no longer needed

Spinosad is pediculicidal and ovicidal killing lice and their eggs in all stages—thereby eliminating the need for extensive combing in most cases. It has less resistance than current pediculicides, so a second course of treatment is needed only about one-quarter of the time. Spinosad received FDA approval for use in patients ≥4 years in 2011.

CAVEATS: Funding of study raises question of bias

The study by Stough et al was funded by ParaPRO, LLC, the manufacturer of Natroba. The company had access to the data and had a role in the interpretation of the findings and the decision to publish the study. However, the assessors were blinded to treatment group.

CHALLENGES TO iMPLEMENTATION: Cost is high and may not be covered

Cost will be a major barrier to treatment for many families. The average cost of an application of permethrin 1% is about $20; an application of spinosad costs $270 without insurance. Individual health plans may not cover it or may require prior authorization.

Acknowledgement

The PURLs Surveillance System is supported in part by Grant Number UL1RR024999 from the National Center for Research Resources, a Clinical Translational Science Award to the University of Chicago. The content is solely the responsibility of the authors and does not necessarily represent the official views of the National Center for Research Resources or the National Institutes of Health.

Click here to view PURL METHODOLOGY

References

1. Stough D, Shellabarger S, Quiring J, et al. Efficacy and Safety of spinosad and permethrin creme rinses for pediculosis capitis (head lice). Pediatrics. 2009;124:e389-e395.

2. Frankowski BL, Weiner LB. American Academy of Pediatrics, Committee on School Health Committee on Infectious Diseases. Head lice. Pediatrics. 2010;126:392-403.

3. Jones KN, English JC. 3rd. Review of common therapeutic options in the US for the treatment of pediculosis capitis. Clinl infect Dis. 2003;36:1355-1361.

4. Flinders D, Schweinitz P. Pediculosis and scabies. Am Fam Physician. 2004;69:341-350.

5. Centers for Disease Control and Prevention Parasites–lice. Available at http://www.cdc.gov/parasites/lice/. Accessed December 14, 2011.

6. US Food and Drug Administration Drugs@FDA. Available at: http://www.accessdata.fda.gov/scripts/cder/drugsatfda/index.cfm?fuseaction=Search. Drug Details. Accessed December 14, 2011.

Author and Disclosure Information

Dionna Brown, MD
The University of Chicago

Kate Rowland, MD
The University of Chicago

PURLs EDITOR
John Hickner, MD, MSc
Cleveland Clinic

Issue
The Journal of Family Practice - 61(1)
Publications
Topics
Page Number
41-42
Legacy Keywords
Dionna Brown;MD; Kate Rowland;MD; PURLs editor John Hickner;MD;MSc; combatting lice; spinosad; Natroba; topical pediculicide; permethrin cream; Ovide; Kwell; lindane; nit-picking; combing
Sections
Author and Disclosure Information

Dionna Brown, MD
The University of Chicago

Kate Rowland, MD
The University of Chicago

PURLs EDITOR
John Hickner, MD, MSc
Cleveland Clinic

Author and Disclosure Information

Dionna Brown, MD
The University of Chicago

Kate Rowland, MD
The University of Chicago

PURLs EDITOR
John Hickner, MD, MSc
Cleveland Clinic

PRACTICE CHANGER

Consider prescribing spinosad—a recently approved topical pediculicide that is more effective than permethrin and eliminates the need for nit combingfor children with head lice.1

Stough D, Shellabarger S, Quiring J, et al. Efficacy and safety of spinosad and permethrin crème rinses for pediculosis capitis (head lice). Pediatrics. 2009;124:e389-e395.

STRENGTH OF RECOMMENDATION

B: Based on a single randomized controlled trial (RCT).

 

ILLUSTRATIVE CASE

A 6-year-old girl is brought in by her mother, who has just been notified of an outbreak of head lice in the child’s school. You examine her scalp and observe viable eggs (nits) at the hair shaft, and tell mom that her daughter has head lice. What’s the most effective treatment?

Head lice are common and easily spread among schoolchildren—and a source of frustration for patients, parents, and family physicians alike. Permethrin cream 1% (Nix), one of the most widely used and well-studied pediculicides, is available over the counter.2

Prescription pediculicides, including malathion (Ovide) and lindane (Kwell), are used less frequently due to their side effect profiles: Malathion is highly flammable due to a high alcohol content and lindane can cause neurotoxicity.2 Both are typically reserved for cases resistant to permethrin 1% cream. Local resistance patterns influence physician prescribing, of course, and greater resistance has increased the need for multiple treatments.3

Dual treatment—and combing— are often required
Permethrin is pediculicidal (ie, it kills only live lice); it is not ovicidal (ie, it does not kill nits at every stage).4,5 Thus, a second application is usually needed 7 to 10 days after the first to ensure complete eradication. And, because permethrin is not ovicidal, nit combing—a tedious, time-consuming, and often painful processis required to remove lice and unhatched eggs from the hair shafts. Another downside: Children in districts that do not allow students to return to school until they are completely nit-free often miss school as a result of this lengthy process.

The US Food and Drug Administration (FDA) recently approved spinosad (Natroba topical suspension 0.9% cream rinse) for the treatment of lice in patients ≥4 years old.6 Spinosad, which is available only by prescription, eliminates the need for nit combing. But how well does it work?

STUDY SUMMARY: For most patients, a single application is sufficient

The study by Stough et al included 2 identical multicenter RCTs comparing 0.9% spinosad without nit combing vs 1% permethrin with combing in patients ≥6 months old who had active head lice. There were 446 patients in the primary treatment (spinosad) group and 470 in the control (permethrin) group. All participants within the same household were assigned to the same group.

Participants were evaluated on Days 1, 7, and 14. Those with active lice on Day 7 received another treatment (a second application with the same medication). The primary outcome was the percentage of participants who were lice free on Day 14.

 

 

 

The findings: After 14 days, 85% of the children in the spinosad group vs 44% of those in the permethrin group were lice free. In addition, 75% of those treated with spinosad required only one application for a cure, compared with 37% in the permethrin group.

There were no serious adverse events in either group. Participants in the permethrin group were more likely to experience scalp erythema than those in the spinosad group (6.8% vs 3.1%; P<.007). Complete metabolic panels and hematology studies were performed, and no significant differences pre- or posttreatment were found. The dropout rate was <10%, for similar reasons in both groups.

WHAT’S NEW: Extensive nit combing no longer needed

Spinosad is pediculicidal and ovicidal killing lice and their eggs in all stages—thereby eliminating the need for extensive combing in most cases. It has less resistance than current pediculicides, so a second course of treatment is needed only about one-quarter of the time. Spinosad received FDA approval for use in patients ≥4 years in 2011.

CAVEATS: Funding of study raises question of bias

The study by Stough et al was funded by ParaPRO, LLC, the manufacturer of Natroba. The company had access to the data and had a role in the interpretation of the findings and the decision to publish the study. However, the assessors were blinded to treatment group.

CHALLENGES TO iMPLEMENTATION: Cost is high and may not be covered

Cost will be a major barrier to treatment for many families. The average cost of an application of permethrin 1% is about $20; an application of spinosad costs $270 without insurance. Individual health plans may not cover it or may require prior authorization.

Acknowledgement

The PURLs Surveillance System is supported in part by Grant Number UL1RR024999 from the National Center for Research Resources, a Clinical Translational Science Award to the University of Chicago. The content is solely the responsibility of the authors and does not necessarily represent the official views of the National Center for Research Resources or the National Institutes of Health.

Click here to view PURL METHODOLOGY

PRACTICE CHANGER

Consider prescribing spinosad—a recently approved topical pediculicide that is more effective than permethrin and eliminates the need for nit combingfor children with head lice.1

Stough D, Shellabarger S, Quiring J, et al. Efficacy and safety of spinosad and permethrin crème rinses for pediculosis capitis (head lice). Pediatrics. 2009;124:e389-e395.

STRENGTH OF RECOMMENDATION

B: Based on a single randomized controlled trial (RCT).

 

ILLUSTRATIVE CASE

A 6-year-old girl is brought in by her mother, who has just been notified of an outbreak of head lice in the child’s school. You examine her scalp and observe viable eggs (nits) at the hair shaft, and tell mom that her daughter has head lice. What’s the most effective treatment?

Head lice are common and easily spread among schoolchildren—and a source of frustration for patients, parents, and family physicians alike. Permethrin cream 1% (Nix), one of the most widely used and well-studied pediculicides, is available over the counter.2

Prescription pediculicides, including malathion (Ovide) and lindane (Kwell), are used less frequently due to their side effect profiles: Malathion is highly flammable due to a high alcohol content and lindane can cause neurotoxicity.2 Both are typically reserved for cases resistant to permethrin 1% cream. Local resistance patterns influence physician prescribing, of course, and greater resistance has increased the need for multiple treatments.3

Dual treatment—and combing— are often required
Permethrin is pediculicidal (ie, it kills only live lice); it is not ovicidal (ie, it does not kill nits at every stage).4,5 Thus, a second application is usually needed 7 to 10 days after the first to ensure complete eradication. And, because permethrin is not ovicidal, nit combing—a tedious, time-consuming, and often painful processis required to remove lice and unhatched eggs from the hair shafts. Another downside: Children in districts that do not allow students to return to school until they are completely nit-free often miss school as a result of this lengthy process.

The US Food and Drug Administration (FDA) recently approved spinosad (Natroba topical suspension 0.9% cream rinse) for the treatment of lice in patients ≥4 years old.6 Spinosad, which is available only by prescription, eliminates the need for nit combing. But how well does it work?

STUDY SUMMARY: For most patients, a single application is sufficient

The study by Stough et al included 2 identical multicenter RCTs comparing 0.9% spinosad without nit combing vs 1% permethrin with combing in patients ≥6 months old who had active head lice. There were 446 patients in the primary treatment (spinosad) group and 470 in the control (permethrin) group. All participants within the same household were assigned to the same group.

Participants were evaluated on Days 1, 7, and 14. Those with active lice on Day 7 received another treatment (a second application with the same medication). The primary outcome was the percentage of participants who were lice free on Day 14.

 

 

 

The findings: After 14 days, 85% of the children in the spinosad group vs 44% of those in the permethrin group were lice free. In addition, 75% of those treated with spinosad required only one application for a cure, compared with 37% in the permethrin group.

There were no serious adverse events in either group. Participants in the permethrin group were more likely to experience scalp erythema than those in the spinosad group (6.8% vs 3.1%; P<.007). Complete metabolic panels and hematology studies were performed, and no significant differences pre- or posttreatment were found. The dropout rate was <10%, for similar reasons in both groups.

WHAT’S NEW: Extensive nit combing no longer needed

Spinosad is pediculicidal and ovicidal killing lice and their eggs in all stages—thereby eliminating the need for extensive combing in most cases. It has less resistance than current pediculicides, so a second course of treatment is needed only about one-quarter of the time. Spinosad received FDA approval for use in patients ≥4 years in 2011.

CAVEATS: Funding of study raises question of bias

The study by Stough et al was funded by ParaPRO, LLC, the manufacturer of Natroba. The company had access to the data and had a role in the interpretation of the findings and the decision to publish the study. However, the assessors were blinded to treatment group.

CHALLENGES TO iMPLEMENTATION: Cost is high and may not be covered

Cost will be a major barrier to treatment for many families. The average cost of an application of permethrin 1% is about $20; an application of spinosad costs $270 without insurance. Individual health plans may not cover it or may require prior authorization.

Acknowledgement

The PURLs Surveillance System is supported in part by Grant Number UL1RR024999 from the National Center for Research Resources, a Clinical Translational Science Award to the University of Chicago. The content is solely the responsibility of the authors and does not necessarily represent the official views of the National Center for Research Resources or the National Institutes of Health.

Click here to view PURL METHODOLOGY

References

1. Stough D, Shellabarger S, Quiring J, et al. Efficacy and Safety of spinosad and permethrin creme rinses for pediculosis capitis (head lice). Pediatrics. 2009;124:e389-e395.

2. Frankowski BL, Weiner LB. American Academy of Pediatrics, Committee on School Health Committee on Infectious Diseases. Head lice. Pediatrics. 2010;126:392-403.

3. Jones KN, English JC. 3rd. Review of common therapeutic options in the US for the treatment of pediculosis capitis. Clinl infect Dis. 2003;36:1355-1361.

4. Flinders D, Schweinitz P. Pediculosis and scabies. Am Fam Physician. 2004;69:341-350.

5. Centers for Disease Control and Prevention Parasites–lice. Available at http://www.cdc.gov/parasites/lice/. Accessed December 14, 2011.

6. US Food and Drug Administration Drugs@FDA. Available at: http://www.accessdata.fda.gov/scripts/cder/drugsatfda/index.cfm?fuseaction=Search. Drug Details. Accessed December 14, 2011.

References

1. Stough D, Shellabarger S, Quiring J, et al. Efficacy and Safety of spinosad and permethrin creme rinses for pediculosis capitis (head lice). Pediatrics. 2009;124:e389-e395.

2. Frankowski BL, Weiner LB. American Academy of Pediatrics, Committee on School Health Committee on Infectious Diseases. Head lice. Pediatrics. 2010;126:392-403.

3. Jones KN, English JC. 3rd. Review of common therapeutic options in the US for the treatment of pediculosis capitis. Clinl infect Dis. 2003;36:1355-1361.

4. Flinders D, Schweinitz P. Pediculosis and scabies. Am Fam Physician. 2004;69:341-350.

5. Centers for Disease Control and Prevention Parasites–lice. Available at http://www.cdc.gov/parasites/lice/. Accessed December 14, 2011.

6. US Food and Drug Administration Drugs@FDA. Available at: http://www.accessdata.fda.gov/scripts/cder/drugsatfda/index.cfm?fuseaction=Search. Drug Details. Accessed December 14, 2011.

Issue
The Journal of Family Practice - 61(1)
Issue
The Journal of Family Practice - 61(1)
Page Number
41-42
Page Number
41-42
Publications
Publications
Topics
Article Type
Display Headline
Combatting lice in a single treatment
Display Headline
Combatting lice in a single treatment
Legacy Keywords
Dionna Brown;MD; Kate Rowland;MD; PURLs editor John Hickner;MD;MSc; combatting lice; spinosad; Natroba; topical pediculicide; permethrin cream; Ovide; Kwell; lindane; nit-picking; combing
Legacy Keywords
Dionna Brown;MD; Kate Rowland;MD; PURLs editor John Hickner;MD;MSc; combatting lice; spinosad; Natroba; topical pediculicide; permethrin cream; Ovide; Kwell; lindane; nit-picking; combing
Sections
PURLs Copyright

Copyright © 2012 The Family Physicians Inquiries Network.
All rights reserved.

Disallow All Ads
Alternative CME

Offer this contraceptive to breastfeeding new moms

Article Type
Changed
Mon, 01/14/2019 - 11:33
Display Headline
Offer this contraceptive to breastfeeding new moms
PRACTICE CHANGER

Recommend the etonogestrel implant to new mothers who plan to breastfeed; the insertion of this contraceptive within the first few days postpartum does not alter breastfeeding outcomes.1

STRENGTH OF RECOMMENDATION

B: Based on a single high-quality randomized controlled trial (RCT).

Gurtcheff SE, Turok DK, Stoddard G, et al. Lactogenesis after early postpartum use of the contraceptive implant. Obstet Gynecol. 2011;117:1114-1121.

 

ILLUSTRATIVE CASE

In the last trimester of pregnancy, a patient asks about her options for postpartum contraception. She plans to breastfeed and does not want to have another child for several years, she says. Her family is scheduled to move 2 weeks after her due date, and she wants to begin using contraception before then. She’s interested in the etonogestrel implant (Implanon) and wonders whether she can have it inserted before she leaves the hospital. What can you tell her?

Approximately 4 million women give birth each year in the United States,2 77% of whom choose to breastfeed their babies.3 Postpartum contraception is recommended, to ensure adequate spacing between (or prevention of) pregnancies.

Hormonal options are limited for nursing moms
Due to the negative effect of estrogens on lactation,4 women who wish to use birth control while breastfeeding have limited choices. Their options include progestin-only oral contraceptives; intrauterine devices, including the levonorgestrel intrauterine contraceptive; barrier methods; and the etonogestrel implant. Yet concerns remain that using a progestin contraceptive in the early postpartum period could negatively affect lactogenesis, as well as the quantity and quality of the breast milk.5

Starting contraception after 6 weeks? The opportunity is often missed
A 2010 systematic review found that progestin-only contraception can be safely used in breastfeeding women. However, the studies included in the review did not consider timing. Thus, the researchers concluded only that initiation of a progestin contraception >6 weeks postpartum is safe.6 The World Health Organization recommends waiting >6 weeks, as well.7 But studies have found that between 10% and 40% of women miss their 6-week postpartum visit,8 thereby missing the opportunity to start contraception.

A 2009 pilot study found that the implant can be safely used <4 weeks postpartum, and did not affect breastfeeding.9 The study we review here is the first RCT to evaluate the impact of early insertion (1-3 days postpartum) of the etonogestrel implant on lactogenesis.

STUDY SUMMARY: Timing of implant did not affect outcomes

The study by Gurtcheff et al was a randomized controlled noninferiority trial of 69 women who wanted to use Implanon for postpartum birth control. Inclusion criteria included good health (of the babies as well as the mothers), the intention to breastfeed, and the willingness to be randomly assigned to either early (1-3 days) or standard (4-8 weeks) insertion. The study was not blinded. No other source of bias was identified.

The primary outcomes studied were time to stage II of lactogenesis (based on maternal perception of when her milk “had come in”) and rates of lactation failure.

Early insertion, the researchers found, was noninferior to standard insertion, both in the time to stage II of lactogenesis and the risk of lactation failure. The time to lactogenesis was 64.3 hours (mean standard deviation [SD], 19.6 hours) for early insertion vs 65.2 hours (mean SD, 18.5 hours) for standard insertion. The mean difference was -1.4 hours (95% confidence interval [CI], -10.6 to 7.7 hours). For lactation failure, the absolute risk difference was 0.03 (95% CI, -0.02 to 0.08).

Secondary outcomes included breastfeeding status, side effects, and bleeding patterns, as well as the contraceptive method actually being used at the time. This information was gathered at 2 weeks, 6 weeks, 3 months, and 6 months postpartum.

There were no statistically significant differences in breastfeeding, formula supplementation, or patient-reported bleeding patterns. However, a third of the women (11 of 34) in the standard group did not have the implant inserted, and opted for an alternate form of birth control.

At 6 weeks, women in both groups had a milk sample analyzed for fat and energy content. There was no significant difference in mean creamatocrit values between the groups.

 

 

 

WHAT’S NEW: Early insertion is safe and fosters compliance

Lactogenesis and lactation failure rates were comparable, whether the etonogestrel implant was inserted between 1 and 3 days postpartum or 4 to 8 weeks postpartum. An advantage of early insertion was increased contraceptive compliance. At 3 months postpartum, 13% of the women in the standard group were not using any birth control. Among those in the early insertion group, compliance was 100%.

CAVEATS: Study sample may not be representative

This was a small study, but it was powered to detect ≥8 hour difference in onset of stage II lactogenesis. Participants were not representative of all populations (91% were white, 73% of whom were Hispanic). Both the mothers and babies were healthy, so we can’t extrapolate to situations where either mom or baby is sick.

CHALLENGES TO IMPLEMENTATION: Finding clinicians trained in insertion technique

Health care providers trained in insertion of the etonogestrel implant would need to be available to promote insertion in the early postpartum period. Ensuring availability of the device in hospitals may require extra logistical planning; incorporating etonogestrel implant insertion into already-hectic morning rounds may be challenging, as well.

Acknowledgement

The PURLs Surveillance System is supported in part by Grant Number UL1RR024999 from the National Center for Research Resources, a Clinical Translational Science Award to the University of Chicago. The content is solely the responsibility of the authors and does not necessarily represent the official views of the National Center for Research Resources or the National Institutes of Health.

Click here to view PURL METHODOLOGY

References

1. Gurtcheff SE, Turok DK, Stoddard G, et al. Lactogenesis after early postpartum use of the contraceptive implant. Obstet Gynecol. 2011;117:1114-1121.

2. American Pregnancy Association. Statistics. Available at: http://www.americanpregnancy.org/main/statistics/html. Accessed November 16, 2011.

3. Centers for Disease Control and Prevention. NCHS data brief. Breastfeeding in the United States: findings from the National Health and Nutrition Examination Survey, 1999-2006. April 2008. Available at: http://www.cdc.gov/nchs/data/databriefs/db05.htm. Accessed November 16, 2011.

4. Tankeyoon M, Dusitsin N, Chalapati S, et al. Effects of hormonal contraceptives on milk volume and infant growth. WHO special programme of research, development and research training in human reproduction task force on oral contraceptives. Contraception. 1984;30:505-522.

5. Kennedy KI, Short RV, Tully MR. Premature introduction of progestin-only contraceptive methods during lactation. Contraception. 1997;55:347-350.

6. Kapp N, Curtis K, Nanda K. Progestogen-only contraceptive use among breastfeeding women: a systematic review. Contraception. 2010;82:17-37.

7. World Health Organization medical eligibility criteria wheel for contraceptive use (2008 update). Available at: http://www.who.int/reproductivehealth/publications/family_planning/wheel_v4_2010_EN.swf. Accessed November 16, 2011.

8. Centers for Disease Control and Prevention (CDC). Postpartum care visits—11 states and New York City, 2004. MMWR Morb Mortal Wkly Rep. 2007;56:1312-1316.Available at: http://www.cdc.gov/mmwr/preview/mmwrhtml/mm5650a2.htm. Accessed November 16, 2011.

9. Brito MB, Ferriani RA, Quintana SM. Safety of the etonogestrel-releasing implant during the immediate postpartum period: a pilot study. Contraception. 2009;80:519-526.

Article PDF
Author and Disclosure Information

Kohar Jones, MD
Department of Family, Medicine, University of Chicago Medical Center

Mari Egan, MD
Department of Family, Medicine, University of Chicago Medical Center

James J. Stevermer, MD, MSPH
Department of Family and Community Medicine, University of Missouri-Columbia

PURLs EDITOR
John Hickner, MD, MSc
Cleveland Clinic

Issue
The Journal of Family Practice - 60(12)
Publications
Topics
Page Number
744-746
Sections
Author and Disclosure Information

Kohar Jones, MD
Department of Family, Medicine, University of Chicago Medical Center

Mari Egan, MD
Department of Family, Medicine, University of Chicago Medical Center

James J. Stevermer, MD, MSPH
Department of Family and Community Medicine, University of Missouri-Columbia

PURLs EDITOR
John Hickner, MD, MSc
Cleveland Clinic

Author and Disclosure Information

Kohar Jones, MD
Department of Family, Medicine, University of Chicago Medical Center

Mari Egan, MD
Department of Family, Medicine, University of Chicago Medical Center

James J. Stevermer, MD, MSPH
Department of Family and Community Medicine, University of Missouri-Columbia

PURLs EDITOR
John Hickner, MD, MSc
Cleveland Clinic

Article PDF
Article PDF
PRACTICE CHANGER

Recommend the etonogestrel implant to new mothers who plan to breastfeed; the insertion of this contraceptive within the first few days postpartum does not alter breastfeeding outcomes.1

STRENGTH OF RECOMMENDATION

B: Based on a single high-quality randomized controlled trial (RCT).

Gurtcheff SE, Turok DK, Stoddard G, et al. Lactogenesis after early postpartum use of the contraceptive implant. Obstet Gynecol. 2011;117:1114-1121.

 

ILLUSTRATIVE CASE

In the last trimester of pregnancy, a patient asks about her options for postpartum contraception. She plans to breastfeed and does not want to have another child for several years, she says. Her family is scheduled to move 2 weeks after her due date, and she wants to begin using contraception before then. She’s interested in the etonogestrel implant (Implanon) and wonders whether she can have it inserted before she leaves the hospital. What can you tell her?

Approximately 4 million women give birth each year in the United States,2 77% of whom choose to breastfeed their babies.3 Postpartum contraception is recommended, to ensure adequate spacing between (or prevention of) pregnancies.

Hormonal options are limited for nursing moms
Due to the negative effect of estrogens on lactation,4 women who wish to use birth control while breastfeeding have limited choices. Their options include progestin-only oral contraceptives; intrauterine devices, including the levonorgestrel intrauterine contraceptive; barrier methods; and the etonogestrel implant. Yet concerns remain that using a progestin contraceptive in the early postpartum period could negatively affect lactogenesis, as well as the quantity and quality of the breast milk.5

Starting contraception after 6 weeks? The opportunity is often missed
A 2010 systematic review found that progestin-only contraception can be safely used in breastfeeding women. However, the studies included in the review did not consider timing. Thus, the researchers concluded only that initiation of a progestin contraception >6 weeks postpartum is safe.6 The World Health Organization recommends waiting >6 weeks, as well.7 But studies have found that between 10% and 40% of women miss their 6-week postpartum visit,8 thereby missing the opportunity to start contraception.

A 2009 pilot study found that the implant can be safely used <4 weeks postpartum, and did not affect breastfeeding.9 The study we review here is the first RCT to evaluate the impact of early insertion (1-3 days postpartum) of the etonogestrel implant on lactogenesis.

STUDY SUMMARY: Timing of implant did not affect outcomes

The study by Gurtcheff et al was a randomized controlled noninferiority trial of 69 women who wanted to use Implanon for postpartum birth control. Inclusion criteria included good health (of the babies as well as the mothers), the intention to breastfeed, and the willingness to be randomly assigned to either early (1-3 days) or standard (4-8 weeks) insertion. The study was not blinded. No other source of bias was identified.

The primary outcomes studied were time to stage II of lactogenesis (based on maternal perception of when her milk “had come in”) and rates of lactation failure.

Early insertion, the researchers found, was noninferior to standard insertion, both in the time to stage II of lactogenesis and the risk of lactation failure. The time to lactogenesis was 64.3 hours (mean standard deviation [SD], 19.6 hours) for early insertion vs 65.2 hours (mean SD, 18.5 hours) for standard insertion. The mean difference was -1.4 hours (95% confidence interval [CI], -10.6 to 7.7 hours). For lactation failure, the absolute risk difference was 0.03 (95% CI, -0.02 to 0.08).

Secondary outcomes included breastfeeding status, side effects, and bleeding patterns, as well as the contraceptive method actually being used at the time. This information was gathered at 2 weeks, 6 weeks, 3 months, and 6 months postpartum.

There were no statistically significant differences in breastfeeding, formula supplementation, or patient-reported bleeding patterns. However, a third of the women (11 of 34) in the standard group did not have the implant inserted, and opted for an alternate form of birth control.

At 6 weeks, women in both groups had a milk sample analyzed for fat and energy content. There was no significant difference in mean creamatocrit values between the groups.

 

 

 

WHAT’S NEW: Early insertion is safe and fosters compliance

Lactogenesis and lactation failure rates were comparable, whether the etonogestrel implant was inserted between 1 and 3 days postpartum or 4 to 8 weeks postpartum. An advantage of early insertion was increased contraceptive compliance. At 3 months postpartum, 13% of the women in the standard group were not using any birth control. Among those in the early insertion group, compliance was 100%.

CAVEATS: Study sample may not be representative

This was a small study, but it was powered to detect ≥8 hour difference in onset of stage II lactogenesis. Participants were not representative of all populations (91% were white, 73% of whom were Hispanic). Both the mothers and babies were healthy, so we can’t extrapolate to situations where either mom or baby is sick.

CHALLENGES TO IMPLEMENTATION: Finding clinicians trained in insertion technique

Health care providers trained in insertion of the etonogestrel implant would need to be available to promote insertion in the early postpartum period. Ensuring availability of the device in hospitals may require extra logistical planning; incorporating etonogestrel implant insertion into already-hectic morning rounds may be challenging, as well.

Acknowledgement

The PURLs Surveillance System is supported in part by Grant Number UL1RR024999 from the National Center for Research Resources, a Clinical Translational Science Award to the University of Chicago. The content is solely the responsibility of the authors and does not necessarily represent the official views of the National Center for Research Resources or the National Institutes of Health.

Click here to view PURL METHODOLOGY

PRACTICE CHANGER

Recommend the etonogestrel implant to new mothers who plan to breastfeed; the insertion of this contraceptive within the first few days postpartum does not alter breastfeeding outcomes.1

STRENGTH OF RECOMMENDATION

B: Based on a single high-quality randomized controlled trial (RCT).

Gurtcheff SE, Turok DK, Stoddard G, et al. Lactogenesis after early postpartum use of the contraceptive implant. Obstet Gynecol. 2011;117:1114-1121.

 

ILLUSTRATIVE CASE

In the last trimester of pregnancy, a patient asks about her options for postpartum contraception. She plans to breastfeed and does not want to have another child for several years, she says. Her family is scheduled to move 2 weeks after her due date, and she wants to begin using contraception before then. She’s interested in the etonogestrel implant (Implanon) and wonders whether she can have it inserted before she leaves the hospital. What can you tell her?

Approximately 4 million women give birth each year in the United States,2 77% of whom choose to breastfeed their babies.3 Postpartum contraception is recommended, to ensure adequate spacing between (or prevention of) pregnancies.

Hormonal options are limited for nursing moms
Due to the negative effect of estrogens on lactation,4 women who wish to use birth control while breastfeeding have limited choices. Their options include progestin-only oral contraceptives; intrauterine devices, including the levonorgestrel intrauterine contraceptive; barrier methods; and the etonogestrel implant. Yet concerns remain that using a progestin contraceptive in the early postpartum period could negatively affect lactogenesis, as well as the quantity and quality of the breast milk.5

Starting contraception after 6 weeks? The opportunity is often missed
A 2010 systematic review found that progestin-only contraception can be safely used in breastfeeding women. However, the studies included in the review did not consider timing. Thus, the researchers concluded only that initiation of a progestin contraception >6 weeks postpartum is safe.6 The World Health Organization recommends waiting >6 weeks, as well.7 But studies have found that between 10% and 40% of women miss their 6-week postpartum visit,8 thereby missing the opportunity to start contraception.

A 2009 pilot study found that the implant can be safely used <4 weeks postpartum, and did not affect breastfeeding.9 The study we review here is the first RCT to evaluate the impact of early insertion (1-3 days postpartum) of the etonogestrel implant on lactogenesis.

STUDY SUMMARY: Timing of implant did not affect outcomes

The study by Gurtcheff et al was a randomized controlled noninferiority trial of 69 women who wanted to use Implanon for postpartum birth control. Inclusion criteria included good health (of the babies as well as the mothers), the intention to breastfeed, and the willingness to be randomly assigned to either early (1-3 days) or standard (4-8 weeks) insertion. The study was not blinded. No other source of bias was identified.

The primary outcomes studied were time to stage II of lactogenesis (based on maternal perception of when her milk “had come in”) and rates of lactation failure.

Early insertion, the researchers found, was noninferior to standard insertion, both in the time to stage II of lactogenesis and the risk of lactation failure. The time to lactogenesis was 64.3 hours (mean standard deviation [SD], 19.6 hours) for early insertion vs 65.2 hours (mean SD, 18.5 hours) for standard insertion. The mean difference was -1.4 hours (95% confidence interval [CI], -10.6 to 7.7 hours). For lactation failure, the absolute risk difference was 0.03 (95% CI, -0.02 to 0.08).

Secondary outcomes included breastfeeding status, side effects, and bleeding patterns, as well as the contraceptive method actually being used at the time. This information was gathered at 2 weeks, 6 weeks, 3 months, and 6 months postpartum.

There were no statistically significant differences in breastfeeding, formula supplementation, or patient-reported bleeding patterns. However, a third of the women (11 of 34) in the standard group did not have the implant inserted, and opted for an alternate form of birth control.

At 6 weeks, women in both groups had a milk sample analyzed for fat and energy content. There was no significant difference in mean creamatocrit values between the groups.

 

 

 

WHAT’S NEW: Early insertion is safe and fosters compliance

Lactogenesis and lactation failure rates were comparable, whether the etonogestrel implant was inserted between 1 and 3 days postpartum or 4 to 8 weeks postpartum. An advantage of early insertion was increased contraceptive compliance. At 3 months postpartum, 13% of the women in the standard group were not using any birth control. Among those in the early insertion group, compliance was 100%.

CAVEATS: Study sample may not be representative

This was a small study, but it was powered to detect ≥8 hour difference in onset of stage II lactogenesis. Participants were not representative of all populations (91% were white, 73% of whom were Hispanic). Both the mothers and babies were healthy, so we can’t extrapolate to situations where either mom or baby is sick.

CHALLENGES TO IMPLEMENTATION: Finding clinicians trained in insertion technique

Health care providers trained in insertion of the etonogestrel implant would need to be available to promote insertion in the early postpartum period. Ensuring availability of the device in hospitals may require extra logistical planning; incorporating etonogestrel implant insertion into already-hectic morning rounds may be challenging, as well.

Acknowledgement

The PURLs Surveillance System is supported in part by Grant Number UL1RR024999 from the National Center for Research Resources, a Clinical Translational Science Award to the University of Chicago. The content is solely the responsibility of the authors and does not necessarily represent the official views of the National Center for Research Resources or the National Institutes of Health.

Click here to view PURL METHODOLOGY

References

1. Gurtcheff SE, Turok DK, Stoddard G, et al. Lactogenesis after early postpartum use of the contraceptive implant. Obstet Gynecol. 2011;117:1114-1121.

2. American Pregnancy Association. Statistics. Available at: http://www.americanpregnancy.org/main/statistics/html. Accessed November 16, 2011.

3. Centers for Disease Control and Prevention. NCHS data brief. Breastfeeding in the United States: findings from the National Health and Nutrition Examination Survey, 1999-2006. April 2008. Available at: http://www.cdc.gov/nchs/data/databriefs/db05.htm. Accessed November 16, 2011.

4. Tankeyoon M, Dusitsin N, Chalapati S, et al. Effects of hormonal contraceptives on milk volume and infant growth. WHO special programme of research, development and research training in human reproduction task force on oral contraceptives. Contraception. 1984;30:505-522.

5. Kennedy KI, Short RV, Tully MR. Premature introduction of progestin-only contraceptive methods during lactation. Contraception. 1997;55:347-350.

6. Kapp N, Curtis K, Nanda K. Progestogen-only contraceptive use among breastfeeding women: a systematic review. Contraception. 2010;82:17-37.

7. World Health Organization medical eligibility criteria wheel for contraceptive use (2008 update). Available at: http://www.who.int/reproductivehealth/publications/family_planning/wheel_v4_2010_EN.swf. Accessed November 16, 2011.

8. Centers for Disease Control and Prevention (CDC). Postpartum care visits—11 states and New York City, 2004. MMWR Morb Mortal Wkly Rep. 2007;56:1312-1316.Available at: http://www.cdc.gov/mmwr/preview/mmwrhtml/mm5650a2.htm. Accessed November 16, 2011.

9. Brito MB, Ferriani RA, Quintana SM. Safety of the etonogestrel-releasing implant during the immediate postpartum period: a pilot study. Contraception. 2009;80:519-526.

References

1. Gurtcheff SE, Turok DK, Stoddard G, et al. Lactogenesis after early postpartum use of the contraceptive implant. Obstet Gynecol. 2011;117:1114-1121.

2. American Pregnancy Association. Statistics. Available at: http://www.americanpregnancy.org/main/statistics/html. Accessed November 16, 2011.

3. Centers for Disease Control and Prevention. NCHS data brief. Breastfeeding in the United States: findings from the National Health and Nutrition Examination Survey, 1999-2006. April 2008. Available at: http://www.cdc.gov/nchs/data/databriefs/db05.htm. Accessed November 16, 2011.

4. Tankeyoon M, Dusitsin N, Chalapati S, et al. Effects of hormonal contraceptives on milk volume and infant growth. WHO special programme of research, development and research training in human reproduction task force on oral contraceptives. Contraception. 1984;30:505-522.

5. Kennedy KI, Short RV, Tully MR. Premature introduction of progestin-only contraceptive methods during lactation. Contraception. 1997;55:347-350.

6. Kapp N, Curtis K, Nanda K. Progestogen-only contraceptive use among breastfeeding women: a systematic review. Contraception. 2010;82:17-37.

7. World Health Organization medical eligibility criteria wheel for contraceptive use (2008 update). Available at: http://www.who.int/reproductivehealth/publications/family_planning/wheel_v4_2010_EN.swf. Accessed November 16, 2011.

8. Centers for Disease Control and Prevention (CDC). Postpartum care visits—11 states and New York City, 2004. MMWR Morb Mortal Wkly Rep. 2007;56:1312-1316.Available at: http://www.cdc.gov/mmwr/preview/mmwrhtml/mm5650a2.htm. Accessed November 16, 2011.

9. Brito MB, Ferriani RA, Quintana SM. Safety of the etonogestrel-releasing implant during the immediate postpartum period: a pilot study. Contraception. 2009;80:519-526.

Issue
The Journal of Family Practice - 60(12)
Issue
The Journal of Family Practice - 60(12)
Page Number
744-746
Page Number
744-746
Publications
Publications
Topics
Article Type
Display Headline
Offer this contraceptive to breastfeeding new moms
Display Headline
Offer this contraceptive to breastfeeding new moms
Sections
PURLs Copyright

Copyright © 2011 The Family Physicians Inquiries Network.
All rights reserved.

Disallow All Ads
Alternative CME
Article PDF Media

Postprostatectomy incontinence? Here’s help

Article Type
Changed
Mon, 01/14/2019 - 11:33
Display Headline
Postprostatectomy incontinence? Here’s help
PRACTICE CHANGER

Recommend behavioral therapy—incorporating pelvic floor muscle training and bladder control strategies—to men experiencing incontinence after prostatectomy.1

STRENGTH OF RECOMMENDATION

B: Based on a single unblinded randomized controlled trial (RCT).

Goode PS, Burgio KL, Johnson TM, et al. Behavioral therapy with or without biofeedback and pelvic floor electrical stimulation for persistent postprostatectomy incontinence: a randomized controlled trial. JAMA. 2011;305:151-159.1

 

ILLUSTRATIVE CASE

Mr. H, age 75, underwent radical prostatectomy for prostate cancer a year ago. Since then, he leaks urine when he coughs and occasionally has complete loss of bladder control. His lack of control has forced him to curtail many of his normal activities and he wants to know what help you can provide.

Routine prostate-specific antigen (PSA) screening has led to an increase in the diagnosis of localized prostate cancer, and prostatectomy is a common treatment. Approximately 90,000 US men undergo radical prostatectomy each year,2 and most are left with some degree of incontinence.

Years later, bladder control problems remain
Surgical advances have attempted to minimize nerve and bladder neck damage, but some degree of incontinence is still common after radical prostatectomy. According to the 2000 Prostate Cancer Outcomes Study, 80% of men experienced some incontinence 6 months after radical prostatectomy. After 2 years, 68% of men still had some degree of incontinence,3 and 8% reported frequent or total incontinence. Five years after prostatectomy, only 35% of patients reported complete bladder control vs 87% presurgery.4

A 2004 systematic review showed that behavioral therapy (mostly biofeedback) reduced symptoms in patients with all types of urinary incontinence.5 Many studies, including a 2010 Cochrane review,6 have shown that women with incontinence benefit from pelvic floor muscle training. No randomized trials have assessed the benefit of behavioral therapy for men with incontinence related to postprostatectomy—until now.

STUDY SUMMARY: Behavioral therapy has long-term payoff

The study by Goode et al was an RCT of behavioral therapy for men who had urinary incontinence after radical prostatectomy and whose symptoms persisted more than a year later. It included patients with ≥3 episodes of incontinence per week. Men were excluded if they were undergoing other treatment for prostate cancer, had a high postvoid residual volume or a history of certain bladder surgeries, or were unable to reliably report symptoms. Participants were permitted to continue taking medication for incontinence, with the exception of anticholinergics.

Participants (n=208 from 3 sites) were randomized to one of 3 arms in a blinded fashion with concealed allocation: behavioral therapy alone, behavioral therapy and biofeedback with electrical stimulation, or a control group of men who could elect to try these therapies at a later date.

Behavioral therapy consisted of 4 visits with a physician or nurse practitioner over an 8-week period. At the first visit, patients received instruction in pelvic floor muscle training. Patients then practiced contraction and relaxation exercises and urinary flow control at home. At the second visit, patients learned techniques to avoid episodes of incontinence, such as performing pelvic muscle contractions with stress symptom triggers. During the final 2 visits, patients received advice about control of persistent problems identified in symptom diaries they were required to keep. In addition, continued daily pelvic floor exercises were recommended at the last session.

Men in the group that received biofeedback with electrical stimulation had the same visit schedule, but received additional pelvic floor training—with electrode-mediated feedback and electrical stimulation of pelvic floor muscles during each visit and daily at home. Patients in the control group had the same visit frequency but received no treatment. After 8 weeks, however, the controls were given the opportunity to try behavioral therapy.

Baseline characteristics and attrition rates were similar in all 3 groups. Outcomes were based on an intention-to-treat analysis. At 8 weeks, men receiving behavioral therapy, with or without electrical stimulation and biofeedback, experienced a 55% decrease in incontinence (from 28 episodes per week at baseline to 13 per week); patients in the control group had a 24% decrease (from 25 episodes to 20 per week) (P=.001). More patients in the behavioral groups were completely continent at 8 weeks (16% vs 6% for the controls); the number needed to treat to achieve complete continence was 10. Electrical stimulation and biofeedback provided no added benefit compared with behavioral therapy alone.

Patients in the 2 treatment groups also had clinically significant benefits in some quality-of-life measures (impact of urinary symptoms on travel, emotion, and voiding) and in symptom-specific quality-of-life scores. Patient satisfaction at 8 weeks was higher in the treatment groups: 26 of the 58 men who received behavioral therapy were “delighted, pleased, or mostly satisfied,” vs 9 of 60 in the control group (P=.006 for overall group difference).

 

 

 

Adherence to the behavioral therapy protocol was 100% at 8 weeks and remained high (91%) one year later. Improvement in symptoms continued at one year, with patients in both treatment groups reporting a clinically significant (50%) reduction in incontinence episodes compared with baseline.

WHAT’S NEW: We have evidence-based help for postprostatectomy incontinence

We now have evidence that an 8-week program of pelvic floor training and bladder control strategies reduces the frequency of incontinence in men who have undergone radical prostatectomy.

CAVEATS: The effects of time weren’t factored in

Patients were obviously aware of group assignment, so there is the possibility of treatment bias contributing to the positive self-reported outcomes. While the treatment groups showed both a greater initial improvement and persistent improvement in their symptoms at one year, symptoms of patients in the control group were not measured after a year, so the sustained improvement could reflect resolution of incontinence with time.

CHALLENGES TO IMPLEMENTATION: Locating clinicians who can train patients

The type of behavioral therapy featured in this study may not be easily accessible to all patients. The researchers suggest consulting the National Association for Continence (http://www.nafc.org), a private nonprofit organization whose members include physical therapists, nurses, and physicians. They also cite the Wound Ostomy and Continence Nurses Society (http://www.wocn.org) as a resource in locating nurses who provide these services.

Acknowledgement

The PURLs Surveillance System is supported in part by Grant Number UL1RR024999 from the National Center for Research Resources, a Clinical Translational Science Award to the University of Chicago. The content is solely the responsibility of the authors and does not necessarily represent the official views of the National Center for Research Resources or the National Institutes of Health.

Click here to view PURL METHODOLOGY

References

1. Goode PS, Burgio KL, Johnson TM, et al. Behavioral therapy with or without biofeedback and pelvic floor electrical stimulation for persistent postprostatectomy incontinence: a randomized controlled trial. JAMA. 2011;305:151-159.

2. Barbash GI, Glied SA. New technology and health care costs—the case of robot-assisted surgery. N Engl J Med. 2010;363:701-704.

3. Stanford JL, Feng Z, Hamilton AS, et al. Urinary and sexual function after radical prostatectomy for clinically localized prostate cancer: the Prostate Cancer Outcomes Study. JAMA. 2000;283:354-360.

4. Penson DF, McLerran D, Feng Z, et al. 5-year urinary and sexual outcomes after radical prostatectomy: results from the prostate cancer outcomes study. J Urol. 2005;173:1701-1705.

5. Teunisse TA, de Jonge A, van Weel C, et al. Treating urinary incontinence in the elderly—conservative therapies that work: a systematic review. J Fam Pract. 2004;53:25-30, 32.

6. Dumoulin C, Hay-Smith J. Pelvic floor muscle training versus no treatment, or inactive control treatments, for urinary incontinence in women. Cochrane Database Syst Rev. 2010;(1):CD005654.-

Article PDF
Author and Disclosure Information

Umang Sharma, MD
The University of Chicago

PURLs EDITOR
John Hickner, MD, MSc
Cleveland Clinic

Issue
The Journal of Family Practice - 60(12)
Publications
Topics
Page Number
741-743
Sections
Author and Disclosure Information

Umang Sharma, MD
The University of Chicago

PURLs EDITOR
John Hickner, MD, MSc
Cleveland Clinic

Author and Disclosure Information

Umang Sharma, MD
The University of Chicago

PURLs EDITOR
John Hickner, MD, MSc
Cleveland Clinic

Article PDF
Article PDF
PRACTICE CHANGER

Recommend behavioral therapy—incorporating pelvic floor muscle training and bladder control strategies—to men experiencing incontinence after prostatectomy.1

STRENGTH OF RECOMMENDATION

B: Based on a single unblinded randomized controlled trial (RCT).

Goode PS, Burgio KL, Johnson TM, et al. Behavioral therapy with or without biofeedback and pelvic floor electrical stimulation for persistent postprostatectomy incontinence: a randomized controlled trial. JAMA. 2011;305:151-159.1

 

ILLUSTRATIVE CASE

Mr. H, age 75, underwent radical prostatectomy for prostate cancer a year ago. Since then, he leaks urine when he coughs and occasionally has complete loss of bladder control. His lack of control has forced him to curtail many of his normal activities and he wants to know what help you can provide.

Routine prostate-specific antigen (PSA) screening has led to an increase in the diagnosis of localized prostate cancer, and prostatectomy is a common treatment. Approximately 90,000 US men undergo radical prostatectomy each year,2 and most are left with some degree of incontinence.

Years later, bladder control problems remain
Surgical advances have attempted to minimize nerve and bladder neck damage, but some degree of incontinence is still common after radical prostatectomy. According to the 2000 Prostate Cancer Outcomes Study, 80% of men experienced some incontinence 6 months after radical prostatectomy. After 2 years, 68% of men still had some degree of incontinence,3 and 8% reported frequent or total incontinence. Five years after prostatectomy, only 35% of patients reported complete bladder control vs 87% presurgery.4

A 2004 systematic review showed that behavioral therapy (mostly biofeedback) reduced symptoms in patients with all types of urinary incontinence.5 Many studies, including a 2010 Cochrane review,6 have shown that women with incontinence benefit from pelvic floor muscle training. No randomized trials have assessed the benefit of behavioral therapy for men with incontinence related to postprostatectomy—until now.

STUDY SUMMARY: Behavioral therapy has long-term payoff

The study by Goode et al was an RCT of behavioral therapy for men who had urinary incontinence after radical prostatectomy and whose symptoms persisted more than a year later. It included patients with ≥3 episodes of incontinence per week. Men were excluded if they were undergoing other treatment for prostate cancer, had a high postvoid residual volume or a history of certain bladder surgeries, or were unable to reliably report symptoms. Participants were permitted to continue taking medication for incontinence, with the exception of anticholinergics.

Participants (n=208 from 3 sites) were randomized to one of 3 arms in a blinded fashion with concealed allocation: behavioral therapy alone, behavioral therapy and biofeedback with electrical stimulation, or a control group of men who could elect to try these therapies at a later date.

Behavioral therapy consisted of 4 visits with a physician or nurse practitioner over an 8-week period. At the first visit, patients received instruction in pelvic floor muscle training. Patients then practiced contraction and relaxation exercises and urinary flow control at home. At the second visit, patients learned techniques to avoid episodes of incontinence, such as performing pelvic muscle contractions with stress symptom triggers. During the final 2 visits, patients received advice about control of persistent problems identified in symptom diaries they were required to keep. In addition, continued daily pelvic floor exercises were recommended at the last session.

Men in the group that received biofeedback with electrical stimulation had the same visit schedule, but received additional pelvic floor training—with electrode-mediated feedback and electrical stimulation of pelvic floor muscles during each visit and daily at home. Patients in the control group had the same visit frequency but received no treatment. After 8 weeks, however, the controls were given the opportunity to try behavioral therapy.

Baseline characteristics and attrition rates were similar in all 3 groups. Outcomes were based on an intention-to-treat analysis. At 8 weeks, men receiving behavioral therapy, with or without electrical stimulation and biofeedback, experienced a 55% decrease in incontinence (from 28 episodes per week at baseline to 13 per week); patients in the control group had a 24% decrease (from 25 episodes to 20 per week) (P=.001). More patients in the behavioral groups were completely continent at 8 weeks (16% vs 6% for the controls); the number needed to treat to achieve complete continence was 10. Electrical stimulation and biofeedback provided no added benefit compared with behavioral therapy alone.

Patients in the 2 treatment groups also had clinically significant benefits in some quality-of-life measures (impact of urinary symptoms on travel, emotion, and voiding) and in symptom-specific quality-of-life scores. Patient satisfaction at 8 weeks was higher in the treatment groups: 26 of the 58 men who received behavioral therapy were “delighted, pleased, or mostly satisfied,” vs 9 of 60 in the control group (P=.006 for overall group difference).

 

 

 

Adherence to the behavioral therapy protocol was 100% at 8 weeks and remained high (91%) one year later. Improvement in symptoms continued at one year, with patients in both treatment groups reporting a clinically significant (50%) reduction in incontinence episodes compared with baseline.

WHAT’S NEW: We have evidence-based help for postprostatectomy incontinence

We now have evidence that an 8-week program of pelvic floor training and bladder control strategies reduces the frequency of incontinence in men who have undergone radical prostatectomy.

CAVEATS: The effects of time weren’t factored in

Patients were obviously aware of group assignment, so there is the possibility of treatment bias contributing to the positive self-reported outcomes. While the treatment groups showed both a greater initial improvement and persistent improvement in their symptoms at one year, symptoms of patients in the control group were not measured after a year, so the sustained improvement could reflect resolution of incontinence with time.

CHALLENGES TO IMPLEMENTATION: Locating clinicians who can train patients

The type of behavioral therapy featured in this study may not be easily accessible to all patients. The researchers suggest consulting the National Association for Continence (http://www.nafc.org), a private nonprofit organization whose members include physical therapists, nurses, and physicians. They also cite the Wound Ostomy and Continence Nurses Society (http://www.wocn.org) as a resource in locating nurses who provide these services.

Acknowledgement

The PURLs Surveillance System is supported in part by Grant Number UL1RR024999 from the National Center for Research Resources, a Clinical Translational Science Award to the University of Chicago. The content is solely the responsibility of the authors and does not necessarily represent the official views of the National Center for Research Resources or the National Institutes of Health.

Click here to view PURL METHODOLOGY

PRACTICE CHANGER

Recommend behavioral therapy—incorporating pelvic floor muscle training and bladder control strategies—to men experiencing incontinence after prostatectomy.1

STRENGTH OF RECOMMENDATION

B: Based on a single unblinded randomized controlled trial (RCT).

Goode PS, Burgio KL, Johnson TM, et al. Behavioral therapy with or without biofeedback and pelvic floor electrical stimulation for persistent postprostatectomy incontinence: a randomized controlled trial. JAMA. 2011;305:151-159.1

 

ILLUSTRATIVE CASE

Mr. H, age 75, underwent radical prostatectomy for prostate cancer a year ago. Since then, he leaks urine when he coughs and occasionally has complete loss of bladder control. His lack of control has forced him to curtail many of his normal activities and he wants to know what help you can provide.

Routine prostate-specific antigen (PSA) screening has led to an increase in the diagnosis of localized prostate cancer, and prostatectomy is a common treatment. Approximately 90,000 US men undergo radical prostatectomy each year,2 and most are left with some degree of incontinence.

Years later, bladder control problems remain
Surgical advances have attempted to minimize nerve and bladder neck damage, but some degree of incontinence is still common after radical prostatectomy. According to the 2000 Prostate Cancer Outcomes Study, 80% of men experienced some incontinence 6 months after radical prostatectomy. After 2 years, 68% of men still had some degree of incontinence,3 and 8% reported frequent or total incontinence. Five years after prostatectomy, only 35% of patients reported complete bladder control vs 87% presurgery.4

A 2004 systematic review showed that behavioral therapy (mostly biofeedback) reduced symptoms in patients with all types of urinary incontinence.5 Many studies, including a 2010 Cochrane review,6 have shown that women with incontinence benefit from pelvic floor muscle training. No randomized trials have assessed the benefit of behavioral therapy for men with incontinence related to postprostatectomy—until now.

STUDY SUMMARY: Behavioral therapy has long-term payoff

The study by Goode et al was an RCT of behavioral therapy for men who had urinary incontinence after radical prostatectomy and whose symptoms persisted more than a year later. It included patients with ≥3 episodes of incontinence per week. Men were excluded if they were undergoing other treatment for prostate cancer, had a high postvoid residual volume or a history of certain bladder surgeries, or were unable to reliably report symptoms. Participants were permitted to continue taking medication for incontinence, with the exception of anticholinergics.

Participants (n=208 from 3 sites) were randomized to one of 3 arms in a blinded fashion with concealed allocation: behavioral therapy alone, behavioral therapy and biofeedback with electrical stimulation, or a control group of men who could elect to try these therapies at a later date.

Behavioral therapy consisted of 4 visits with a physician or nurse practitioner over an 8-week period. At the first visit, patients received instruction in pelvic floor muscle training. Patients then practiced contraction and relaxation exercises and urinary flow control at home. At the second visit, patients learned techniques to avoid episodes of incontinence, such as performing pelvic muscle contractions with stress symptom triggers. During the final 2 visits, patients received advice about control of persistent problems identified in symptom diaries they were required to keep. In addition, continued daily pelvic floor exercises were recommended at the last session.

Men in the group that received biofeedback with electrical stimulation had the same visit schedule, but received additional pelvic floor training—with electrode-mediated feedback and electrical stimulation of pelvic floor muscles during each visit and daily at home. Patients in the control group had the same visit frequency but received no treatment. After 8 weeks, however, the controls were given the opportunity to try behavioral therapy.

Baseline characteristics and attrition rates were similar in all 3 groups. Outcomes were based on an intention-to-treat analysis. At 8 weeks, men receiving behavioral therapy, with or without electrical stimulation and biofeedback, experienced a 55% decrease in incontinence (from 28 episodes per week at baseline to 13 per week); patients in the control group had a 24% decrease (from 25 episodes to 20 per week) (P=.001). More patients in the behavioral groups were completely continent at 8 weeks (16% vs 6% for the controls); the number needed to treat to achieve complete continence was 10. Electrical stimulation and biofeedback provided no added benefit compared with behavioral therapy alone.

Patients in the 2 treatment groups also had clinically significant benefits in some quality-of-life measures (impact of urinary symptoms on travel, emotion, and voiding) and in symptom-specific quality-of-life scores. Patient satisfaction at 8 weeks was higher in the treatment groups: 26 of the 58 men who received behavioral therapy were “delighted, pleased, or mostly satisfied,” vs 9 of 60 in the control group (P=.006 for overall group difference).

 

 

 

Adherence to the behavioral therapy protocol was 100% at 8 weeks and remained high (91%) one year later. Improvement in symptoms continued at one year, with patients in both treatment groups reporting a clinically significant (50%) reduction in incontinence episodes compared with baseline.

WHAT’S NEW: We have evidence-based help for postprostatectomy incontinence

We now have evidence that an 8-week program of pelvic floor training and bladder control strategies reduces the frequency of incontinence in men who have undergone radical prostatectomy.

CAVEATS: The effects of time weren’t factored in

Patients were obviously aware of group assignment, so there is the possibility of treatment bias contributing to the positive self-reported outcomes. While the treatment groups showed both a greater initial improvement and persistent improvement in their symptoms at one year, symptoms of patients in the control group were not measured after a year, so the sustained improvement could reflect resolution of incontinence with time.

CHALLENGES TO IMPLEMENTATION: Locating clinicians who can train patients

The type of behavioral therapy featured in this study may not be easily accessible to all patients. The researchers suggest consulting the National Association for Continence (http://www.nafc.org), a private nonprofit organization whose members include physical therapists, nurses, and physicians. They also cite the Wound Ostomy and Continence Nurses Society (http://www.wocn.org) as a resource in locating nurses who provide these services.

Acknowledgement

The PURLs Surveillance System is supported in part by Grant Number UL1RR024999 from the National Center for Research Resources, a Clinical Translational Science Award to the University of Chicago. The content is solely the responsibility of the authors and does not necessarily represent the official views of the National Center for Research Resources or the National Institutes of Health.

Click here to view PURL METHODOLOGY

References

1. Goode PS, Burgio KL, Johnson TM, et al. Behavioral therapy with or without biofeedback and pelvic floor electrical stimulation for persistent postprostatectomy incontinence: a randomized controlled trial. JAMA. 2011;305:151-159.

2. Barbash GI, Glied SA. New technology and health care costs—the case of robot-assisted surgery. N Engl J Med. 2010;363:701-704.

3. Stanford JL, Feng Z, Hamilton AS, et al. Urinary and sexual function after radical prostatectomy for clinically localized prostate cancer: the Prostate Cancer Outcomes Study. JAMA. 2000;283:354-360.

4. Penson DF, McLerran D, Feng Z, et al. 5-year urinary and sexual outcomes after radical prostatectomy: results from the prostate cancer outcomes study. J Urol. 2005;173:1701-1705.

5. Teunisse TA, de Jonge A, van Weel C, et al. Treating urinary incontinence in the elderly—conservative therapies that work: a systematic review. J Fam Pract. 2004;53:25-30, 32.

6. Dumoulin C, Hay-Smith J. Pelvic floor muscle training versus no treatment, or inactive control treatments, for urinary incontinence in women. Cochrane Database Syst Rev. 2010;(1):CD005654.-

References

1. Goode PS, Burgio KL, Johnson TM, et al. Behavioral therapy with or without biofeedback and pelvic floor electrical stimulation for persistent postprostatectomy incontinence: a randomized controlled trial. JAMA. 2011;305:151-159.

2. Barbash GI, Glied SA. New technology and health care costs—the case of robot-assisted surgery. N Engl J Med. 2010;363:701-704.

3. Stanford JL, Feng Z, Hamilton AS, et al. Urinary and sexual function after radical prostatectomy for clinically localized prostate cancer: the Prostate Cancer Outcomes Study. JAMA. 2000;283:354-360.

4. Penson DF, McLerran D, Feng Z, et al. 5-year urinary and sexual outcomes after radical prostatectomy: results from the prostate cancer outcomes study. J Urol. 2005;173:1701-1705.

5. Teunisse TA, de Jonge A, van Weel C, et al. Treating urinary incontinence in the elderly—conservative therapies that work: a systematic review. J Fam Pract. 2004;53:25-30, 32.

6. Dumoulin C, Hay-Smith J. Pelvic floor muscle training versus no treatment, or inactive control treatments, for urinary incontinence in women. Cochrane Database Syst Rev. 2010;(1):CD005654.-

Issue
The Journal of Family Practice - 60(12)
Issue
The Journal of Family Practice - 60(12)
Page Number
741-743
Page Number
741-743
Publications
Publications
Topics
Article Type
Display Headline
Postprostatectomy incontinence? Here’s help
Display Headline
Postprostatectomy incontinence? Here’s help
Sections
PURLs Copyright

Copyright © 2011 The Family Physicians Inquiries Network.
All rights reserved.

Disallow All Ads
Alternative CME
Article PDF Media

Ear wax removal: Help patients help themselves

Article Type
Changed
Mon, 01/14/2019 - 13:40
Display Headline
Ear wax removal: Help patients help themselves
PRACTICE CHANGER

Suggest that patients use drops to soften the wax in their ears and a bulb syringe to remove it. Reassure them that the process is safe, easy, and effective.1

B: A single well-designed randomized controlled trial (RCT)

Coppin R, Wicke D, Little P. Randomized trial of bulb syringes for earwax: impact on health service utilization. Ann Fam Med .2011;9:110-114.

 

ILLUSTRATIVE CASE

Alarmed because she recently noticed a decrease in her hearing, a 61-year-old woman requests an urgent visit. When you examine her ears, you find bilateral occlusion with cerumen. The patient says that she’s needed office irrigation multiple times in the past and wants to know how to clean her ears at home to prevent wax build-up. What can you recommend?

Cerumen impaction is associated with a variety of symptoms, including hearing loss, pain, itching, and a feeling of fullness, as well as dizziness, tinnitus, and a reflex cough.2 Eight million ear irrigations are carried out in US medical offices each year.3 Yet there is no reason to believe (and little evidence to suggest) that home irrigation would not be an effective approach.

Drops and wax removal kits are widely available
Patients can purchase wax-softening drops. Carbamide peroxide substances, for instance, are sold under a variety of trade names, such as Auraphene-B, Debrox, Mollifene, and Murine Ear Drops. Mineral oil is a common home remedy, as well, although it has no official indication for ear wax removal.

Home irrigation kits, which typically include a bulb syringe, are sold over the counter and cost anywhere from $3 to $400.4 These prices represent the varying degrees of automation available for cerumen removal, from wax-softening drops and a bulb syringe packed together in a “kit” to systems that connect to the faucet for continuous water pressure and include a temperature sensor. Most kits cost less than $20.

Bulb syringe irrigation is generally considered safe and effective. But it has never been compared with other methods5 and clinicians rarely recommend it, we suspect because of a lack of knowledge of its safety and efficacy.

STUDY SUMMARY: Every 2 patients given wax removal kits = 1 less office visit

Coppin et al conducted a blinded study of adults with cerumen impaction to assess the efficacy of bulb syringe irrigation compared with standard care.1 The authors recruited patients from 7 practices in England. To be eligible for the study, patients had to have symptoms of blockage and visible occluding ear wax. The researchers assessed 434 patients and randomized 237; of these, only 3 were lost to follow-up.

Using concealed allocation, a nurse randomly gave all the patients identical-looking envelopes. Half of the envelopes contained ear drops and instructions in usual care (ear irrigation by a clinician after the use of ear drops). The other half contained ear drops and a 25-mL ear bulb syringe (not available over the counter in the United Kingdom). Instructions provided with the syringes indicated that they could be cleaned and reused, but did not specifically instruct patients as to when to use them. Baseline characteristics were balanced between the 2 groups.

After 2 weeks, the nurse reassessed the patients and irrigated the ears of any patient with evidence of occlusion. The authors used National Health Service computerized records to track ear wax–related visits over the next 2 years for participants in both groups.

During the 2-year follow-up, more of the patients in the control group returned to the clinic with episodes of ear wax compared with those in the intervention group (73% vs 60%; risk ratio=1.21; 95% confidence interval [CI], 1.01-1.37; P=.038).

The researchers also found that, among the returnees, patients in the control group had, on average, 50% more visits. That is, for every 2 patients who were given a bulb syringe, there was one less visit (incidence rate ratio=1.79; 95% CI, 1.05-3.04; P=.032). A secondary analysis found no significant difference in adverse events between the intervention and the control groups.

 

 

 

WHAT’S NEW: Do-it-yourself wax removal is now evidence-based

The American Academy of Otolaryngology-Head and Neck Surgery Foundation’s 2008 clinical practice guideline—based primarily on expert opinion—recommends clinician irrigation only, due to a lack of quality evidence.3

This RCT is the first to provide evidence that some patients do not need to spend time (or money) on a medical visit for ear wax irrigation. The fact that patients who were given bulb syringes had fewer visits, not only for the initial wax removal but also for subsequent episodes of cerumen impaction, suggests that they were self-treating at home without an increase in adverse effects.

CAVEATS: Home irrigation is not for every patient

This intervention cannot be extrapolated to young children or to others who are unable to perform self-irrigation. It is possible that if a patient self-irrigates without prior visualization by a clinician, a contraindication such as ruptured tympanic membrane or active infection could be present.

This study was performed in England, where bulb syringes are not readily available. It is possible that this intervention may be less effective at avoiding cerumen-related office visits in the United States, especially if patients are already using bulb syringes for this purpose. Finally, we note that 60% of the patients in the home irrigation group did return for a visit for cerumen removal during the 2-year follow-up, so home irrigation did not entirely replace office irrigation.

CHALLENGES TO IMPLEMENTATION: Getting buy-in from patients

The greatest challenge to implementation might be convincing patients that they can safely perform self-irrigation at home. This may require written patient instructions, preferably with illustrations. The steps will need to be written clearly and include details such as recommended ear wax softeners, water temperature, use of peroxide (or not), warning symptoms, and when to contact a physician.

A healthy physician-patient relationship, and perhaps, giving patients the bulb syringe and instructions in using it before they leave the clinic, will help to overcome patient hesitancy. Physician inertia may also be a problem, but it should be easy to put this new information into practice once provider resistance is overcome.

Acknowledgement

The PURLs Surveillance System is supported in part by Grant Number UL1RR024999 from the National Center For Research Resources, a Clinical Translational Science Award to the University of Chicago. The content is solely the responsibility of the authors and does not necessarily represent the official views of the National Center For Research Resources or the National Institutes of Health.

Click here to view PURL METHODOLOGY

References

1. Coppin R, Wicke D, Little P. Randomized trial of bulb syringes for earwax: impact on health service utilization. Ann Fam Med. 2011;9:110-114.

2. Mitka M. Cerumen removal guidelines wax practical. JAMA. 2008;300:1506.-

3. Roland PS, Smith TL, Schwartz SR, et al. Clinical practice guideline: cerumen impaction. Otolaryngol Head Neck Surg. 2008;139(suppl 2):S1-S21.

4. Ear irrigation products. Available at: http://www.nextag.com/ear-irrigation/stores-html. Accessed June 6, 2011.

5. Coppin R, Wicke D, Little P. Managing earwax in primary care: efficacy of self-treatment using a bulb syringe. Br J Gen Pract. 2008;58:44-49.

Author and Disclosure Information

Nina Rogers, MD
Department of Family Medicine, The University of Chicago

James J. Stevermer, MD, MSPH
Department of Family and Community Medicine, University of Missouri – Columbia

PURLs EDITOR
Bernard Ewigman, MD, MSPH
The University of Chicago

Issue
The Journal of Family Practice - 60(11)
Publications
Topics
Page Number
671-673
Legacy Keywords
Nina Rogers;MD; James J. Stevermer;MD;MSPH; PURLs; ear wax removal; wax-softening drops; carbamide peroxide; home irrigation kits; bulb syringe; bulb syringe irrigation; cerumen; impaction; ear irrigations
Sections
Author and Disclosure Information

Nina Rogers, MD
Department of Family Medicine, The University of Chicago

James J. Stevermer, MD, MSPH
Department of Family and Community Medicine, University of Missouri – Columbia

PURLs EDITOR
Bernard Ewigman, MD, MSPH
The University of Chicago

Author and Disclosure Information

Nina Rogers, MD
Department of Family Medicine, The University of Chicago

James J. Stevermer, MD, MSPH
Department of Family and Community Medicine, University of Missouri – Columbia

PURLs EDITOR
Bernard Ewigman, MD, MSPH
The University of Chicago

PRACTICE CHANGER

Suggest that patients use drops to soften the wax in their ears and a bulb syringe to remove it. Reassure them that the process is safe, easy, and effective.1

B: A single well-designed randomized controlled trial (RCT)

Coppin R, Wicke D, Little P. Randomized trial of bulb syringes for earwax: impact on health service utilization. Ann Fam Med .2011;9:110-114.

 

ILLUSTRATIVE CASE

Alarmed because she recently noticed a decrease in her hearing, a 61-year-old woman requests an urgent visit. When you examine her ears, you find bilateral occlusion with cerumen. The patient says that she’s needed office irrigation multiple times in the past and wants to know how to clean her ears at home to prevent wax build-up. What can you recommend?

Cerumen impaction is associated with a variety of symptoms, including hearing loss, pain, itching, and a feeling of fullness, as well as dizziness, tinnitus, and a reflex cough.2 Eight million ear irrigations are carried out in US medical offices each year.3 Yet there is no reason to believe (and little evidence to suggest) that home irrigation would not be an effective approach.

Drops and wax removal kits are widely available
Patients can purchase wax-softening drops. Carbamide peroxide substances, for instance, are sold under a variety of trade names, such as Auraphene-B, Debrox, Mollifene, and Murine Ear Drops. Mineral oil is a common home remedy, as well, although it has no official indication for ear wax removal.

Home irrigation kits, which typically include a bulb syringe, are sold over the counter and cost anywhere from $3 to $400.4 These prices represent the varying degrees of automation available for cerumen removal, from wax-softening drops and a bulb syringe packed together in a “kit” to systems that connect to the faucet for continuous water pressure and include a temperature sensor. Most kits cost less than $20.

Bulb syringe irrigation is generally considered safe and effective. But it has never been compared with other methods5 and clinicians rarely recommend it, we suspect because of a lack of knowledge of its safety and efficacy.

STUDY SUMMARY: Every 2 patients given wax removal kits = 1 less office visit

Coppin et al conducted a blinded study of adults with cerumen impaction to assess the efficacy of bulb syringe irrigation compared with standard care.1 The authors recruited patients from 7 practices in England. To be eligible for the study, patients had to have symptoms of blockage and visible occluding ear wax. The researchers assessed 434 patients and randomized 237; of these, only 3 were lost to follow-up.

Using concealed allocation, a nurse randomly gave all the patients identical-looking envelopes. Half of the envelopes contained ear drops and instructions in usual care (ear irrigation by a clinician after the use of ear drops). The other half contained ear drops and a 25-mL ear bulb syringe (not available over the counter in the United Kingdom). Instructions provided with the syringes indicated that they could be cleaned and reused, but did not specifically instruct patients as to when to use them. Baseline characteristics were balanced between the 2 groups.

After 2 weeks, the nurse reassessed the patients and irrigated the ears of any patient with evidence of occlusion. The authors used National Health Service computerized records to track ear wax–related visits over the next 2 years for participants in both groups.

During the 2-year follow-up, more of the patients in the control group returned to the clinic with episodes of ear wax compared with those in the intervention group (73% vs 60%; risk ratio=1.21; 95% confidence interval [CI], 1.01-1.37; P=.038).

The researchers also found that, among the returnees, patients in the control group had, on average, 50% more visits. That is, for every 2 patients who were given a bulb syringe, there was one less visit (incidence rate ratio=1.79; 95% CI, 1.05-3.04; P=.032). A secondary analysis found no significant difference in adverse events between the intervention and the control groups.

 

 

 

WHAT’S NEW: Do-it-yourself wax removal is now evidence-based

The American Academy of Otolaryngology-Head and Neck Surgery Foundation’s 2008 clinical practice guideline—based primarily on expert opinion—recommends clinician irrigation only, due to a lack of quality evidence.3

This RCT is the first to provide evidence that some patients do not need to spend time (or money) on a medical visit for ear wax irrigation. The fact that patients who were given bulb syringes had fewer visits, not only for the initial wax removal but also for subsequent episodes of cerumen impaction, suggests that they were self-treating at home without an increase in adverse effects.

CAVEATS: Home irrigation is not for every patient

This intervention cannot be extrapolated to young children or to others who are unable to perform self-irrigation. It is possible that if a patient self-irrigates without prior visualization by a clinician, a contraindication such as ruptured tympanic membrane or active infection could be present.

This study was performed in England, where bulb syringes are not readily available. It is possible that this intervention may be less effective at avoiding cerumen-related office visits in the United States, especially if patients are already using bulb syringes for this purpose. Finally, we note that 60% of the patients in the home irrigation group did return for a visit for cerumen removal during the 2-year follow-up, so home irrigation did not entirely replace office irrigation.

CHALLENGES TO IMPLEMENTATION: Getting buy-in from patients

The greatest challenge to implementation might be convincing patients that they can safely perform self-irrigation at home. This may require written patient instructions, preferably with illustrations. The steps will need to be written clearly and include details such as recommended ear wax softeners, water temperature, use of peroxide (or not), warning symptoms, and when to contact a physician.

A healthy physician-patient relationship, and perhaps, giving patients the bulb syringe and instructions in using it before they leave the clinic, will help to overcome patient hesitancy. Physician inertia may also be a problem, but it should be easy to put this new information into practice once provider resistance is overcome.

Acknowledgement

The PURLs Surveillance System is supported in part by Grant Number UL1RR024999 from the National Center For Research Resources, a Clinical Translational Science Award to the University of Chicago. The content is solely the responsibility of the authors and does not necessarily represent the official views of the National Center For Research Resources or the National Institutes of Health.

Click here to view PURL METHODOLOGY

PRACTICE CHANGER

Suggest that patients use drops to soften the wax in their ears and a bulb syringe to remove it. Reassure them that the process is safe, easy, and effective.1

B: A single well-designed randomized controlled trial (RCT)

Coppin R, Wicke D, Little P. Randomized trial of bulb syringes for earwax: impact on health service utilization. Ann Fam Med .2011;9:110-114.

 

ILLUSTRATIVE CASE

Alarmed because she recently noticed a decrease in her hearing, a 61-year-old woman requests an urgent visit. When you examine her ears, you find bilateral occlusion with cerumen. The patient says that she’s needed office irrigation multiple times in the past and wants to know how to clean her ears at home to prevent wax build-up. What can you recommend?

Cerumen impaction is associated with a variety of symptoms, including hearing loss, pain, itching, and a feeling of fullness, as well as dizziness, tinnitus, and a reflex cough.2 Eight million ear irrigations are carried out in US medical offices each year.3 Yet there is no reason to believe (and little evidence to suggest) that home irrigation would not be an effective approach.

Drops and wax removal kits are widely available
Patients can purchase wax-softening drops. Carbamide peroxide substances, for instance, are sold under a variety of trade names, such as Auraphene-B, Debrox, Mollifene, and Murine Ear Drops. Mineral oil is a common home remedy, as well, although it has no official indication for ear wax removal.

Home irrigation kits, which typically include a bulb syringe, are sold over the counter and cost anywhere from $3 to $400.4 These prices represent the varying degrees of automation available for cerumen removal, from wax-softening drops and a bulb syringe packed together in a “kit” to systems that connect to the faucet for continuous water pressure and include a temperature sensor. Most kits cost less than $20.

Bulb syringe irrigation is generally considered safe and effective. But it has never been compared with other methods5 and clinicians rarely recommend it, we suspect because of a lack of knowledge of its safety and efficacy.

STUDY SUMMARY: Every 2 patients given wax removal kits = 1 less office visit

Coppin et al conducted a blinded study of adults with cerumen impaction to assess the efficacy of bulb syringe irrigation compared with standard care.1 The authors recruited patients from 7 practices in England. To be eligible for the study, patients had to have symptoms of blockage and visible occluding ear wax. The researchers assessed 434 patients and randomized 237; of these, only 3 were lost to follow-up.

Using concealed allocation, a nurse randomly gave all the patients identical-looking envelopes. Half of the envelopes contained ear drops and instructions in usual care (ear irrigation by a clinician after the use of ear drops). The other half contained ear drops and a 25-mL ear bulb syringe (not available over the counter in the United Kingdom). Instructions provided with the syringes indicated that they could be cleaned and reused, but did not specifically instruct patients as to when to use them. Baseline characteristics were balanced between the 2 groups.

After 2 weeks, the nurse reassessed the patients and irrigated the ears of any patient with evidence of occlusion. The authors used National Health Service computerized records to track ear wax–related visits over the next 2 years for participants in both groups.

During the 2-year follow-up, more of the patients in the control group returned to the clinic with episodes of ear wax compared with those in the intervention group (73% vs 60%; risk ratio=1.21; 95% confidence interval [CI], 1.01-1.37; P=.038).

The researchers also found that, among the returnees, patients in the control group had, on average, 50% more visits. That is, for every 2 patients who were given a bulb syringe, there was one less visit (incidence rate ratio=1.79; 95% CI, 1.05-3.04; P=.032). A secondary analysis found no significant difference in adverse events between the intervention and the control groups.

 

 

 

WHAT’S NEW: Do-it-yourself wax removal is now evidence-based

The American Academy of Otolaryngology-Head and Neck Surgery Foundation’s 2008 clinical practice guideline—based primarily on expert opinion—recommends clinician irrigation only, due to a lack of quality evidence.3

This RCT is the first to provide evidence that some patients do not need to spend time (or money) on a medical visit for ear wax irrigation. The fact that patients who were given bulb syringes had fewer visits, not only for the initial wax removal but also for subsequent episodes of cerumen impaction, suggests that they were self-treating at home without an increase in adverse effects.

CAVEATS: Home irrigation is not for every patient

This intervention cannot be extrapolated to young children or to others who are unable to perform self-irrigation. It is possible that if a patient self-irrigates without prior visualization by a clinician, a contraindication such as ruptured tympanic membrane or active infection could be present.

This study was performed in England, where bulb syringes are not readily available. It is possible that this intervention may be less effective at avoiding cerumen-related office visits in the United States, especially if patients are already using bulb syringes for this purpose. Finally, we note that 60% of the patients in the home irrigation group did return for a visit for cerumen removal during the 2-year follow-up, so home irrigation did not entirely replace office irrigation.

CHALLENGES TO IMPLEMENTATION: Getting buy-in from patients

The greatest challenge to implementation might be convincing patients that they can safely perform self-irrigation at home. This may require written patient instructions, preferably with illustrations. The steps will need to be written clearly and include details such as recommended ear wax softeners, water temperature, use of peroxide (or not), warning symptoms, and when to contact a physician.

A healthy physician-patient relationship, and perhaps, giving patients the bulb syringe and instructions in using it before they leave the clinic, will help to overcome patient hesitancy. Physician inertia may also be a problem, but it should be easy to put this new information into practice once provider resistance is overcome.

Acknowledgement

The PURLs Surveillance System is supported in part by Grant Number UL1RR024999 from the National Center For Research Resources, a Clinical Translational Science Award to the University of Chicago. The content is solely the responsibility of the authors and does not necessarily represent the official views of the National Center For Research Resources or the National Institutes of Health.

Click here to view PURL METHODOLOGY

References

1. Coppin R, Wicke D, Little P. Randomized trial of bulb syringes for earwax: impact on health service utilization. Ann Fam Med. 2011;9:110-114.

2. Mitka M. Cerumen removal guidelines wax practical. JAMA. 2008;300:1506.-

3. Roland PS, Smith TL, Schwartz SR, et al. Clinical practice guideline: cerumen impaction. Otolaryngol Head Neck Surg. 2008;139(suppl 2):S1-S21.

4. Ear irrigation products. Available at: http://www.nextag.com/ear-irrigation/stores-html. Accessed June 6, 2011.

5. Coppin R, Wicke D, Little P. Managing earwax in primary care: efficacy of self-treatment using a bulb syringe. Br J Gen Pract. 2008;58:44-49.

References

1. Coppin R, Wicke D, Little P. Randomized trial of bulb syringes for earwax: impact on health service utilization. Ann Fam Med. 2011;9:110-114.

2. Mitka M. Cerumen removal guidelines wax practical. JAMA. 2008;300:1506.-

3. Roland PS, Smith TL, Schwartz SR, et al. Clinical practice guideline: cerumen impaction. Otolaryngol Head Neck Surg. 2008;139(suppl 2):S1-S21.

4. Ear irrigation products. Available at: http://www.nextag.com/ear-irrigation/stores-html. Accessed June 6, 2011.

5. Coppin R, Wicke D, Little P. Managing earwax in primary care: efficacy of self-treatment using a bulb syringe. Br J Gen Pract. 2008;58:44-49.

Issue
The Journal of Family Practice - 60(11)
Issue
The Journal of Family Practice - 60(11)
Page Number
671-673
Page Number
671-673
Publications
Publications
Topics
Article Type
Display Headline
Ear wax removal: Help patients help themselves
Display Headline
Ear wax removal: Help patients help themselves
Legacy Keywords
Nina Rogers;MD; James J. Stevermer;MD;MSPH; PURLs; ear wax removal; wax-softening drops; carbamide peroxide; home irrigation kits; bulb syringe; bulb syringe irrigation; cerumen; impaction; ear irrigations
Legacy Keywords
Nina Rogers;MD; James J. Stevermer;MD;MSPH; PURLs; ear wax removal; wax-softening drops; carbamide peroxide; home irrigation kits; bulb syringe; bulb syringe irrigation; cerumen; impaction; ear irrigations
Sections
PURLs Copyright

Copyright © 2011 The Family Physicians Inquiries Network.
All rights reserved.

Disallow All Ads
Alternative CME